Вы находитесь на странице: 1из 174

2009 Adult Spine Self-Assessment Examination

Evaluation and CME

ACCREDITATION
The American Academy of Orthopaedic Surgeons is accredited by the Accreditation Council for
Continuing Medical Education to provide continuing medical education for physicians.

CME CREDIT
U. S. Physicians: The American Academy of Orthopaedic Surgeons designates this educational
activity for a maximum of 10 AMA PRA Category 1 Credits. Physicians should only claim
credits commensurate with the extent of their participation in the activity.

International Physicians: The AMA has determined that physicians not licensed in the United
States but who participate in this CME activity are eligible for AMA PRA Category 1 Credits.

Allied Health Professionals: The Academy is not accredited to offer credit for nurses and other allied
health professionals. To determine if activities offering Category 1 CME credit are acceptable for your
licensing or certification needs please contact the relevant organizations directly.

To obtain AMA PRA Category 1 CreditsTM for this examination, please complete the online
evaluation form and submit no later than December 31, 2011.

1. This educational program (check all that apply)

confirmed that my knowledge in this subject area is up-to-date


presented me with new knowledge on the topic
presented me with new knowledge directly applicable to my practice
stimulated me to gain further information on the topic

2. Did the 2009 Adult Spine Self-Assessment Examination meet the stated objectives?

Differentiate treatment options for fractures involving the cervical, thoracic, and lumbar
spine
Yes No Explain:

Develop clinical decision-making knowledge for treating spinal deformity and


degenerative disorders
Yes No Explain:

Improve knowledge of spinal anatomy and normal anatomic variance to discriminate


normal anatomy from injury or pathology
Yes No Explain:

Develop treatment plans for common injuries to the spine, including fractures (traumatic
and osteoporotic) and nerve compression disorders
Yes No Explain:

3. Did you find the educational content of this examination met your learning needs in this
content area?

Fell short of my expectations


Met my expectations
Exceeded my expectations

4. To what extent will you implement knowledge gained through the 2009 Adult Spine Self-
Assessment Examination activity into your practice within the next year?

Not at all
Some extent
Moderate extent
Great extent

List the changes you will make in your practice within the next year as a result of completing this
examination program:

5. Was the educational value of this examination compromised by author bias?

Yes
No

Explain:

6. Did you observe any instance(s) of commercial bias in the examination (i.e., use of brand
names vs. generic; favoritism toward one companys devices vs. all others)? If so, please
indicate the question(s) below. Yes No

Explain:

Please add any additional comments about this examination that will assist us in planning for
future examinations:

Document your actual number of credits (hours of study) for this activity:
Enter a value of 1-10
ACCREDITATION
The American Academy of Orthopaedic Surgeons is accredited by the Accreditation Council for
Continuing Medical Education to provide continuing medical education for physicians.

CME CREDIT
U. S. Physicians: The American Academy of Orthopaedic Surgeons designates this educational
activity for a maximum of 10 AMA PRA Category 1 Credits. Physicians should only claim
credits commensurate with the extent of their participation in the activity.
International Physicians: The AMA has determined that physicians not licensed in the United
States but who participate in this CME activity are eligible for AMA PRA Category 1 Credits.
Allied Health Professionals: The Academy is not accredited to offer credit for nurses and other allied
health professionals. To determine if activities offering Category 1 CME credit are acceptable for your
licensing or certification needs please contact the relevant organizations directly.

EDUCATIONAL OBJECTIVES
Differentiate treatment options for fractures involving the cervical, thoracic, and lumbar spine
Develop clinical decision-making knowledge for treating spinal deformity and degenerative
disorders
Improve knowledge of spinal anatomy and normal anatomic variance to discriminate normal
anatomy from injury or pathology
Develop treatment plans for common injuries to the spine, including fractures
(traumatic and osteoporotic) and nerve compression disorders

It is the goal of the Academy to promote safe and effective orthopaedic care in all our programs,
products, and services.

Produced by the American Academy of Orthopaedic Surgeons


Adult Spine Evaluation Subcommittee of
the Evaluation Committee

William C. Lauerman, MD, Chairman


Professor of Orthopaedic Surgery, Georgetown University Hospital, Washington, DC
Robert J. Banco, MD
Associate Clinical Professor, Department of Orthopaedic Surgery, Tufts University School of
Medicine, Newton Wellesley Hospital; Orthopaedic Surgeon, The Boston Spine Group, Newton, MA
Jeffrey D. Coe, MD
Orthopaedic Surgeon, Private Practice, Campbell, CA
Cameron B. Huckell, MD
Partner Pinnacle Orthopedic and Spine Specialists PLLC; Assistant Professor of Orthopaedics,
State University of New York at Buffalo, Buffalo, NY
Isador H. Lieberman, MD, MBA, FRCSC
Professor of Surgery, Cleveland Clinic Lerner College of Medicine; Chairman, Medical
Interventional & Surgical Spine Center, Cleveland Clinic Florida, Weston, FL
William J. Richardson, MD
Professor, Orthopaedic Surgery, Vice Chair, Department of Surgery for Clinical Effectiveness,
Durham University Medical Center, Durham, NC
William Sukovich, MD
Director, Spinal Surgery Associates, PLC, Martha Jefferson Hospital, Charlottesville, VA
Steven M. Theiss, MD
Associate Professor, Division of Orthopaedic Surgery, University of Alabama at Birmingham,
Birmingham, AL
Stephen J. Timon, MD
Clinical Assistant Professor, University of Texas at Southwestern Medical Center; Orthopaedic
Spine Surgeon, Private Practice, Dallas, TX
Jed S. Vanichkachorn, MD
Orthopaedic Spine Surgeon, Private Practice, Tuckahoe Orthopaedic Associates, Richmond, VA
Kirkham B. Wood, MD
Associate Professor, Department of Orthopaedic Surgery, Harvard Medical School, Massachusetts
General Hospital, Boston, MA
Michael J. Yaszemski, MD, PhD
Professor of Orthopaedics and Biomedical Engineering, College of Medicine, Mayo Clinic,
Rochester, MN

Disclosure Information for Members of Committee: Adult Spine Evaluation Subcommittee


William C Lauerman, MD (Chair): 4 (DePuy, A Johnson & Johnson Company; Stryker); 5A
(DePuy, A Johnson & Johnson Company). Submitted on: 04/03/2008.
Robert J Banco, MD (Member): 5A (DePuy, A Johnson & Johnson Company; Medtronic Sofamor
Danek; Stryker; Impliant; Surgicraft); 8 (Breakaway Imaging, recently sold to Medtronic). Submitted
on: 11/05/2007 and last confirmed as accurate on 08/13/2008.
Jeffrey Dean Coe, MD (Member): 1 (Cervical Spine Research Society; Allez Spine, LLC); 4
(Nuvasive; PriCara (Ortho-McNeil, J & J)); 5A (N Spine; Arthrocare; Medtronic Sofamor Danek;
Synthes; Nuvasive); 7 (Medtronic Sofamor Danek); 8 (N Spine; Arthrocare). Submitted on:
08/19/2008.
Cameron B Huckell, MD (Member): 2 (Orthopedics); 4 (Synthes); 8 (Johnson & Johnson;
Medtronic). Submitted on: 01/29/2008.
Isador H Lieberman, MD, MBA, FRCSC (Member): 1 (Scoliosis Research Society; Cleveland
Clinic Foundation, Board of Governors;); 2 (Springer; Wolters Kluwer Health - Lippincott Williams
& Wilkins; Journal of Bone and Joint Surgery - American; Orthopedic Clinics of North America;
European Spine Journal; Saunders/Mosby-Elsevier); 3 (AxioMed; DePuy, A Johnson & Johnson
Company; MAZOR Surgical Technologies; Merlot Orthopaedix; Stryker; Trans1; CrossTrees); 5A
(AxioMed; MAZOR Surgical Technologies; Merlot Orthopaedix; Trans1; Crosstrees; Kyphon); 7
(AxioMed; MAZOR Surgical Technologies; Merlot Orthopaedix; Trans1; Crosstrees; Kyphon;
Medtronic; Orthovita; Stryker; ; Abbott; DePuy, A Johnson & Johnson Company); 8 (AxioMed;
MAZOR Surgical Technologies; Merlot Orthopaedix; Trans1; Crosstrees;); 10 (AxioMed; MAZOR
Surgical Technologies; Merlot Orthopaedix; Trans1; Crosstrees;). Submitted on: 04/03/2008.
William J Richardson, MD (Member): 6 (Wheeless Text Book of Surgery Online); 7 (EBI; DePuy,
A Johnson & Johnson Company). Submitted on: 11/19/2007 and last confirmed as accurate on
08/08/2008.
William Sukovich, MD (Member): 8 (HydroCision; Nuvasive). Submitted on: 04/03/2008.
Steven M Theiss, MD (Member): 4 (Biomet; Synthes); 5A (Biomet; Synthes); 7 (Medtronic
Sofamor Danek; Synthes). Submitted on: 08/28/2007 and last confirmed as accurate on 08/19/2008.
Stephen J Timon, MD (Member): 1 (Irving Coppell Surgical Hospital Pine Creek Medical Center);
3 (Seaspine); 5A (Seaspine Stryker Vertbebron). Submitted on: 08/19/2008.
Jed S Vanichkachorn, MD (Member): 5A (Blackstone Medical). Submitted on: 11/19/2007 and last
confirmed as accurate on 04/28/2008.
Kirkham B Wood, MD (Member): 2 (Spine; Journal of Trauma); 4 (DePuy, A Johnson & Johnson
Company); 5A (Inion, Inc.); 7 (Blackstone Medical; DePuy, A Johnson & Johnson Company;
Medtronic Sofamor Danek; Synthes; Globus Medical); 8 (TranS1; Inion Inc.). Submitted on:
08/19/2008.
Michael J Yaszemski, MD, PhD (Member): 1 (Scoliosis Research Society; Society of Military
Orthopaedic Surgeons); 2 (Journal of Biomedical Materials Research-Part A; Biomaterials); 5A
(IsoTis Orthobiologics; National Institutes of Health (NIAMS & NICHD); Osteotech;
Wyeth;BonWRX); 6 (None); 7 (Health and Human Services; National Institutes of Health (NIAMS
& NICHD)). Submitted on: 10/30/2007.

Disclosure Items: (n) = Respondent answered 'No' to all items indicating no conflicts. 1= Board
member/owner/officer/committee appointments; 2= Medical/Orthopaedic Publications; 3= Royalties; 4=
Speakers bureau/paid presentations; 5A= Paid consultant; 5B= Unpaid consultant; 6= Research or
institutional support from a publisher; 7= Research or institutional support from a company or supplier;
8= Stock or Stock Options; 9= Other financial/material support from a publisher; 10= Other
financial/material support from a company or supplier.

American Academy of Orthopaedic Surgeons


Mark Wieting, Chief Education Officer
Howard Mevis, Director, Electronic Media, Evaluation & Course Operations
Laura Hruska, Examinations Manager
Marcie L. Lampert, Senior Examinations Coordinator

Editorial Staff
Marilyn L. Fox, PhD, Director, Publications
Gayle Murray, Associate Senior Editor

Disclaimer
The material presented in this special interest examination has been made available by the American
Academy of Orthopaedic Surgeons for educational purposes only. This material is not intended to
present the only, or necessarily the best, methods or procedures for the medical situations discussed, but
rather is intended to represent an approach, view, statement, or opinion of the author(s) or producer(s),
which may be helpful to others who face similar situations.

Some drugs or medical devices demonstrated in Academy courses or described in Academy print or
electronic publications have not been cleared by the Food and Drug Administration (FDA) or have been
cleared for specific uses only. The FDA has stated that it is the responsibility of the physician to
determine the FDA clearance status of each drug or device he or she wishes to use in clinical practice.

At the time of this writing, bone screws placed posteriorly into vertebral elements have been cleared for
use in this specific manner by the Food and Drug Administration (FDA) to provide immobilization and
stabilization as an adjunct to fusion in the treatment of the following acute and chronic instability or
deformities of the thoracic, lumbar, and sacral spine: degenerative spondylolisthesis with objective
evidence of neurologic impairment; fracture; dislocation; scoliosis; kyphosis; spinal tumor, and failed
previous fusion (pseudarthrosis). In addition, anterior vertebral body screws (cervical, thoracic, and
lumbar) are Class II devices and can be used as labeled in vertebral bodies.

Furthermore, any statements about commercial products are solely the opinion(s) of the author(s) and do
not represent an Academy endorsement or evaluation of these products. These statements may not be
used in advertising or for any commercial purpose.

Copyright 2009
by the American Academy of Orthopaedic Surgeons

All rights reserved. No part of Adult Spine Self-Assessment Examination may be reproduced, stored in a
retrieval system, or transmitted in any form or by any means, electronic, mechanical, photocopying,
recording, or otherwise without the prior written permission of the publisher.
Questions
1. Which is the best initial study for the diagnostic evaluation of diskogenic low back pain?

1- MRI
2- Diskography
3- CT-diskography
4- Radiography
5- CT

2. A patient who is an observant Jehovahs Witness requires major surgery for scoliosis that
will likely result in significant blood loss. Which of the following might the patient consider
allowing the surgical team to use?

1- Transfusion of whole blood


2- Transfusion of packed red blood cells
3- A cell saver with continuity maintained in a closed circuit
4- Transfusion of plasma
5- Transfusion of platelets

3. Which of the following is a contraindication to laminoplasty in a patient with cervical


spondylotic myelopathy?

1- Space available for the cord of less than 8 mm


2- Ossification of the posterior longitudinal ligament
3- Fixed cervical kyphosis
4- Previous posterior surgery
5- Concomitant cervical radiculopathy
4. A 44-year-old man reports persistent left leg pain following a L5-S1 hemilaminotomy
and partial diskectomy. Examination shows a grade 4 weakness of the left extensor
hallucis longus and a positive left straight leg raise. A radiograph is shown in Figure 1a,
and sagittal and axial MRI scans are shown in Figures 1b and 1c. Nonsurgical
management consisting of medication, physical therapy, and injections has failed to
provide relief. Surgical management should consist of

1- revision L5-S1 hemilaminotomy.


2- L5-S1 total disk arthroplasty.
3- L5 Gill laminectomy.
4- posterior foraminal decompression and fusion at L5-S1 with instrumentation and
bone graft.
5- stand-alone posterior lumbar interbody fusion.

5. Bisphosphonates are indicated in the treatment of osteoporosis in patients who have a


DEXA T-score of

1- between 0 and 1.
2- between 0 and -1.
3- -3.5 and are already on teriparatide.
4- within one standard deviation from the mean.
5- less than -1.

6. A 45-year-old man reports that he awoke 2 weeks ago with severe pain in his right arm.
Examination reveals weakness in the biceps, brachialis, and wrist extensors. There is
decreased sensation in the thumb and index finger and a diminished brachioradialis
reflex. Assuming this patient has a posterolateral herniated nucleus pulposus, what level
is involved?

1- C2-3
2- C3-4
3- C4-5
4- C5-6
5- C6-7
7. A 42-year-old woman underwent an instrumented posterior spinal fusion at L3-S1 with
transforaminal lumbar interbody fusion. She had an excellent clinical result with
complete resolution of leg pain. Three months later she now reports increasing back pain
and weakness in her legs. Examination reveals weakness in the quadriceps and tibialis
anterior. Radiographs show no interval changes in the position of the hardware. MRI
scans are shown in Figures 2a through 2c. What is the next most appropriate step in
management?

1- Observation
2- Oral antibiotics only
3- IV antibiotics only
4- Irrigation and debridement of the surgical site
5- Irrigation and debridement of the surgical site with hardware removal

8. What is the primary reason for including the ilium in the distal fixation of long
instrumentation constructs in adult scoliosis?

1- Better coronal balance


2- Better pelvic balance
3- Reduced fretting and corrosion
4- Improved curve correction
5- Improved fusion success

9. A 60-year-old man is evaluated in the ICU after a rollover motor vehicle accident 3 days
ago. He has multiple upper and lower extremity trauma and was found unresponsive at
the accident scene. Surgery is planned for the extremity trauma once the patient is
medically stable. He remains intubated and the cervical spine is immobilized in a semi-
rigid collar. Examination reveals mild erythema in the posterior occipital cervical region.
Initial AP and lateral radiographs of the cervical spine have not revealed any obvious
fracture. What is the most appropriate treatment option at this time?

1- Continued semi-rigid immobilization until the extremity surgeries are completed


2- Halo skeletal fixation prior to the extremity surgery
3- Definitive clearance of the cervical spine with CT and/or MRI
4- Removal of the semi-rigid collar and physical examination when the patient is
responsive
5- Soft collar immobilization and local wound care
10. A 46-year-old woman who was involved in a motor vehicle
accident reports a 4-month history of right-sided lower back pain
and pain radiating into the right thigh. The patient underwent an
extensive 3-month course of physical therapy and now is
dependent on narcotic medication for pain control. Epidural
injection therapy has failed to improve her symptoms.
Examination is significant for weakness of hip flexion in the
seated position and for decreased sensation to light touch in the
medial anterior thigh region. Straight leg raise is negative, but the
femoral stretch test reproduces anterior thigh pain. A CT
myelogram image, at L3-L4, is shown in Figure 3. What is the
most appropriate management at this time?

1- Repeat epidural steroid injections


2- Wide lumbar laminectomy
3- Microdiskectomy from either a midline approach or far lateral approach
4- Referral to pain management
5- Minimally invasive posterior lumbar interbody fusion

11. A 73-year-old woman reports a 4-month history of severe


left-sided posterior buttock pain and left leg pain. The leg
pain radiates into the left lateral thigh and posterior calf with
cramping. Examination reveals mild difficulty with a
single-leg toe raise on the left side and a diminished ankle
reflex. There is also a significant straight leg raise test at 45
degrees which exacerbates symptoms. An MRI scan is
shown in Figure 4. What is the most appropriate treatment
at this time?

1- Lumbar laminectomy with synovial cyst excision


2- Repeat epidural steroid injection
3- Microdiskectomy at L4-5
4- Nonsteroidal medication and outpatient physical therapy
5- Left-sided facet blocks at L4-5 and L5-S1

12. Osteoporotic vertebral compression fractures are associated with

1- neurologic deterioration in 33% of patients.


2- osteomalacia in 50% of patients.
3- a further fracture risk rate of 20%.
4- chronic pain in 75% of patients.
5- a 2-year mortality rate that is less than that associated with hip fractures.
13. When compared to smokers who do not quit, an improvement in the rate of lumbar fusion
is seen in patients who cease smoking for at least how many months postoperatively?

1- 1 month
2- 2 months
3- 4 months
4- 6 months
5- 12 months

14. A 19-year-old woman reports persistent neck pain for 2 years. Pain is
relieved with aspirin. A bone scan shows intense uptake in the
superior, posterior portion of the C3 vertebral body. A sagittal CT
reconstruction is shown in Figure 5. Treatment should consist of

1- radiation therapy.
2- en bloc excision.
3- posterior fusion at C2-C3 with instrumentation.
4- CT-guided aspiration followed by IV antibiotics.
5- radiofrequency ablation.

15. A 56-year-old man with a history of chronic lower back pain from lumbar spondylosis
reports a 2-day history of acute incapacitating back pain. He denies any history of acute
trauma, although he reports the pain starting after a coughing spell. He also reports
difficulty urinating and some fecal incontinence. Examination reveals generalized lower
extremity weakness, saddle paresthesia, hyporeflexia in the lower extremities, and loss of
rectal tone. What is the most appropriate management at this time?

1- Immediate MRI of the lumbar spine


2- General reassurance, anti-inflammatory drugs, and an early home exercise
program
3- Immediate radiographs of the lumbar spine and pain medications with 2 days of
bed rest if the radiographs are normal
4- Office caudal epidural steroid injection with follow-up in 1 week
5- Outpatient MRI of the lumbar spine with follow-up in 1 week for test results
16. A 55-year-old woman with a long history of low back and left lower extremity pain has
failed to respond to exhaustive nonsurgical management. MRI scans show bulging and
degeneration at L3-4 and L4-5 as well as a normal disk at L2-3 and L5-S1. She undergoes
provocative lumbar diskography at L3-4, L4-5, and L5-S1. Post-diskography axial CT images of
L3-4 and L4-5 are shown in Figures 6a and 6b, respectively. The injections at L3-4 and L4-5
produce no pain. The injection at L5-S1 produces 10/10 concordant back pain with radiation to
the lower extremity. What is the most appropriate recommendation at this time?

1- Consider fusion surgery


2- Intradiskal ozone therapy
3- Lumbar laminectomy
4- Vertebral augmentation
5- Cognitive intervention and exercise

17. A 36-year-old woman is brought to the emergency department intubated and sedated
following a motor vehicle accident. She is moving her upper and lower extremities
spontaneously. She cannot follow commands. CT scans are shown in Figures 7a through
7c. The initial survey does not reveal any other injuries. Initial management of the
cervical injury should consist of immediate

1- immobilization with a halo ring and vest with reduction when medically stable.
2- closed traction reduction using Gardner-Wells tongs.
3- posterior open reduction, stabilization, and fusion.
4- cervical MRI followed by reduction.
5- anterior open reduction, stabilization, and fusion.
18. A 51-year-old woman with no preoperative neurologic deficit is undergoing elective
anterior cervical diskectomy and fusion (ACDF) with plating and fusion for a C5-6 disk
herniation with right-sided neck pain. Thirty minutes into the surgery the
neurophysiologic monitoring shows a rapid drop and then loss of amplitude in the right
cortical somatosensory-evoked potential waveform. All other waveforms remained
normal and unchanged, including right-sided cervical (subcortical) and peripheral (Erbs
point), and those from the left-sided upper extremity and both lower extremities. What is
the most likely cause of the change?

1- Electrode placement
2- Stimulation failure
3- Anesthetic effect
4- Cord ischemia from retraction
5- Cerebral ischemia from retraction

19. A 68-year-old woman undergoes a complicated four-level anterior cervical diskectomy


and fusion at C3-7 with iliac crest bone graft and instrumentation for multilevel cervical
stenosis. Surgical time was approximately 6 hours and estimated blood loss was 800 mL.
Neuromonitoring was stable throughout the procedure. The patients history is
significant for smoking. The most immediate appropriate postoperative management for
this patient should include

1- normal postoperative orders with frequent neurologic evaluations for the first 24
hours.
2- administration of IV steroids and placement of a soft cervical collar for 24 hours.
3- placement of both deep and superficial surgical drains prior to wound closure.
4- administration of IV mannitol and placement of a soft collar.
5- maintaining intubation for up to 24 to 48 hours.

20. A 22-year-old woman reports a 4-year history of worsening low back


and left lower extremity pain following a motor vehicle accident.
Management consisting of physical therapy, chiropractic
manipulation, and interventional pain management, including
sacroiliac joint injections and epidural steroid injections, has failed to
provide relief. A sagittal T 2 -weighted MRI scan is shown in Figure 8.
No nerve root compression is seen on axial images. She is currently
working and lives with her fianc. She smokes half a pack of
cigarettes per day and reports depression on her health history. She is
being maintained on narcotic analgesics and is having increasing
difficulty performing her activities of daily living secondary to pain.
What is the most appropriate management at this time?

1- Provocative lumbar diskography


2- Laboratory studies, including a complete blood cell (CBC) count, erythrocyte
sedimentation rate (ESR), and urinalysis
3- Cognitive intervention, exercise, and smoking cessation
4- Bilateral lower extremity electromyography and nerve conduction velocity studies
5- Lumbar myelogram with a postmyelography CT scan of the lumbar spine
21. A 42-year-old man with a history of renal cell carcinoma has progressive weakness in the
lower extremities for the past 3 weeks. The patient desires intervention. A sagittal T 2 -
weighted MRI scan is shown in Figure 9a, and a sagittal contrast enhanced T 1 -weighted
MRI scan is shown in Figure 9b. He currently ambulates minimal distances with a
walker. His life expectancy is 8 months. Treatment of the spine lesion should consist of

1- radiation therapy.
2- posterior laminectomy.
3- anterior corpectomy and reconstruction.
4- posterior laminectomy and fusion.
5- kyphoplasty.

22. A 40-year-old man has intractable pain following 2 years of nonsurgical management for
high-grade spondylolisthesis. What is the best surgical option?

1- Posterolateral fusion
2- Posterolateral fusion with instrumentation
3- Circumferential fusion
4- Transforaminal lumbar interbody fusion
5- Anterior lumbar interbody fusion

23. An adult patient with a grade I isthmic spondylolisthesis at L5-S1 is most likely to have
weakness of the

1- anterior tibialis.
2- quadriceps.
3- gastrocsoleus.
4- extensor hallucis longus.
5- iliopsoas.
24. When performing a long fusion to the sacrum in an osteopenic patient in whom optimal
sagittal balance is restored, which of the following is a benefit of extending the distal
fixation to the pelvis, rather than the sacrum alone?

1- Decreased risk of sacral fractures


2- Decreased risk of proximal functional kyphosis
3- Easier contouring of the instrumentation
4- Reduced risk of late pubic ramus fractures
5- Improved coronal plane correction

25. Which of the following statements describing chordomas is false?

1- Treatment consisting of complete surgical resection with clean margins offers the
best survival.
2- They occur in the clivus and sacrum and occur only 15% of the time in the rest of
the spine.
3- They are locally aggressive and invasive tumors.
4- They are highly radiosensitive.
5- They have the ability to become malignant.

26. A previously healthy 35-year-old man was involved in a rollover motor vehicle accident
2 days ago. He was placed in a semi-rigid cervical orthosis. He now reports mostly axial
neck pain with attempted range of motion. Examination reveals the mechanical neck
pain but no obvious neurologic deficits. AP, flexion, and extension radiographs are
shown in Figures 10a through 10c, and sagittal and coronal CT scans are shown in
Figures 10d and 10e. What is the most appropriate management at this time?

1- Continued immobilization in a semi-rigid cervical orthosis for 6 to 8 weeks


2- Posterior occipital-cervical fusion with iliac crest bone graft
3- Open reduction and internal fixation of the odontoid process with an anterior
odontoid screw
4- Resection of the odontoid process through a transoral approach
5- Reduction with Gardner-Wells tong traction and 6 weeks of skeletal traction
27. Which of the following palpable bony landmarks is correctly matched with its
corresponding vertebral level?

1- Angle of the mandible and the C2-C3 interspace


2- Hyoid bone and C6
3- Carotid tubercle and C6
4- Superior portion of the thyroid cartilage and the C3 vertebral body
5- Cricoid cartilage and C7-T 1

28. What root is most commonly involved with a segmental root level palsy after
laminoplasty?

1- C3
2- C4
3- C5
4- C6
5- C7

29. Up to what time frame are the risks minimized in anterior revision disk replacement
surgery?

1- 3 days
2- 1 week
3- 10 days
4- 2 weeks
5- 6 weeks

30. Which of the following best describes the use of epidural morphine and steroid paste after
laminectomy?

1- Associated with an 11% rate of postoperative surgical site complications


2- Associated with a less than 1% rate of surgical site infections
3- Associated with a decreased rate of postoperative urinary retention
4- Considered the standard for outpatient microdiskectomy
5- Should only be used in the absence of radiculopathy
31. Figures 11a and 11b show the T 2 -weighted MRI scans of the lumbar spine of a 53-year-
old woman who has low back and right lower extremity pain. What structure is the arrow
pointing to in Figure 11a?

1- Ligamentum flavum
2- Lumbar synovial cyst
3- Tarlov cyst
4- Pseudomeningocele
5- Herniated nucleus pulposus

32. A 38-year-old man reports a 2-week history of acute lower back pain with radiation into
the left lower extremity. There is no history of trauma and no systemic signs are noted.
Examination reveals a positive straight leg test at 35 degrees on the left side and a
contralateral straight leg raise on the right side. Motor testing demonstrates mild
weakness of the gluteus medius and weakness of the extensor hallucis longus of 3+/5.
Sensory examination demonstrates decreased sensation along the lateral aspect of the calf
and top of the foot. Knee and ankle reflexes are intact and symmetrical. Radiographs
demonstrate no obvious abnormality. MRI scans show a posterolateral disk hernation.
The diagnosis at this time is consistent with a herniated nucleus pulposus at

1- L1-2.
2- L2-3.
3- L3-4.
4- L4-5.
5- L5-S1.
33. A 42-year-old woman is brought to the emergency department following a motor vehicle
accident. She has sustained multiple injuries, and she is intubated and pharmacologically
paralyzed. Sagittal cervical CT scans through the right cervical facets, the left cervical
facets, and the midline are shown in Figures 12a through 12c, respectively. Definitive
management of her cervical injury should consist of

1- anterior diskectomy and fusion at C4-C5.


2- immobilization in a Philadelphia collar and voluntary flexion and extension
radiographs when awake.
3- occipital-cervical fusion with instrumentation.
4- halo immobilization for 12 weeks.
5- left C6 superior facetectomy and posterior fusion at C6-C7 with instrumentation.

34. A 32-year-old motorcycle rider is involved in a motor vehicle accident and radiographs
show a burst fracture at L2 with 20 degrees of kyphosis. The neurologic examination is
consistent with unilateral motor and sensory involvement of the L5, S1, S2, S3, and S4
nerve roots. He has no other injuries. CT demonstrates 20% anterior canal compromise
with displaced laminar fractures at the level of injury. What is the best option for
management of this patient?

1- Bed rest for 6 weeks, followed by mobilization in a thoracolumbosacral orthosis


until the fracture has healed
2- Anterior corpectomy with strut grafting and placement of an anterior plate
spanning L1 to L3
3- Anterior corpectomy with strut grafting, followed by posterior spinal fusion and
instrumentation
4- Posterior spinal fusion and instrumentation from T11 to L4
5- L2 laminectomy and posterior spinal fusion and instrumentation from T11 to L4
35. A patient who underwent a L4-L5 hemilaminotomy and partial diskectomy for
radiculopathy 8 weeks ago now reports increasing low back pain without neurologic
symptoms. A sagittal T 2 -weighted MRI scan is shown in Figure 13a, and a contrast
enhanced T 1 -weighted MRI scan is shown in Figure 13b. What is the most appropriate
management for the patients symptoms?

1- Physical therapy
2- CT-guided needle biopsy and IV antibiotics
3- Revision laminotomy and diskectomy
4- L4-L5 anterior debridement and fusion
5- Open repair of the L4-L5 pseudomeningocele

36. What is the heaviest weight that can be safely applied to the adult cervical spine via
Gardner-Wells tong traction?

1- 40 pounds
2- 50 pounds
3- 75 pounds
4- 100 pounds
5- Greater than 100 pounds
37. A 68-year-old man reports a 4-week history of progressive left-sided lower back and hip
pain. The pain is in the posterior buttock region with radiation to the groin and to the left
anterior knee region. The pain is aggravated with walking and improves with rest. There
is no history of previous trauma. Radiographs are seen in Figures 14a and 14b, and MRI
scans are seen in Figures 14c through 14e. What is the most appropriate treatment option
at this time?

1- Epidural steroid injection at L4-5


2- Outpatient physical therapy for the lower back
3- Non-weight-bearing of the left lower extremity
4- Home exercise program, analgesics, and limited use of muscle relaxants
5- Cortisone injection of the left greater trochanter region

38. A 79-year-old woman reports a history of left leg pain with walking. Her pain is
exacerbated with walking and stair climbing, and her symptoms are improved by
standing after she stops walking. Lumbar flexion does not provide any significant
improvement of the symptoms and sitting does not significantly change symptoms. Her
leg pain is worse at night and she obtains relief by hanging her leg over the side of the
bed. The neurologic examination is essentially normal. Examination of the lower
extremities demonstrates mild early trophic changes, and her pulses distally are palpable
but are diminished bilaterally. Radiographs are shown in Figures 15a and 15b. What is
the next most appropriate step in management?

1- Decompression and posterior fusion at L4-L5


2- Epidural steroid injection at L4-5
3- Nonsteroidal medications and physical therapy for 6 weeks
4- Measurement of the ankle-brachial index
5- CT myelogram
39. Figure 16 shows the MRI scan of a 43-year-old man who has had
worsening low back pain for the past 4 months. What is the most likely
diagnosis?

1- Osteochondroma
2- Posttraumatic kyphosis
3- Staphylococcus aureus osteomyelitis
4- Ankylosing spondylitis
5- Tuberculosis

40. In patients without spondylolisthesis or scoliosis undergoing laminectomy for lumbar


spinal stenosis, spinal fusion is generally recommended if

1- a dural tear is repaired.


2- more than one level requires decompression.
3- less than one half of each facet is removed bilaterally.
4- the pars interarticularis is fractured.
5- the patient is a smoker.

41. An 18-year-old collegiate basketball player has had a 3-month history of activity-related
back pain. She describes isolated low back pain without radiation that increases with
training and playing basketball. Her pain resolves with rest. Physical therapy for 6
weeks has failed to provide relief. An axial CT scan is shown in Figure 17a, and Figures
17b and 17c show sagittal CT reconstructions through the right and left lumbar facets,
respectively. Further management should consist of which of the following?

1- CT-guided needle biopsy followed by radiation therapy


2- L5-S1 fusion with instrumentation
3- L5-S1 hemilaminotomy and partial diskectomy
4- Activity restriction and bracing
5- L5-S1 total disk arthroplasty
42. Radiating pain associated with a posterolateral thoracic disk herniation typically follows
what pattern?

1- Extending down the spine into the lumbosacral region


2- Down the inner aspect of either upper extremity
3- Cephalad up to the cervicothoracic junction
4- Around or through the chest to the anterior wall
5- Down the contralateral lower extremity

43. A 53-year-old man reports a 5-week history of worsening low back pain accompanied by
bilateral knee and ankle pain and swelling. He also reports a lesser degree of neck and
left elbow pain. He denies any history of trauma or provocative episodes. His medical
history is significant for Reiters syndrome more than 25 years ago, with no subsequent
exacerbations. Furthermore, he has recently returned from a vacation in Costa Rica and
noted the development of infectious gastroenteritis with diarrhea within 1 week of his
return. This was treated with a 10-day course of oral antibiotics and has since resolved.
He denies any significant bowel or urinary symptoms at this time. His neurologic
examination is essentially within normal limits, but is somewhat limited by his low back
and leg pain. What further investigation is most appropriate at this time?

1- Radiographs of the lumbar spine and bilateral knees and ankles


2- MRI of the lumbar spine with and without gadolinium contrast
3- Synovial fluid analysis of the involved joints for crystals and bacteria
4- Laboratory tests including a CBC count, erythrocyte sedimentation rate (ESR),
and C-reactive protein (CRP)
5- Laboratory tests including CBC count, rheumatoid factor (RF), antinuclear
antibodies (ANA), and human leukocyte antigen-B27 (HLA-B27)

44. The 5-year outcome for patients with sciatica secondary to lumbar disk herniation shows
which of the following results?

1- Patients have the same likelihood of receiving disability whether treated with or
without surgery.
2- Sixty percent of surgically treated patients undergo at least one more operation
within 5 years.
3- Only 20% of patients treated with surgery report improved symptoms of back
and/or leg pain.
4- A smaller portion of surgical patients, compared to nonsurgically treated patients,
report improvement.
5- Fifty percent of patients treated nonsurgically seek surgery within 5 years.
45. What is one of the principle concerns when a fracture such as the one
seen in Figure is encountered?

1- Fractures of the lower extremities


2- Paroxysmal hypertension
3- Infection
4- Epidural hematoma
5- Gastrointestinal bleeding

46. Retrograde ejaculation is most commonly associated with what surgical approach?

1- Anterior retroperitoneal approach to L5-S1


2- Anterior transperitoneal approach to L5-S1
3- Anterior retroperitoneal approach to L4-5
4- Minimally invasive lateral trans-psoas approach to L4-5
5- Open lateral approach to L4-5

47. What nerve is most likely to be injured during the anterior exposure of C2-3?

1- Facial
2- Superior laryngeal
3- Vagus
4- Hypoglossal
5- Phrenic

48. A 24-year-old man sustains the injury shown in Figures 19a through 19e in a paragliding
accident. He is neurologically intact. He also sustained fractures of his left femur and
right distal radius. Which of the following represents the best option for management of
the spinal injury?

1- Bed rest for 6 weeks, followed by mobilization in a thoracolumbosacral orthosis


(TLSO) until the fracture has healed
2- Immediate mobilization in a TLSO, continuing until the fracture has healed
3- Anterior corpectomy with strut grafting and placement of anterior fixation
4- Anterior corpectomy and strut grafting followed by posterior spinal fusion and
instrumentation
5- Posterior spinal fusion and instrumentation
49. An 82-year-old man is seen in consultation after being admitted for a fall from ground
level. There was no loss of consciousness and the patient recalls striking his head and
sustaining a hyperextension-type injury to the cervical spine. Examination reveals an
8-cm head laceration with only mild axial neck tenderness. He has generalized weakness
throughout the upper extremities and maintained motor function of the lower extremities.
There are no obvious sensory deficits, and the bulbocavernous reflex and deep tendon
reflexes are maintained. What is the most appropriate diagnosis at this time?

1- Anterior cord syndrome


2- Central cord syndrome
3- Posterior cord syndrome
4- Brown-Squard syndrome
5- Spinal shock

50. Kyphosis from a vertebral osteoporotic compression fracture often results in progressive
kyphosis due to

1- progressive increase in lumbar lordosis.


2- load transfer to the superior adjacent vertebra.
3- normalization of load transfer with working kyphosis.
4- reduced strain at the occipito-cervical junction.
5- reduced strain at the apex of the deformity.

51. A 58-year-old woman with rheumatoid arthritis has progressive neck pain, upper
extremity and lower extremity weakness, and difficulty with fine motor movements.
Examination reveals hyperreflexia with mild to moderate objective weakness but the
patient has no difficulty with ambulation for short distances. What is the most important
preoperative imaging finding that predicts full neurologic recovery with surgical
stabilization?

1- Basilar invagination of less than 1 cm


2- Anterior atlanto-dens interval of 4 mm
3- Posterior atlanto-dens interval of greater than 14 mm
4- Rotatory subluxation of less than 10 degrees
5- Subaxial subluxation of less than 3.5 mm
52. Figures 20a through 20d show the radiographs and MRI scans of a 59-year-old woman
who has had symptoms consistent with progressive neurogenic claudication and back pain for
the past 9 months. In the last 6 months, nonsurgical management consisting of nonsteroidal anti-
inflammatory drugs, physical therapy, and a series of epidural steroid injections have been used;
however the injections, while beneficial, have provided only temporary relief of her symptoms.
What is the most appropriate management at this time?

1- Repeat trial of epidural steroid injections


2- Pain management referral for narcotic management of symptoms
3- Lumbar laminectomies at L4-5
4- Lumbar laminectomies and fusion at L4-5
5- Bilateral lumbar laminotomies at L3-4 and L4-5

53. A 29-year-old man reports a 2-week history of severe neck pain after being struck sharply
on the back of the head and neck while moving a refrigerator down a flight of stairs.
Initial evaluation in the emergency department revealed no obvious fracture and he was
discharged in a soft collar. Neurologic examination is within normal limits, and
radiographs taken in the office are shown in Figures 21a through 21c. Subsequent MRI
scans show intra-substance rupture of the transverse atlantal ligament. What is the most
appropriate treatment option at this time?

1- Discontinue use of the soft collar and encourage range of motion


2- Semi-rigid collar immobilization for 6 to 8 weeks
3- Surgical stabilization
4- Halo skeletal fixation
5- Outpatient physical therapy with isometric neck exercises
54. Figure 22 reveals what anatomic variant of the lumbar spine?

1- Spina bifida occulta


2- Unilateral sacralization
3- An aplastic or hypoplastic pedicle
4- Lumbarization
5- Facet tropism

55. Posterior lumbar spine arthrodesis may be associated with adjacent segment degeneration
cephalad or caudad to the fusion segment. Which of the following is the predicted rate of
symptomatic degeneration at an adjacent segment warranting either decompression
and/or arthrodesis at mid-range follow-up (5-10 years) after lumbar fusion?

1- 2%
2- 10%
3- 25%
4- 50%
5- 80%

56. A 24-year-old man who was involved in a high speed motor vehicle accident is
transferred for definitive care after having been diagnosed with an acute spinal cord
injury from a fracture-dislocation at C6-7. He has a complete C6 neurologic level and it
is now approximately 10 hours from his injury. What is the most appropriate
pharmacologic treatment at this time?

1- No pharmacologic intervention is recommended at this time


2- Administration of methylprednisolone with an initial bolus of 30 mg/kg followed
by 5.4 mg/kg for 24 hours
3- Administration of methylprednisolone with an initial bolus of 30 mg/kg followed
by 5.4 mg/kg for 48 hours
4- Administration of naloxone with an initial bolus of 30 mg/kg followed by
5.4 mg/kg for 24 hours
5- Administration of naloxone with an initial bolus of 30 mg/kg followed by
5.4 mg/kg for 48 hours
57. Figures 23a and 23b show the MRI scans of a 50-year-old woman who has increasing
gait disturbance. She reports three falls in the past week. Examination reveals
hyperreflexia, motor weakness in the biceps and triceps, and a positive Hoffmans sign.
What is the most appropriate treatment plan?

1- Observation
2- Physical therapy
3- Epidural steroid injections
4- Cervical laminectomy
5- Anterior cervical diskectomy and fusion

58. What structure (arrow) is shown in Figure 24?

1- Ilioinguinal nerve
2- Sympathetic chain
3- Ureter
4- Iliac vein
5- L5 nerve

59. The best patient-related outcomes, following the surgical treatment of cauda equina
syndrome secondary to a large L5-S1 disk herniation, are most closely related to which of
the following?

1- Extent of bowel and bladder dysfunction


2- Extent of the motor deficit
3- Extent of the perianal saddle anesthesia
4- Timing of surgery
5- Location of the herniation
60. A 45-year-old man undergoes an anterior cervical diskectomy and
fusion at C5-6 and C6-7 with instrumentation. During the first
postoperative visit at 1 week, the patient reports difficulty
swallowing and mild anterior cervical tightness. The anterior
wound is benign and the patient denies any dyspnea or shortness
of breath. A postoperative radiograph is seen in Figure 25. What
is the most appropriate management at this time?

1- Admit for observation and reassurance


2- Surgical exploration and removal of the anterior
instrumentation
3- Esophageal swallowing study
4- Soft cervical collar and early range-of-motion exercises
5- CT of the cervical spine

61. Steroids are thought to prevent neurologic deterioration after traumatic spinal cord injury
by which of the following mechanisms?

1- Maintains calcium influx into damaged cells


2- Destabilizes lysosomal membranes in the zone of injury
3- Reduces TNF-alpha expression
4- Increases NF-kB binding capacity
5- Maintains free radical oxidation

62. Which of the following mechanisms of inhibition has been linked to cigarette smoking
and lumbar spinal fusion?

1- Diminished revascularization of cancellous bone graft


2- Increased activity of osteoblasts
3- Increased activity of osteocytes
4- Antibody-induced necrosis
5- Inhibition of prostaglandins

63. Which of the following is considered the most effective means of identifying an evolving
motor tract injury during cervical spine surgery?

1- Mean arterial blood pressure monitoring


2- SSEP monitoring
3- Free-run electromyography
4- Transcranial motor monitoring (tceMEP)
5- Wake-up test
64. A previously healthy 29-year-old man reports a 2-day history of severe atraumatic lower
back pain. He denies any bowel or bladder difficulties and no constitutional signs.
Examination is consistent with mechanical back pain. No focal neurologic deficits or
pathologic reflexes are noted. What is the most appropriate management?

1- Radiographs, including anterior, lateral, and oblique views


2- MRI of the lumbar spine and follow-up at the clinic in 1 week
3- Caudal epidural steroid injection
4- Reassurance, limited analgesics, and early range of motion as tolerated
5- Immediate MRI of the lumbar spine and possible urgent surgical decompression

65. Sacral fractures are most likely to be associated with neurologic deficits when they
involve what portion of the sacrum?

1- Zone 1 (the ala)


2- Zone 2 (the foramina)
3- Zone 3 (the central canal)
4- Zones 1 and 2
5- The sacral laminae

66. Which of the following is associated with the use of bisphosphonates in the setting of
metastatic breast cancer to the spine?

1- Reduction in skeletal-related events by 30% to 40%


2- Jaw osteonecrosis in 15% of patients
3- Pain improvement in only 30% of patients
4- Improvement in serum hypocalcemia in 40% of patients
5- Accelerated bone destruction in 10% of patients

67. A 67-year-old retired steelworker was involved in a motor vehicle accident and sustained
a midcervical spinal cord injury. Radiographs and MRI scans reveal severe cervical
stenosis and spondylosis without fractures or dislocations. Neurologic examination
reveals an ASIA C spinal cord impairment with greater motor involvement of the upper
extremities than the lower extremities. What is the probability that the patient eventually
will become ambulatory?

1- 2% to 5%
2- 15% to 20%
3- 35% to 45%
4- 60% to 70%
5- Greater than 90%
68. A 20-year-old man involved in a motor vehicle accident is brought to the emergency
department with a C6-7 unilateral facet dislocation. His neurologic examination reveals
a focal left-sided C7 nerve root palsy. He is awake and cooperative with questioning
and has no other obvious traumatic injuries. What is the most appropriate treatment at
this time?

1- Further imaging studies, including MRI


2- An awake closed reduction with Gardner-Wells traction with neurologic
examination
3- Immobilization in a halo skeletal fixation for definitive treatment
4- Closed reduction under general anesthesia
5- Immediate open reduction and internal fixation in the surgical suite

69. A 66-year-old man reports a 2-week history of worsening low back and leg pain. He
reports that his pain is aggravated by lying down and relieved by standing and walking.
He notes that he has been losing weight recently and that his pain has been awakening
him during the night. His medical history is significant for hypertension, coronary artery
disease, and prostate cancer. His physical examination is essentially unremarkable.
Lumbar radiographs are within normal limits. What is the most appropriate management
for this patient?

1- MRI of chest
2- Laboratory studies, including a complete blood cell (CBC) count, erythrocyte
sedimentation rate (ESR), and urinalysis, PSA, CEA
3- Activity alterations to avoid undue back irritation
4- Comfort measures, including medications
5- Spinal manipulative therapy within the first 6 weeks

70. Which of the following increases radiation exposure to patients and personnel during
surgery?

1- Orienting the beam in the opposite direction of the working team and keeping the
team outside a 6-foot radius of the fluoroscopy machine
2- Orienting the cathode ray tube beneath the patient with the image intensifier
receptor as close to the patient as possible
3- Limiting the beam on time to only what is clinically important
4- The use of continuous fluoroscopy whenever possible to ensure proper placement
of implants
5- The use of lead glasses, a thyroid shield, and a lead apron with an equivalent lead
thickness of 0.25 mm
71. A 78-year-old woman undergoes her third lumbar decompression and fusion from L3 to
L5 without complication. On the morning of postoperative day 3, examination reveals painless,
flaccid weakness of both lower extremities. She also has an absent bulbocavernous reflex and a
mild saddle paresthesia. MRI scans of the lumbar spine are shown in Figures 26a and 26b.
What is the most appropriate management at this time?

1- Continued serial neurologic examinations


2- CT with a myelogram of the lumbar spine
3- Immediate surgical exploration and hematoma drainage
4- Electromyography of bilateral lower extremities
5- IV antibiotics for 24 hours, followed by surgical exploration if symptoms persist

72. Figures 27a through 27c show the radiographs and CT scan of a 27-year-old man who
sustained a low-velocity gunshot wound to the neck. He is quadriplegic (ASIA A),
hemodynamically stable, and does not have drainage from his wound. After initial
resuscitation and stabilization, the cervical spine and spinal cord injuries are best
managed by

1- wound debridement, anterior corpectomy, spinal cord decompression, dural


repair, and anterior fusion with strut graft and anterior plating.
2- wound debridement, anterior corpectomy, spinal cord decompression, dural
repair, anterior fusion with strut graft and anterior plating followed by posterior
laminectomy, and spinal cord decompression and dural repair with excision of the
bullet fragment.
3- wound debridement, anterior corpectomy, spinal cord decompression, dural
repair, anterior fusion with strut graft and anterior plating followed by
laminectomy and posterior fusion, and spinal cord decompression and dural repair
with excision of the bullet fragment.
4- laminectomy and posterior fusion, and spinal cord decompression and dural repair
with excision of the bullet fragment.
5- surgical treatment based on extraspinal pathology with orthotic treatment of the
spinal fractures.
73. Which of the following is a true statement regarding thoracic disk herniations?

1- Are most commonly discovered during the fifth to seventh decades of life
2- Occur with similar frequency as cervical disk herniations
3- Occur most commonly in the midthoracic or apical region of the spine
4- Can be found in 40% of asymptomatic individuals
5- Are best treated surgically with posterior laminectomy and excision

74. A sentinel event is defined as an unexpected occurrence involving death or serious


physical or psychological injury, or the risk thereof. What is the most common sentinel
event related to spine surgery?

1- Surgery on the wrong patient


2- Surgery on the wrong side
3- Incorrect procedure performed
4- Intraoperative death
5- Surgery on the wrong level

75. What structure is most at risk with anterior penetration of C1 lateral mass screws?

1- Vertebral artery
2- External carotid artery
3- Internal carotid artery
4- Pharynx
5- Glossopharyngeal nerve

76. During the application of halo skeletal fixation, the most appropriate position for the
placement of the anterior halo pins is approximately 1 cm above the superior orbital
rim and

1- lateral placement, directly within the temporalis muscle.


2- lateral to the medial third of the orbit.
3- lateral to the lateral aspect of the orbit.
4- above the medial third of the orbit.
5- lateral between the temporalis muscle and zygomatic temporal nerve.
77. Figures 28a and 28b show the sagittal and axial lumbar MRI scans of a 72-year-old man
who reports dull aching back pain that spreads to his legs, calves, and buttocks. He has had the
pain for several years and it is precipitated by standing and walking and relieved by sitting. His
symptoms have been worsening over the past year and he notes that he is leaning forward while
walking to help relieve his symptoms. He has had no treatment to date. What is his prognosis if
he chooses to pursue nonsurgical management for this condition?

1- He can expect complete resolution of his symptoms during the first month.
2- All patients experience relief within 3 months and continue to improve over the
next 4 years.
3- Most patients experience some pain relief within the first 3 months.
4- He may experience some improvement but if he requires surgery at a later date he
will have a poorer result because of the delay.
5- The patient requires immediate surgery to avoid permanent nerve damage.

78. Which of the following vertebrae has the smallest pedicle isthmic width in a
nondeformity patient?

1- T10
2- T11
3- T12
4- L1
5- L2

79. Which of the following represents a contraindication for interspinous process


decompression for the treatment of lumbar spinal stenosis?

1- Grade I degenerative spondylolisthesis


2- Inability to walk at least 100 feet
3- Cauda equina syndrome
4- Fixed sensory deficit
5- Intermittent foot drop
80. Which of the following statements about hoarseness due to vocal cord paralysis after
anterior cervical diskectomy and fusion is most accurate?

1- Vocal cord paralysis is three times as likely with a right-sided approach as


compared to a left-sided approach.
2- Vocal cord paralysis is twice as likely with a right-sided approach as compared to
a left-sided approach.
3- Vocal cord paralysis is equally likely with either a right-sided or a left-sided
approach.
4- Vocal cord paralysis is three times as likely with a left-sided approach as
compared to a right-sided approach.
5- Vocal cord paralysis is twice as likely with a left-sided approach as compared to a
right-sided approach.

81. A 23-year-old man is involved in a motor vehicle accident. An AP radiograph is shown


in Figure 29a, and axial and sagittal CT scans are shown in Figures 29b and 29c.
Neurologic examination shows 1/5 strength of his quadriceps and iliopsoas on the right,
with 1/5 quadriceps function on the left. Definitive treatment of his injury should
consist of

1- anterior corpectomy with interbody strut.


2- posterior fusion with instrumentation and posterolateral decompression.
3- closed reduction and a thoracolumbosacral orthosis (TLSO).
4- anterior reduction and instrumentation.
5- supine bed rest for 6 weeks, followed by immobilization in a TLSO.

82. Surgical treatment for symptomatic disk herniations is associated with which of the
following?

1- Substantial rate of nerve root injury


2- Early relief of pain sustained out to 2 years
3- Recurrent herniation rate of 35%
4- Outcomes that are substantially worse than nonsurgical management
5- 10% rate of infectious diskitis
83. A 25-year-old man is unresponsive at the scene of a high-speed motor vehicle accident
and remains obtunded. Initial evaluation in the emergency department reveals a left-
sided femoral shaft fracture and a right-sided humeral shaft fracture. The cervical spine
remains immobilized in a semi-rigid cervical collar, and the initial AP and lateral
radiographs obtained in the emergency department are unremarkable. What is the most
appropriate management at this time?

1- Lateral radiographs with passive flexion/extension views


2- Helical CT scan of the cervical-thoracic region
3- Careful manual palpation of the cervical spine for subtle defects or step-offs
4- MRI of the cervical spine
5- Continued use of the cervical collar until the patient becomes responsive for
examination

84. A 55-year-old woman undergoes an anterior cervical diskectomy and fusion at C5-C6
through a left-sided approach. One year later, she requires an anterior cervical
diskectomy and fusion on another level. Which of the following is considered a
contraindication to performing a right-sided approach for the revision procedure?

1- Revision surgery caudad to C6


2- Persistent left cervical radiculopathy
3- History of a left-sided Horners syndrome
4- Transient dysphagia following the initial anterior cervical procedure
5- Nonfunctional left vocal cord

85. A 56-year-old woman sustained the fracture shown in Figures 30a and 30b in a motor
vehicle accident. What mechanism is most likely responsible for the injury?

1- Flexion distraction
2- Vertical shear
3- Extension distraction
4- Flexion compression
5- Axial load
86. In providing culturally competent care to a Muslim woman with a cervical spine injury,
which of the following most accurately describes the steps a male orthopaedist should
take to respect her religious beliefs during his examination?

1- No one should be in the exam room except the patient and the physician.
2- Another woman should be in the exam room and only the affected body part
should be exposed.
3- A chaperone of either gender should be in the exam room and no skin should be
exposed.
4- No particular steps need to be taken in this case.
5- The patients closest male relative should be in the exam room but a standard
hospital gown may be used.

87. Figure 31 shows the radiograph of a 64-year-old woman who is


seen in the emergency department following a motor vehicle
accident. She has no voluntary motor function in her distal upper
extremities or lower extremities. She does not have a
bulbocavernosus reflex. She has a blood pressure of 80/50 mm Hg
with a pulse of 50/min. Her hypotension does not improve with
initial fluid resuscitation. Further treatment of her hypotension
should consist of

1- continued rapid fluid infusion.


2- administration of broad-spectrum antibiotics.
3- administration of 30/mg/kg methylprednisolone over 1 hour.
4- administration of pressors.
5- cardioversion and implantation of a pacemaker.

88. What is the typical axial plane transverse angulation of the thoracic pedicles?

1- 5 degrees medial at T 1 and T 2 ; 10 degrees from T3 to T10


2- 5 degrees lateral at T 1 ; neutral at T 2 ; 5 degrees medial from T3 to T12
3- 10 degrees medial from T1 to T10; 15 degrees medial at T11 and T12
4- 10 degrees medial from T 1 to T12
5- 25 degrees medial at T 1 ; 15 degrees at T 2 ; and 10 degrees medial from T3 to T10

89. What muscle is most often encountered during surgical approaches to C5-6?

1- Omohyoid
2- Cricohyoid
3- Splenius capitus
4- Thyrohyoid
5- Posterior digastric
90. Which of the following lumbar disk components has the highest tensile modulus to resist
torsional, axial, and tensile loads?

1- Nucleus pulposus
2- Cartilaginous end plate
3- Anterior longitudinal ligament
4- Annulus fibrosis
5- Cellular matrix

91. When comparing the overall outcomes of surgical versus nonsurgical treatment of stable
thoracolumbar burst fractures in patients without neurologic injury, 5 years following
injury, the principle differences lie in

1- fracture kyphosis.
2- reduction of retropulsed bone.
3- pain reduction.
4- incidence of complications.
5- return to work.

92. A 42-year-old woman who has had an 18-month history of severe low back pain is
referred to your office for surgical evaluation. She reports that the pain initially began
with right lower extremity pain and management consisted of oral analgesics,
nonsteroidal anti-inflammatory drugs, and muscle relaxants. She has seen a chiropractor
as well as a pain management specialist and she is status-post epidural steroid injections.
She has also completed exhaustive physical therapy, as she is a certified athletic trainer
and runs a health fitness program at a community hospital. Currently, she denies lower
extremity pain and her pain is isolated to her low back and is subjectively graded as 8/10,
with 10 being the worst pain she has ever experienced. The pain is interfering with her
activities of daily living and she is seeking definitive treatment. Figures 32a through 32c
show current MRI scans. Based on the current available medical literature, what is the
most appropriate treatment?

1- Continued nonsurgical management to include long-acting narcotic analgesics


2- Referral for vertebral axial decompression
3- Referral to interventional pain management for a spinal cord stimulator
4- Intradiskal electrothermal therapy (IDET) at L5-S1
5- Lumbar spinal fusion at L5-S1
93. Figure 33 shows the MRI scan of a 55-year-old woman who has had a 6-week history of
back and leg pain. Which of the following clinical scenarios is most consistent with the
MRI scan findings at L4-L5?

1- L4 nerve root radiculopathy


2- L5 nerve root radiculopathy
3- Associated bowel and bladder dysfunction
4- Symptoms associated with arachnoiditis
5- Wide-based gait, left-sided Hoffmans sign

94. Intradiskal electrothermal therapy (IDET) uses an intradiskal catheter to deliver


controlled thermal energy to the inner periphery of the annulus fibrosis of a chronically
painful intervertebral disk. Lumbar diskography is used diagnostically to identify the
presumed pain generator to be targeted with IDET. Based on the medical literature, what
can be said about the current status of IDET?

1- IDET has been proven to seal annular tears in the annulus fibrosis.
2- IDET restores segmental stability by shrinking collagen fibrils in the disk.
3- IDET has demonstrated no significant benefit over placebo in controlled trials.
4- IDET is an unsafe procedure with significant risk of permanent complications.
5- IDET has demonstrated poor clinical results in all reported series to date.

95. A 56-year-old mechanic has had pain in the hypothenar region of his dominant right hand
for the past 6 months. He reports weakness in his grip and pain is worse with activity.
Which of the following examination findings is most suggestive of a cervical etiology?

1- Relief of symptoms with shoulder abduction (placing hand over the head)
2- Hypothenar atrophy
3- Reproduction of pain with hyperflexion and contralateral rotation of the head
4- Positive Tinels sign at the levator scapulae
5- Subluxable ulnar nerve at the cubital tunnel
96. A 35-year-old woman reports an 8-week history of neck pain radiating to her right upper
extremity. She denies any history of trauma or provocative event. Examination reveals
decreased pinprick sensation in her right middle finger, otherwise sensation is intact bilaterally.
Finger flexors and interossei demonstrate 5/5 motor strength bilaterally. Finger extensors are 4/5
on the right and 5/5 on the left. The triceps reflex is 1+ on the right and 2+ on the left. The most
likely diagnosis is a herniated nucleus pulposus at what level?

1- C3-4
2- C4-5
3- C5-6
4- C6-7
5- C7-T 1

97. What is the most common nonanesthetic-related reversible cause of changes in


intraoperative neurophysiologic monitoring data?

1- Pedicle screw misplacement


2- Patient positioning
3- Spinal cord ischemia
4- Retractor placement
5- Hypotension

98. During a left-sided transforaminal lumbar interbody fusion at the L4-5 level, the surgeon
notes a significant amount of bleeding that cannot be controlled while using a pituitary
rongeur. What anatomic structure has been injured?

1- Aorta
2- Common iliac artery
3- Common iliac vein
4- External iliac artery
5- External iliac vein

99. Six weeks after onset, what is the most clearly accepted indication for surgical
management for lumbar disk herniation?

1- Stable sensory loss


2- Stable motor weakness
3- Refractory radicular pain
4- Size of the herniation
5- Lost time at work
100. A 45-year-old woman has idiopathic scoliosis. Surgery is to include an anterior thoracic
release through an open left thoracotomy. The thoracotomy will have what effect on the
patients pulmonary function postoperatively?

1- Unaffected
2- Transiently reduced postoperatively but ultimately improves to greater than
preoperative function
3- Transiently reduced immediately postoperatively but then quickly returns to
preoperative levels
4- Improves postoperatively due to correction of the scoliosis and is maintained long
term
5- Reduced postoperatively and often remains reduced long term
Answers
1.PREFERRED RESPONSE: 4

DISCUSSION: Radiography is the best initial study for the evaluation of diskogenic low back
pain. The normal degenerative process can be evaluated. Vacuum phenomenon may be found
within the disk space. Other possible sources for back pain should also be evaluated. The other
tests may be beneficial but represent later imaging options.

REFERENCE: Spivak JM, Connolly PJ (eds): Orthopaedic Knowledge Update: Spine 3.


Rosemont, IL, American Academy of Orthopaedic Surgeons, 2006, pp 319-329.

2.PREFERRED RESPONSE: 3

DISCUSSION: Jehovahs Witnesses will not accept the transfusion of blood or blood products
such as packed red or white cells, platelets, or plasma. However, many Jehovahs Witnesses will
accept the use of a cell saver in a closed circuit.

REFERENCES: Jimenez R, Lewis VO (eds): Culturally Competent Care Guidebook. Rosemont,


IL, American Academy of Orthopaedic Surgeons, 2007.
www.watchtower.org. Official Web Site of Jehovahs Witnesses. Link verified as active as of
August 8, 2008.

3.PREFERRED RESPONSE: 3

DISCUSSION: Laminoplasty or any posterior decompressive procedure is contraindicated in


patients with cervical spondylotic myelopathy and cervical kyphosis. The residual kyphotic
posture of the cervical spine results in persistent spinal cord compression. The other choices are
not contraindications for laminoplasty. Concomitant cervical radiculopathy can be addressed at
the time of laminoplasty with a keyhole foraminotomy.

REFERENCES: Emery SE: Cervical spondylotic myelopathy: Diagnosis and treatment. J Am


Acad Orthop Surg 2001;9:376-388.
Spivak JM, Connolly PJ (eds): Orthopaedic Knowledge Update: Spine 3. Rosemont, IL,
American Academy of Orthopaedic Surgeons, 2006, pp 235-247.

4.PREFERRED RESPONSE: 4

DISCUSSION: The patient has a grade I isthmic spondylolisthesis at L5-S1. He has an L5


radiculopathy with foraminal stenosis. Any further treatment needs to include an arthrodesis and
foraminal decompression. Isolated interbody fusion is contraindicated in patients with
spondylolisthesis, as is total disk arthroplasty. Therefore, the best procedure is a posterior fusion
with instrumentation and bone graft along with a foraminal decompression.

REFERENCES: Spivak JM, Connolly PJ (eds): Orthopaedic Knowledge Update: Spine 3.


Rosemont, IL, American Academy of Orthopaedic Surgeons, 2006, pp 311-317.
Moller H, Hedlund R: Instrumented and noninstrumented posterolateral fusion in adult
spondylolisthesis: A prospective randomized study: Part 2. Spine 2000;25:1716-1721.

5. PREFERRED RESPONSE: 5

DISCUSSION: Bisphosphonates are indicated in the treatment of osteoporosis. They have been
shown to reduce the incidence of vertebral and extremity fractures in patients with a T-score of
less than -1.

REFERENCE: Gass M, Dawson-Hughs B: Preventing osteoporosis-related fractures: An


overview. Am J Med 2006;119:S3-S11.

6. PREFERRED RESPONSE: 4

DISCUSSION: This is a classic C6 nerve injury, and it is most likely the result of a herniated
nucleus pulposus at C5-6. The C5 nerve root controls the elbow flexors, shoulder abductors, and
external rotators. The C7 nerve root controls the elbow extensors, wrist pronators, and the
triceps reflex.

REFERENCES: Standaert CJ: The patient history and physical examination: Cervical, thoracic
and lumbar, in Herkowitz HN, Garfin SR, Eismont FJ, et al (eds): Rothman-Simeone The Spine,
ed 5. Philadelphia, PA, Saunders Elsevier, 2006, vol 1, pp 171-186.
Bates B: A Guide to Physical Examination and History Taking, ed 5. Philadelphia, PA,
JB Lippincott, 1991.

7.PREFERRED RESPONSE: 4

DISCUSSION: The MRI scans reveal a postoperative infection. Observation and antibiotics are
not appropriate choices. There is a large fluid collection and this requires decompression
because the patient has neurologic changes. There is considerable debate regarding the removal
of hardware. Many contend that biofilm on the implants can harbor the infection. However,
these complications usually can be treated with serial irrigations, debridements, and IV
antibiotics. The incidence of infection has been widely studied with varying rates in fusions with
instrumentation. Rates appear to be increased with instrumentation, yet these infections usually
can be managed without hardware removal.

REFERENCES: Glassman SD, Dimar JR, Puno RM, et al: Salvage of instrumental lumbar
fusions complicated by surgical wound infection. Spine 1996;21:2163-2169.
Fang A, Hu SS, Endres N, et al: Risk factors for infection after spinal surgery. Spine
2005;30:1460-1465.

8. PREFERRED RESPONSE: 5

DISCUSSION: Studies have shown that when compared with fixation to the sacrum alone, the
success rate of fusion across the lumbosacral junction increases when both the sacrum and ilium
are included in the posterolateral construct. Curve correction, coronal balance, and pelvic
balance are all attended to within the thoracolumbar spine and are not directly related to the
pelvic fixation. Fretting and corrosion are a byproduct of metal-to-metal connections.

REFERENCES: Islam NC, Wood KB, Transfeldt EE, et al: Extension of fusions to the pelvis in
idiopathic scoliosis. Spine 2001;26:166-173.
Emami A, Deviren V, Berven S, et al: Outcome and complications of long fusions to the sacrum
in adult spine deformity: Luque-Galveston, combined iliac and sacral screws, and sacral fixation.
Spine 2002;27:776-786.

9. PREFERRED RESPONSE: 3

DISCUSSION: Ackland and associates demonstrated that the failure to achieve early spinal
clearance in an unconscious blunt trauma patient predisposed the patient to increased morbidity
secondary to the prolonged used of cervical immobilization. They demonstrated that the four
significant predictors of collar-related ulcers were ICU admission, mechanical ventilation, the
necessity for cervical MRI, and the time to cervical spine clearance and collar removal. The risk
of pressure-related ulceration increased by 66% for every 1-day increase in Philadelphia collar
time and this highlights the need for definitive C-spine clearance.

REFERENCES: Ackland HM, Cooper DJ, Malham GM, et al: Factors predicting cervical collar-
related decubitus ulceration in major trauma patients. Spine 2007;32:423-428.
Hewitt S: Skin necrosis caused by semi-rigid cervical collar in a ventilated patient with multiple
injuries. Injury 1994;25:323-324.

10.PREFERRED RESPONSE: 3

DISCUSSION: The CT scan reveals a right-sided lateral disk protrusion at L3-4 that has been
symptomatic for more than 4 months despite appropriate nonsurgical management. Relative
surgical indications include persistent radiculopathy despite an adequate trial of nonsurgical
management, recurrent episodes of sciatica, persistent motor deficit with tension signs and pain,
and pseudoclaudication caused by underlying stenosis. Whereas studies have shown
improvement in patients with sciatica from a lumbar disk herniation treated either nonsurgically
or surgically, those undergoing surgical treatment had an overall greater improvement of
symptoms.

REFERENCES: Weinstein JN, Lurie JD, Tosteson TD, et al: Surgical vs nonoperative treatment
for lumbar disk herniation: The Spine Patient Outcomes Research Trial (SPORT) observational
cohort. JAMA 2006;296:2451-2459.
Yorimitsu E, Chiba K, Toyama Y, et al: Long-term outcomes of standard discectomy for lumbar
disc herniation: A follow-up study of more than 10 years. Spine 2001;26:652-657.

11. PREFERRED RESPONSE: 1

DISCUSSION: Lumbar spinal stenosis with lumbar radiculopathy can be commonly caused by a
synovial cyst arising from the facet joints. Lyons and associates reported on the surgical
treatment of synovial cysts in 194 patients. Of the 147 with follow-up data, 91% reported good
pain relief and 82% had improvement of their motor deficits. Epstein reported a 58% to 63%
incidence of good/excellent results and a 38 to 42 point improvement on the SF-36 Physical
Function Scale. It was also suggested that since the presence of a synovial cyst indicates facet
pathology, possible fusion should be considered in these patients, especially those with
underlying spondylolisthesis.

REFERENCES: Lyons MK, Atkinson JL, Wharen RE, et al: Surgical evaluation and
management of lumbar synovial cysts: The Mayo Clinic Experience. J Neurosurg
2000;93:53-57.
Khan AM, Synnot K, Cammisa FP, et al: Lumbar synovial cysts of the spine: An evaluation of
surgical outcome. J Spinal Disord Tech 2005;18:127-131.
Epstein NE: Lumbar laminectomy for the resection of synovial cysts and coexisting lumbar
spinal stenosis or degenerative spondylolisthesis: An outcome study. Spine 2004;29:1049-1055.

12. PREFERRED RESPONSE: 3

DISCUSSION: Osteoporotic vertebral compression fractures are associated with neurologic


complications in less than 1% of patients. After the initial fracture however, patients have a 20%
risk of further fractures. The mortality rate of patients with vertebral fractures exceeds that of
patients with hip fractures when they are followed beyond 6 months.

REFERENCES: Gass M, Dawson-Hughs B: Preventing osteoporosis-related fractures: An


overview. Am J Med 2006;119:S3-S11.
Lindsay R, Silverman SL, Cooper C, et al: Risk of new vertebral fracture in the year following a
fracture. JAMA 2001;285:320-323.
Kado DM, Duong T, Stone KL, et al: Incident vertebral fractures and mortality in older women:
A prospective study. Osteoporos Int 2003;14:589-594.

13. PREFERRED RESPONSE: 4

DISCUSSION: The effects of cigarette smoking and smoking cessation on spinal fusion have
been studied extensively. Although permanent smoking cessation is ideal, significant
improvements in fusion rates are seen in patients who avoid smoking for greater than 6 months
postoperatively.

REFERENCE: Glassman SD, Anagnost SC, Parker A, et al: The effect of cigarette smoking and
smoking cessation on spinal fusion. Spine 2000;25:2608-2615.

14. PREFERRED RESPONSE: 2

DISCUSSION: The CT scan shows an osteoblastic nidus pathognomic for an osteoid osteoma.
Surgical treatment should include an en bloc excision of the lesion. Surgical treatment is not
mandatory because the lesion often becomes asymptomatic over time. This lesion is not
amenable to radiofrequency ablation due to its proximity to the spinal cord. A complete
corpectomy is not necessary to adequately resect the lesion, as only the nidus needs to be
removed. Radiation therapy and antibiotics are not appropriate treatments for an osteoid
osteoma. Posterior C2-C3 fusion will not address the pathology.

REFERENCES: Spivak JM, Connolly PJ (eds): Orthopaedic Knowledge Update: Spine 3.


Rosemont, IL, American Academy of Orthopaedic Surgeons, 2006, pp 351-366.
Hadjipavlou AG, Lander PH, Marchesi D, et al: Minimally invasive surgery for ablation of
osteoid osteoma of the spine. Spine 2003;28:E472-E477.

15.PREFERRED RESPONSE: 1

DISCUSSION: Cauda equina syndrome is a medical emergency that must be quickly diagnosed
and treated to avoid long-term complications. Cauda equina syndrome typically presents with
low back pain, unilateral or usually bilateral sciatica, saddle sensory disturbances, bladder and
bowel dysfunction, and variable lower extremity motor and sensory loss. Although a number of
pathologies can cause cauda equina syndrome, in a patient with a history of chronic back pain,
disk pathology is the most common cause of acute onset cauda equina syndrome. Whereas
radiographs may be useful in a traumatic onset of symptoms, MRI is the most appropriate study.
Cauda equina syndrome should be evaluated on an emergent basis and admission for work-up is
appropriate.

REFERENCES: Ahn UM, Ahn NU, Buchowski JM, et al: Cauda equina syndrome secondary to
lumbar disc herniation: A meta-analysis of surgical outcomes. Spine 2000;25:1515-1522.
Small SA, Perron AD, Brady WJ: Orthopedic pitfalls: Cauda equina syndrome. Am J Emerg
Med 2005;23:159-163.

16. PREFERRED RESPONSE: 5

DISCUSSION: The results of this patients lumbar diskography are equivocal at best. The two
disks most likely to be her pain generators, based on their MRI appearance, produced 10/10 pain,
however it was nonconcordant and did not reproduce any of her typical left-sided radicular
symptoms. The only disk that produced concordant back pain was the normal disk at the L5-S1
level and it reproduced radicular symptoms on the side opposite of her typical pain. Based on
these findings, it would be difficult to select a level or levels to include in a lumbar fusion. As
such, continued nonsurgical management is the safest treatment option at the current time. Brox
and associates reported on a randomized clinical trial comparing lumbar fusion to cognitive
intervention and exercise and found similar results in both groups, with significantly less risk in
the latter.

REFERENCES: Brox JI, Sorensen R, Friis A, et al: Randomized clinical trial of lumbar
instrumented fusion and cognitive intervention and exercises in patients with chronic low back
pain and disc degeneration. Spine 2003;28:1913-1921.
Carragee EJ: Clinical practice: Persistent low back pain. N Engl J Med 2005;352:1891-1898.

17. PREFERRED RESPONSE: 4

DISCUSSION: The patient has a bilateral facet dislocation of C6-C7 with preservation of at least
some neurologic function. Urgent reduction is necessary. However, because she is sedated and
unable to follow commands, an MRI scan is necessary before any closed or open posterior
reduction to look for an associated disk herniation. If a disk herniation is present, it must be
removed prior to any reduction maneuver to prevent iatrogenic neurologic injury. It is very
unlikely that this injury can be reduced with an open anterior procedure alone.
REFERENCES: Spivak JM, Connolly PJ (eds): Orthopaedic Knowledge Update: Spine 3.
Rosemont, IL, American Academy of Orthopaedic Surgeons, 2006, pp 189-199.
Kwon BK, Vaccaro AR, Grauer JN, et al: Subaxial cervical spine trauma. J Am Acad Orthop
Surg 2006;14:78-89.

18. PREFERRED RESPONSE: 5

DISCUSSION: The change noted is focal and confined to the cortex, sparing the opposite side,
both lower extremities, and the subcortical waveforms, making all the choices unlikely with the
exception of carotid compression with focal cortical ischemia. This may be associated with poor
collateral flow from the opposite hemisphere due to an incomplete circle of Willis.

REFERENCES: Drummond JC, Englander RN, Gallo CJ: Cerebral ischemia as an apparent
complication of anterior cervical discectomy in a patient with an incomplete circle of Willis.
Anesth Analg 2006;102:896-899.
Yeh YC, Sun WZ, Lin CP, et al: Prolonged retraction on the normal common carotid artery
induced lethal stroke after cervical spine surgery. Spine 2004;29:E431-E434.

19. PREFERRED RESPONSE: 5

DISCUSSION: Airway complications after anterior cervical surgery can be a catastrophic event
necessitating emergent intubation for airway protection. Multilevel surgeries requiring long
intubation and prolonged soft-tissue retraction as well as preexisting comorbidities may
predispose a patient to postoperative airway complications. Sagi and associates reported that
surgical times greater than 5 hours, blood loss greater than 300 mL, and multilevel surgery at or
above C3-4 are risk factors for airway complications. In surgical procedures with the
aforementioned factors, serious consideration should be given to elective intubation for 1 to 3
days to avoid urgent reintubation.

REFERENCES: Sagi HC, Beutler W, Carroll E, et al: Airway complications associated with
surgery on the anterior cervical spine. Spine 2002;27:949-953.
Epstein NE, Hollingsworth R, Nardi D, et al: Can airway complications following multilevel
anterior cervical surgery be avoided? J Neurosurg 2001;94:185-188.
Emery SE, Smith MD, Bohlman HH: Upper-airway obstruction after multi-level cervical
corpectomy for myelopathy. J Bone Joint Surg Am 1991;73:544-551.

20. PREFERRED RESPONSE: 3

DISCUSSION: The MRI scan reveals a rudimentary disk at the L5-S1 level, suggesting
transitional anatomy. There is a posterior disk bulge at L3-4. At L4-5, there is disk desiccation
and loss of disk height, with a posterior disk bulge and a high intensity zone in the posterior
annulus, suggesting an annular tear. While these and similar radiographic findings have been
associated with the severity of a patients pain, they are also commonly found in cross-sectional
studies of asymptomatic subjects. Carragee and associates found 59% of symptomatic patients
undergoing diskography have high intensity zones as compared to 25% of asymptomatic subjects
of a similar patient profile. Diskographic injections provoked pain in disks with high intensity
zones approximately 70% of the time whether the individual was previously symptomatic or not.
This patients non-specific pain pattern does not require further work-up as she is not a surgical
candidate.

REFERENCES: Carragee EJ, Paragioudakis SJ, Khurana S: 2000 Volvo Award winner in
clinical studies: Lumbar high-intensity zone and discography in subjects without low back
problems. Spine 2000;25:2987-2992.
Pneumaticos SG, Reitman CA, Lindsey RW: Diskography in the evaluation of low back pain.
J Am Acad Orthop Surg 2006;14:46-55.
Brox JI, Sorensen R, Friis A, et al: Randomized clinical trial of lumbar instrumented fusion and
cognitive intervention and exercises in patients with chronic low back pain and disc
degeneration. Spine 2003;28:1913-1921.
Carragee EJ: Clinical practice: Persistent low back pain. N Engl J Med 2005;352:1891-1898.

21. PREFERRED RESPONSE: 3

DISCUSSION: The MRI scans show a metastatic lesion in two contiguous vertebral bodies in
the lower thoracic spine. Posterior laminectomy is not indicated because this does not
adequately decompress the neural elements and will lead to progressive kyphosis. A posterior
fusion may prevent progressive kyphosis but will not decompress the spinal cord. Renal cell
carcinoma is not radiosensitive; therefore, radiation therapy would not be helpful in relieving
neurologic compression. The lesion should be treated by an anterior corpectomy and
reconstruction. This will allow for complete decompression as well as reconstruction of the
anterior column. Kyphoplasty is not indicated in a lesion with disruption of the posterior cortex
and neurologic impairment.

REFERENCES: Spivak JM, Connolly PJ (eds): Orthopaedic Knowledge Update: Spine 3.


Rosemont, IL, American Academy of Orthopaedic Surgeons, 2006, pp 351-366.
White AP, Kwon BK, Lindskog DM, et al: Metastatic disease of the spine. J Am Acad Orthop
Surg 2006;14:587-598.

22. PREFERRED RESPONSE: 3

DISCUSSION: Circumferential fusion is the preferred choice for patients undergoing revision
surgery following failed posterolateral fusions for isthmic spondylolisthesis as well as for those
patients having primary surgery for high-grade isthmic spondylolisthesis.

REFERENCE: Spivak JM, Connolly PJ (eds): Orthopaedic Knowledge Update: Spine 3.


Rosemont, IL, American Academy of Orthopaedic Surgeons, 2006, pp 311-317.

23. PREFERRED RESPONSE: 4

DISCUSSION: Adult patients with isthmic spondylolisthesis most commonly have neurologic
symptoms due to foraminal stenosis at the level of the spondylolisthesis. In this scenario, the
patient is most likely to have weakness of the L5 myotome, which would cause weakness of the
extensor hallucis longus.

REFERENCES: Spivak JM, Connolly PJ (eds): Orthopaedic Knowledge Update: Spine 3.


Rosemont, IL, American Academy of Orthopaedic Surgeons, 2006, pp 311-317.
Lauerman WC, Cain JE: Isthmic spondylolisthesis in the adult. J Am Acad Orthop Surg
1996;4:201-208.

24. PREFERRED RESPONSE: 1

DISCUSSION: In osteopenic individuals, even those with excellent obtained or maintained


balance, long instrumented fusions to the sacrum impart a high degree of strain, and the sacrum
may fail in a transverse fracture or fracture-dislocation pattern. The risk of proximal functional
kyphosis is unrelated to distal fixation as are coronal plane correction and rod contouring. Pubic
ramus fractures have been shown to be associated with both fixation to the sacrum alone as well
as to the ilium.

REFERENCE: Hu SS, Berven SH, Bradford DS: Adult spinal deformity, in Frymoyer JW,
Wiesel SW (eds): The Adult and Pediatric Spine, ed 3. Philadelphia, PA, Lippincott Williams
and Wilkins, 2004, pp 465-477.

25. PREFERRED RESPONSE: 4

DISCUSSION: Casali and associates provided a recent review of the treatment options for
chordomas. These tumors are not radiosensitive; however, modern intensity modulated
radiosurgery techniques may be of value. The combination of surgery and radiotherapy
compared to surgery alone results in the same disease-free survival time. Complete surgical
resection of the chondroma with clean margins offers the best survival; however, its location
may make total removal impossible. Thus subtotal resection followed by radiotherapy results in
better survival despite the tumors lack of radiosensitivity.

REFERENCE: Casali PG, Stacchiotti S, Sangalli C, et al: Chordoma. Curr Opin Oncol
2007;19:367-370.

26. PREFERRED RESPONSE: 3

DISCUSSION: Odontoid fractures can be classified based on the anatomic position of the
fracture within the dens itself. Type I is an oblique fracture through the upper part of the
odontoid process. Type II is a fracture that occurs at the base of the odontoid as it attaches to the
body of C2; type III occurs when the fracture line extends through the body of the axis. Type 1
fractures typically can be treated nonsurgically with 6 to 8 weeks of immobilization with a semi-
rigid cervical orthosis. Nondisplaced, deep type III fractures generally are treated with skeletal
halo fixation. Deep, displaced, and angled type III fractures can be treated with closed reduction
and skeletal halo fixation. Shallow type III fractures are sometimes amenable to anterior
odontoid screw fixation. Type II fractures can be managed nonsurgically or surgically.
Treatment options include halo immobilization, internal fixation (odontoid screw fixation), and
posterior atlantoaxial arthrodesis. Management with the halo vest usually is considered if the
initial dens displacement is less than 6 mm, the reduction is performed within 1 week of the
injury and is able to be maintained, and the patient is younger than age 60 years. Halo vest
immobilization can lead to a healing rate of more than 90%. Posterior surgical fusion techniques
provide high fusion success rates but do so at the expense of cervical rotation. Up to 50% of
rotation is lost with these techniques. Anterior odontoid single screw fixation is often tolerated
better than skeletal halo fixation and also is noted to preserve the normal rotation at C1/C2.
Studies have shown less of a malunion and nonunion rate in the treatment of type II odontoid
fractures with anterior odontoid screw fixation. Osteoporosis, short neck and barrel-chested
anatomy, and fractures that are more than 4 weeks old preclude anterior odontoid fixation.

REFERENCES: Shilpakar S, McLaughlin MR, Haid RW Jr, et al: Management of acute


odontoid fractures: Operative techniques and complication avoidance. Neurosurg Focus
2000;8:e3.
Subach BR, Morone MA, Haid RW Jr, et al: Management of acute odontoid fractures with
single-screw anterior fixation. Neurosurgery 1999;45:812-819.
Fountas KN, Kapsalaki EZ, Karampelas I, et al: Results of long-term follow-up in patients
undergoing anterior screw fixation for type II and rostral type III odontoid fractures. Spine
2005;30:661-669.

27. PREFERRED RESPONSE: 3

DISCUSSION: The carotid tubercle is usually located at the level of C6. The angle of the
mandible is at C1-C2; the hyoid is at C4; the superior portion of the thyroid cartilage is C4-C5;
and the cricoid cartilage is at C6.

REFERENCES: Smith GW, Robinson RA: The treatment of certain cervical-spine disorders by
anterior removal of the intervertebral disc and interbody fusion. J Bone Joint Surg Am
1958;40:607.
An HS: Surgical Exposure and Fusion Techniques of the Spine: Principles and Techniques of
Spine Surgery. Baltimore, MD, Williams and William, 1998, pp 31-62.

28. PREFERRED RESPONSE: 3

DISCUSSION: The postoperative incidence of C5 root palsy after laminoplasty ranges from 5%
to 12%. Other roots also may be affected. The palsies tend to be motor dominant, although
sensory dysfunction and radicular pain are also possible. The palsy may arise during the
immediate postoperative period or up to 20 days later. C5 may be preferentially involved
because it is at the apex of the cervical lordosis. Recovery usually occurs over weeks to months.

REFERENCES: Spivak JM, Connolly PJ (eds): Orthopaedic Knowledge Update: Spine 3.


Rosemont, IL, American Academy of Orthopaedic Surgeons, 2006, pp 235-249.
Uematsu Y, Tokuhashi Y, Matsuzaki H: Radiculopathy after laminoplasty of the cervical spine.
Spine 1998;23:2057-2062.

29. PREFERRED RESPONSE: 4

DISCUSSION: Revision anterior exposure within 2 weeks of total disk replacement incurs
relatively little additional morbidity because adhesion formation is minimal. Surgeons should
have a low threshold for revising implants that are clearly dangerously malpositioned or show
early migration within this 2-week window. Beyond this time period, a revision strategy must be
individualized to the particular clinical situation. A posterior fusion with instrumentation with or
without a laminectomy is currently the most effective salvage procedure.
REFERENCE: Tortolani JP, McAfee PC, Saiedy S: Failures of lumbar disc replacement. Sem
Spine Surg 2006;18:78-86.

30. PREFERRED RESPONSE: 1

DISCUSSION: Kramer and associates conducted a retrospective review during an epidemic


period to identify the risk factors associated with a sudden increase in the rate of surgical site
infections. They found in a multivariate analysis that the use of morphine nerve paste resulted in
a 7.6-fold increase in postoperative surgical wound debridement, and an 11% rate of surgical site
complications.

REFERENCES: Kramer MH, Mangram AJ, Pearson ML, et al: Surgical-site complications
associated with a morphine nerve paste used for postoperative pain control after laminectomy.
Infect Control Hosp Epidemiol 1999;20:183-186.
Lowell TD, Errico TJ, Eskenazi MS: Use of steroids after discectomy may predispose to
infection. Spine 2000;25:516-519.

31. PREFERRED RESPONSE: 2

DISCUSSION: The arrow is pointing to a cystic-appearing structure with high signal intensity
on T 2 -weighted image sequencing. It appears to be contiguous with the hypertrophied right facet
joint, which appears to also have high signal intensity. The mass significantly narrows the right
lateral recess. The high signal intensity suggests that this is a fluid-filled mass. In addition, the
facet joints are degenerative and there is a very mild degree of anterolisthesis on the sagittal
image. These findings make a lumbar synovial cyst the most likely diagnosis. Most lumbar
juxtafacet cysts are observed at the L4-5 level, extradurally and adjacent to the degenerative
facet joint. They may contain synovial fluid and/or extruded synovium. Presentation is
indistinguishable from that of a herniated disk. The etiology of spinal cysts remains unclear, but
there appears to be a strong association between their formation and worsening spinal instability.
They occasionally regress spontaneously and may respond to aspiration and injection of
corticosteroids, though there is a high recurrence rate with nonsurgical management. Synovial
cysts resistant to nonsurgical management should be treated surgically. If the patients
symptoms can be attributable to radicular findings, a microsurgical decompression that limits
further destabilization should suffice. However, if there is significant low back pain attributable
to spinal instability, decompression and fusion remains an appropriate option.

REFERENCES: Banning CS, Thorell WE, Leibrock LG: Patient outcome after resection of
lumbar juxtafacet cysts. Spine 2001;26:969-972.
Deinsberger R, Kinn E, Ungersbock K: Microsurgical treatment of juxta facet cysts of the
lumbar spine. J Spinal Disord Tech 2006;19:155-160.
Khan AM, Synnot K, Cammisa FP, et al: Lumbar synovial cysts of the spine: An evaluation of
surgical outcome. J Spinal Disord Tech 2005;18:127-131.
32. PREFERRED RESPONSE: 4

DISCUSSION: The patients history and physical examination findings are consistent with a
lumbar disk herniation at the L4-5 level. Weakness of the extensor hallucis longus and gluteus
medius are consistent with an L5 lumbar radiculopathy. Nerve root tension signs are also
consistent with sciatica from a lumbar disk herniation. The MRI scans confirm a posterolateral
disk herniation at L4-5, which typically affects the exiting L5 nerve root.

REFERENCES: Hoppenfeld S: Orthopedic Neurology. Philadelphia, PA, JB Lippincott, 1977,


pp 45-74.
Spivak JM, Connolly PJ (eds): Orthopaedic Knowledge Update: Spine 3. Rosemont, IL,
American Academy of Orthopaedic Surgeons, 2006, pp 43-56.

33. PREFERRED RESPONSE: 3

DISCUSSION: The CT scans reveal an occipital-cervical dissociation with subluxation of the


occipitocervical joints bilaterally. Definitive management should consist of an occipital-cervical
fusion with instrumentation. Immobilization in a Philadelphia collar is inadequate for this highly
unstable injury, and halo immobilization, while affording adequate temporary immobilization, is
not appropriate definitive management for this ligamentous injury. The patient does not have an
injury at C4-C5 or C6-C7.

REFERENCES: Jackson RS, Banit DM, Rhyne AL III, et al: Upper cervical spine injuries.
J Am Acad Orthop Surg 2002;10:271-280.
Spivak JM, Connolly PJ (eds): Orthopaedic Knowledge Update: Spine 3. Rosemont, IL,
American Academy of Orthopaedic Surgeons, 2006, pp 201-216.

34. PREFERRED RESPONSE: 5

DISCUSSION: The patient has a burst fracture with probable unilateral entrapment of the cauda
equina within the elements of the fractured lamina. A dural tear is likely in this scenario as well.
It is recommended that this type of burst fracture be treated surgically with laminectomy, freeing
of the entrapped nerve roots, and dural repair followed by stabilization of the fracture by either a
posterior or combined approach. The degree of kyphosis and the extent of anterior canal
compromise does not warrant corpectomy in this patient. Therefore, after completing the
laminectomy and dural repair, posterior fusion and instrumentation should be sufficient to
stabilize the fracture.

REFERENCES: Cammisa FP Jr, Eismont FJ, Green BA: Dural laceration occurring with burst
fractures and associated laminar fractures. J Bone Joint Surg Am 1989;71:1044-1052.
Spivak JM, Connolly PJ (eds): Orthopaedic Knowledge Update: Spine 3. Rosemont, IL,
American Academy of Orthopaedic Surgeons, 2006, pp 201-216.
35. PREFERRED RESPONSE: 1

DISCUSSION: The MRI scans show Modic changes in the L4-L5 vertebral bodies due to
spondylosis. There is no increased fluid signal or enhancement in the L4-L5 disk to suggest
infection or any other pathologic process. Therefore, the patients pain should be treated with a
course of physical therapy and rehabilitation. There is no infection; therefore, IV antibiotics and
debridement are not indicated. Similarly, a pseudomeningocele is not present. A revision
diskectomy is useful for recurrent radiculopathy but would not be helpful for degenerative low
back pain.

REFERENCES: Spivak JM, Connolly PJ (eds): Orthopaedic Knowledge Update: Spine 3.


Rosemont, IL, American Academy of Orthopaedic Surgeons, 2006, pp 319-329.
Shen FH, Samartzis D, Andersson GBJ: Nonsurgical management of acute and chronic low back
pain. J Am Acad Orthop Surg 2006;14:477-487.

36. PREFERRED RESPONSE: 5

DISCUSSION: Cotler and associates reported on the use of awake skeletal traction to reduce
facet fracture-dislocations in 24 patients. Seventeen patients required more than 50 pounds of
traction (the traditional limit) to achieve reduction. More than 100 pounds of traction was
safely used in one-third of the patients in this study. A cadaver study has supported the safe use
of traction with weights in excess of 100 pounds.

REFERENCES: Cotler JM, Herbison GJ, Nasuti JF, et al: Closed reduction of traumatic cervical
spine dislocation using traction weights up to 140 pounds. Spine 1993;18:386-390.
Anderson DG, Vacccaro AR, Gavin K: Cervical orthoses and cranioskeletal traction, in Clark
CR (ed): The Cervical Spine, ed 4. Philadelphia, PA, Lippincott Williams & Wilkins, 2005,
pp 110-121.

37. PREFERRED RESPONSE: 3

DISCUSSION: Although the imaging reveals generalized lumbar spondylosis and stenosis, in
particular at L4-5, the MRI scan of the left hip clearly reveals a stress fracture of the femoral
neck. Therefore, the treatment of choice is non-weight-bearing of the left lower extremity.
During the evaluation of acute back pain, clinicians must include other possibilities within the
differential diagnosis that may mimic mechanical axial back pain; thus, potential complications
from a missed diagnosis can be avoided.

REFERENCES: Wong DA, Transfeldt E: Macnabs Backache, ed 4. Philadelphia, PA,


Lippincott Williams and Wilkins, 2007, pp 339-361.
Spivak JM, Connolly PJ (ed): Orthopaedic Knowledge Update: Spine 3. Rosemont, IL,
American Academy of Orthopaedic Surgeons, 2006, pp 43-56.
38. PREFERRED RESPONSE: 4

DISCUSSION: The patient has symptoms that are more consistent with vascular claudication
than with the pseudoclaudication anticipated from lumbar spinal stenosis. Therefore, the patient
is a candidate for further vascular work-up. The radiographs reveal early spinal stenosis and
spondylolisthesis at L4-5 but also show significant calcification of the iliac arteries, suggestive of
peripheral vascular disease. Vascular claudication is a manifestation of peripheral vascular
disease and presents with crampy leg pain that is exacerbated by physical exertion. The pain is
easily relieved by standing still or sitting. Unlike pseudoclaudication, a forward-flexed posture
and/or sitting does not improve the symptoms. Night pain is common in vascular claudication
due to the elevation of the extremities and patients often report pain improvement by hanging
their extremities in a dependent position. In evaluation of a patient with suspected vascular
claudication, the five Ps of vascular insufficiency should be monitored, including
pulselessness, paralysis, paresthesia, pallor, and pain. While pain and paresthesias can be
common in both vascular claudication and pseudoclaudication, the presence of any of the
remaining symptoms is suggestive of vascular disease.

REFERENCES: Aufderheide TP: Peripheral arteriovascular disease, in Rosen P, Barkin R (eds):


Emergency Medicine: Concepts and Clinical Practice, ed 4. St Louis, MO, Mosby, 1998,
pp 1826-1844.
Mirkovic S, Garfin SR: Spinal stenosis: History and physical examination. Instr Course Lect
1994;43:435-440.

39. PREFERRED RESPONSE: 5

DISCUSSION: Tuberculosis of the spine is seen in 50% to 60% of skeletal disease and is most
commonly found in the lower thoracic or upper lumbar spine. Typically two or more adjacent
bodies are involved as seen in this MRI scan. The disk space is narrowed but still relatively
preserved as opposed to pyogenic infections (black arrow). Epidural extensions often spread
from vertebrae to vertebrae (white arrow); however, the posterior elements are not frequently
involved (arrowhead). Tumors rarely spread to adjacent vertebrae. The anterior and posterior
spread of the infectious process rules out trauma.

REFERENCES: Boachie-Adjei O, Squillante RG: Tuberculosis of the spine. Orthop Clin North
Am 1996;27:95-103.
Currier BL, Eismont FJ: Infections of the spine, in Rothman RH, Simeone FA (eds): The Spine.
Philadelphia, PA, WB Saunders, 1992, p 2614.

40. PREFERRED RESPONSE: 4

DISCUSSION: With the notable exception of fusion for degenerative spondylolisthesis and
scoliosis, there is a paucity of evidence on the indications for spinal fusion in patients undergoing
laminectomy for spinal stenosis. However, it is generally recommended that if the spine is
destabilized (for example by removal of one complete facet joint or by an iatrogenic pars
fracture), spinal fusion should be considered. Although fusion can be considered for a very long
laminectomy, a two-level laminectomy does not represent, by itself, a clear indication for the
addition of a spinal fusion. The repair of a dural tear and the use of nicotine by the patient play
no role in the determination of whether or not to add fusion to a laminectomy procedure.
REFERENCES: Spivak JM, Connolly PJ (eds): Orthopaedic Knowledge Update: Spine 3.
Rosemont, IL, American Academy of Orthopaedic Surgeons, 2006, pp 299-409.
Fischgrund JS, Mackay M, Herkowitz HN, et al: 1997 Volvo Award winner in clinical studies.
Degenerative lumbar spondylolisthesis with spinal stenosis: A prospective, randomized study
comparing decompressive laminectomy and arthrodesis with and without spinal instrumentation.
Spine 1997;22:2807-2812.

41. PREFERRED RESPONSE: 4

DISCUSSION: The sagittal and axial CT scans show a bilateral spondylolysis at L5. The defect
is in the pars interarticularis on the right side but at the base of the pedicle on the left. Having
failed a trial of physical therapy with only a 3-month history of pain, the next most appropriate
step in management should consist of activity modification and bracing in an antilordotic
lumbosacral orthosis. Surgical intervention is reserved for patients who have failed to respond to
a trial of bracing and activity restriction.

REFERENCES: Debnath UK, Freeman BJ, Grevitt MP, et al: Clinical outcome of symptomatic
unilateral stress injuries of the lumbar pars interarticularis. Spine 2007;32:995-1000.
Bono CM: Low-back pain in athletes. J Bone Joint Surg Am 2004;86:382-396.

42. PREFERRED RESPONSE: 4

DISCUSSION: Although symptomatic thoracic disk herniations can affect more caudal
structures, even to the point of paralysis, the pattern of radiating pain has been described as either
following the dermatomal band around the chest or feeling to the patient as if the pain passes
straight anteriorly to the chest wall.

REFERENCE: Skubic JW, Kostuik JP: Thoracic pain syndromes and thoracic disc herniation, in
Frymoyer JW (ed): The Adult Spine: Principles and Practice. New York, NY, Raven Press,
1991, pp 1443-1464.

43. PREFERRED RESPONSE: 4

DISCUSSION: The patient has pain involving the cervical and lumbar spine as well as pain and
swelling in both the knees and ankles. As such, this can be classified as polyarticular arthritis.
The presence of multiple joint symptoms in the lower extremities, the absence of a history of
trauma, and the multiple joints involved direct attention away from the spine as the etiology of
this patients pain. Radiographs of the involved joints are not likely to yield much useful
information to assist with a diagnosis. Likewise, an MRI scan of the lumbar spine is not likely to
provide much information regarding the etiology of the patients condition. When a
rheumatologic illness is suspected, the selective use of confirmatory laboratory testing can aid in
arriving at a correct diagnosis. A presumed case of gout or chondrocalcinosis can be confirmed
by the presence of the appropriate crystals in a joint-fluid aspiration. Because of the patients
recent trip to Costa Rica and the subsequent gastroenteritis, a CBC count, ESR, and CRP should
be ordered to rule out infectious and inflammatory versus noninflammatory conditions.
Rheumatoid factor (RF) in general should only be ordered for patients with polyarticular joint
inflammation for more than 6 weeks. The presence of rheumatoid factor does not indicate
rheumatoid arthritis. Antinuclear antibodies (ANA) should be ordered when a connective tissue
disease such as systemic lupus erythematosus (SLE) is suspected on the basis of specific history
and physical examination findings, such as inflammatory arthritis. Human leukocyte antigen-
B27 (HLA-B27) should be ordered only when the patients history is compatible with ankylosing
spondylitis or Reiters syndrome and this patient had a history of Reiters syndrome.

REFERENCES: Gardner GC, Kadel NJ: Ordering and interpreting rheumatologic laboratory
tests. J Am Acad Orthop Surg 2003;11:60-67.
Shojania K: Rheumatology: 2. What laboratory tests are needed? CMAJ 2000;162:1157-1163.

44. PREFERRED RESPONSE: 1

DISCUSSION: Atlas and associates, in the Maine Lumbar Spine Study, reported that overall,
patients treated initially with surgery reported better outcomes. By 5 years, 19% of surgical
patients had undergone at least one additional lumbar spine operation, and 16% of nonsurgical
patients had opted for at least one lumbar spine operation. At the 5-year follow-up, 70% of
patients initially treated surgically reported improvement in their predominant symptom (back or
leg pain) versus 56% of those initially treated nonsurgically. They also noted that there was no
difference in the proportion of patients receiving disability compensation at the 5-year follow-up.

REFERENCE: Atlas SJ, Keller RB, Chang Y, et al: Surgical and nonsurgical management of
sciatica secondary to a lumbar disc herniation: Five-year outcomes from the Maine Lumbar
Spine Study. Spine 2001;26:1179-1187.

45. PREFERRED RESPONSE: 4

DISCUSSION: The injury shown is a fracture-dislocation and it is highly unstable. In addition


to this concern, spinal epidural hematomas have a much higher incidence in people with
ankylosing spondylitis following knee fracture. It is felt to be due to disrupted epidural veins,
with hypervascular epidural soft tissue in the setting of a rigid spinal canal. Patients with
ankylosing spondylitis may have other significant comorbidities, especially cardiac and
pulmonary, and these should be carefully assessed.

REFERENCES: Ludwig S, Zarro CM: Complications encountered in the management of


patients with ankylosing spondylitis, in Vaccaro AR, Regan JJ, Crawford AH, et al (eds):
Complications of Pediatric and Adult Spine Surgery. New York, NY, Marcel Dekker, 2004,
pp 279-290.
Wu CT, Lee ST: Spinal epidural hematoma and ankylosing spondylitis: Case report and review
of the literature. J Trauma 1998;44:558-561.

46. PREFERRED RESPONSE: 2

DISCUSSION: Retrograde ejaculation is the sequela of an injury to the superior hypogastric


plexus. This structure needs protection, especially during anterior exposure of the lumbosacral
junction. Although the superior hypogastric plexus can be injured with anterior or anterolateral
spine surgery at any lumbar level, it is most at risk with anterior transperitoneal approaches to
the lumbosacral junction. To avoid this complication, the use of monopolar electrocautery
should be avoided during deep dissection in this region. The ideal anterior exposure starts with
blunt dissection just to the medial aspect of the left common iliac vein sweeping the prevertebral
tissues toward the patients right side.

REFERENCES: Flynn JC, Price CT: Sexual complications of anterior fusion of the lumbar
spine. Spine 1984;9:489-492.
Watkins RG (ed): Surgical Approaches to the Spine. New York, NY, Springer-Verlag, 1983,
p 107.
An HS, Riley LH III: An Atlas of Surgery of the Spine. New York, NY, Lippincott Raven,
1998, p 263.

47. PREFERRED RESPONSE: 4

DISCUSSION: The hypoglossal nerve exits from the ansa cervicalis at approximately the C2-3
level and can be injured during retraction up to the C2 level. The superior laryngeal nerve lies at
about C4-5. The facial nerve is much higher. The vagus nerve runs with the internal jugular and
carotid much more laterally. The phrenic nerve exits posteriorly.

REFERENCES: Chang U, Lee MC, Kim DH: Anterior approach to the midcervical spine, in
Kim DH, Henn JS, Vaccaro AR, et al (eds): Surgical Anatomy and Techniques to the Spine.
Philadelphia, PA, Saunders Elsevier, 2006, pp 45-54.
Netter GH: Atlas of Human Anatomy. Summit, NJ, Ciba-Geigy Corporation, 1989.

48. PREFERRED RESPONSE: 5

DISCUSSION: The injury pattern is that of a burst fracture at L1 contiguous with a compression
fracture at T12. There is associated kyphosis and slight spondylolisthesis of T12 on L1.
Treatment of this type of burst fracture in neurologically intact patients is somewhat
controversial, with at least one study demonstrating equal long-term results comparing
nonsurgical treatment to surgical treatment. In this study, however, body casts were used
initially in the nonsurgical group. Moreover, because this patient has multiple fractures, spinal
fracture stabilization should be considered to facilitate early mobilization. Surgical stabilization
and fusion via a posterior approach is the best treatment option in this patient. Anterior
decompression is not necessary since the patient is neurologically intact.

REFERENCES: McLain RF, Benson DR: Urgent surgical stabilization of spinal fractures in
polytrauma patients. Spine 1999;24:1646-1654.
Wood K, Butterman G, Mehbod A, et al: Operative compared with nonoperative treatment of a
thoracolumbar burst fracture without neurological deficit: A prospective, randomized study.
J Bone Joint Surg Am 2003;85:773-781.
Spivak JM, Connolly PJ (eds): Orthopaedic Knowledge Update: Spine 3. Rosemont, IL,
American Academy of Orthopaedic Surgeons, 2006, pp 201-216.

49. PREFERRED RESPONSE: 2

DISCUSSION: Incomplete cord syndromes have variable neurologic findings with partial loss of
sensory and/or motor function below the level of injury. Incomplete cord syndromes include the
anterior cord syndrome, the Brown-Squard syndrome, central cord syndrome, and posterior
cord syndrome. Central cord syndrome is characterized with greater motor weakness in the
upper extremities than in the lower extremities. The pattern of motor weakness shows greater
distal involvement in the affected extremity than proximal muscle weakness. Anterior cord
syndrome involves a variable loss of motor function and pain and/or temperature sensation, with
preservation of proprioception. The Brown-Squard syndrome involves a relatively greater
ipsilateral loss of proprioception and motor function, with contralateral loss of pain and
temperature sensation. Posterior cord syndrome is a rare injury and is characterized by
preservation of motor function, sense of pain, and light touch, with loss of proprioception and
temperature sensation below the level of the lesion. Spinal shock is the period of time, usually
24 hours, after a spinal injury that is characterized by absent reflexes, flaccidity, and loss of
sensation below the level of the injury.

REFERENCES: Penrod LE, Hegde SK, Ditunno JF: Age effect on prognosis for functional
recovery in acute, traumatic central cord syndrome. Arch Phys Med Rehab 1990;71:963-968.
Harrop JS, Sharan A, Ratliff J: Central cord injury: Pathophysiology, management, and
outcomes. Spine J 2006;6:198S-206S.

50. PREFERRED RESPONSE: 2

DISCUSSION: Kayanja and associates, in a number of biomechanical studies, showed that in a


kyphotic spine the strain is located at the apex of the deformity, the force is transmitted to the
superior adjacent vertebrae, and that realignment and cement augmentation effectively normalize
the load transfer.

REFERENCES: Kayanja MM, Ferrara LA, Lieberman IH: Distribution of anterior cortical shear
strain after a thoracic wedge compression fracture. Spine J 2004;4:76-87.
Kayanja MM, Togawa D, Lieberman IH: Biomechanical changes after the augmentation of
experimental osteoporotic vertebral compression fractures in the cadaveric thoracic spine. Spine
J 2005;5:55-63.
Kayanja MM, Schlenk R, Togawa D, et al: The biomechanics of 1, 2, and 3 levels of vertebral
augmentation with polymethylmethacrylate in multilevel spinal segments. Spine 2006;31:769-
774.
Kayanja M, Evans K, Milks R, et al: The mechanics of polymethylmethacrylate augmentation.
Clin Orthop Relat Res 2006;443:124-130.

51. PREFERRED RESPONSE: 3

DISCUSSION: Boden and associates article presents compelling evidence that patients with
rheumatoid arthritis and neurologic deterioration in C1-2 instability are more likely to achieve
some improvement if the posterior atlanto-dens interval is greater than 10 mm on preoperative
studies. All the patients in their series who had neurologic deterioration and a preoperative
posterior atlanto-dens interval of greater than 14 mm achieved complete motor recovery.

REFERENCES: Boden SD, Dodge LD, Bohlman HH, et al: Rheumatoid arthritis of the cervical
spine: A long-term analysis with predictors of paralysis and recovery. J Bone Joint Surg Am
1993;75:1282-1297.
Boden SD, Clark CR: Rheumatoid arthritis of the cervical spine, in Clark CR (ed): The Cervical
Spine, ed 3. Philadelphia, PA, Lippincott Raven, 1998, pp 755-764.
Monsey RD: Rheumatoid arthritis of the cervical spine. J Am Acad Orthop Surg 1997;5:240-
248.

52. PREFERRED RESPONSE: 4

DISCUSSION: Patients with a degenerative spondylolisthesis and severe stenosis who have
failed appropriate nonsurgical management are candidates for surgical intervention. Most
studies show good to excellent results in more than 85% of patients after lumbar decompression
for stenosis. Atlas and associates found that at 8- to 10-year follow-up, leg pain relief and back-
related functional status were greater in those patients opting for surgical treatment of the
stenosis. Similarly, the decision to fuse a spondylolisthetic segment has been supported in the
literature. Herkowitz and Kurz compared decompressive laminectomy alone and decompressive
laminectomy with intertransverse arthrodesis in 50 patients with single-level spinal stenosis and
degenerative spondylolisthesis. They demonstrated good to excellent results in 90% of the fused
group compared to 44% in the nonfusion group. The decision to include instrumentation during
the fusion is more controversial. Whereas the use of instrumentation has shown to improve
fusion rates, it has not been conclusively shown to improve the overall clinical outcomes of
patients.

REFERENCES: Atlas SJ, Keller RB, Wu YA, et al: Long-term outcomes of surgical and
nonsurgical management of lumbar spinal stenosis: 8 to 10 year results from the Maine lumbar
spine study. Spine 2005;30:936-943.
Herkowitz HN, Kurz LT: Degenerative lumbar spondylolisthesis with spinal stenosis: A
prospective study comparing decompression with decompression and intratransverse process
arthrodesis. J Bone Joint Surg Am 1991;73:802-808.
Fischgrund JS, Mackay M, Herkowitz HN, et al: 1997 Volvo Award winner in clinical studies.
Degenerative lumbar spondylolisthesis with spinal stenosis: A prospective, randomized study
comparing decompressive laminectomy and arthrodesis with and without spinal instrumentation.
Spine 1997;22:2807-2812.

53. PREFERRED RESPONSE: 3

DISCUSSION: Dickman and associates classified injuries of the transverse atlantal ligament into
two categories. Type I injuries are disruptions through the substance of the ligament itself. Type
II injuries render the transverse ligament physiologically incompetent through fractures and
avulsions involving the tubercle of insertion of the transverse ligament on the C1 lateral mass.
Type I injuries are incapable of healing without supplemental internal fixation. Type II injuries
can be treated with a rigid cervical orthosis with a success rate of 74%. Surgery may be required
for type II injures that fail to heal with 3 to 4 months of nonsurgical management.

REFERENCES: Findlay JM: Injuries involving the transverse atlantal ligament: Classification
and treatment guidelines based upon experience with 39 injuries. Neurosurgery 1996;39:210.
Dickman CA, Mamourian A, Sonntag VK, et al: Magnetic resonance imaging of the transverse
atlantal ligament for the evaluation of atlantoaxial instability. J Neurosurgery 1991;75:221-227.
54. PREFERRED RESPONSE: 2

DISCUSSION: Unilateral sacralization of the fifth lumbar vertebra was first described by
Bertolotti in 1917. Bertolottis syndrome is present in 12% to 21% of the population. The
altered biomechanics have been postulated to cause low back pain by placing increased stress on
the adjacent cephalad disk, thus contributing to accelerated degenerative disk disease at this
level. It has also been found that the neoarticulation between the enlarged transverse process and
the sacrum and/or ilium may be a source of neural impingement on the exited L5 nerve root and
results in radicular pain syndrome. Brault and associates reported on a case treated surgically at
the Mayo Clinic, in which the pain generator was found to be the contralateral facet joint.

REFERENCES: Brault JS, Smith J, Currier BL: Partial lumbosacral transitional vertebra
resection for contralateral facetogenic pain. Spine 2001;26:226-229.
Quinlan JF, Duke D, Eustace S: Bertolottis syndrome: A cause of back pain in young people.
J Bone Joint Surg Br 2006;88:1183-1186.
Whelan MA, Feldman F: The variant lumbar pedicle. Neuroradiology 1982;22:235-242.

55. PREFERRED RESPONSE: 3

DISCUSSION: The rate of symptomatic degeneration at an adjacent segment warranting either


decompression or arthrodesis was predicted to be 16.5% at 5 years and 36.1% at 10 years based
on a Kaplan-Meier analysis.

REFERENCE: Ghiselli G, Wang J, Bhatia NN, et al: Adjacent segment degeneration in the
lumbar spine. J Bone Joint Surg Am 2004;86:1497-1503.

56. PREFERRED RESPONSE: 1

DISCUSSION: The standard practice in the pharmacologic treatment of a spinal cord injury in
the United States has been the administration of methylprednisolone with an initial bolus of 30
mg/kg followed by 5.4 mg/kg for 24 hours, in accordance with the findings of the second and
third National Acute Spinal Cord Injury Studies (NASCIS). Although the studies have
subsequently drawn criticism for their methodology and outcomes, it has been generally
accepted that beneficial neurologic outcomes were anticipated in patients who were able to start
the protocol within 8 hours of their initial injury. Further improvement was noted in patients
receiving the methylprednisolone within 3 hours of their injury and continuing an infusion for
48 hours. In this patient, who is outside the 8-hour treatment window, no studies have supported
starting the methylprednisolone protocol at this time.

REFERENCES: Braken MB, Shepard MJ, Holford TR, et al: Administration of


methylprednisolone for 24 or 48 hours or tirilazad mesylate for 48 hours in the treatment of acute
spinal cord injury: Results of the third National Acute Spinal Cord Injury Randomized
Controlled Trial. National Acute Spinal Cord Injury Study. JAMA 1997;277:1597-1604.
Kwon BK, Tetzlaff W, Grauer JN, et al: Pathophysiology and pharmacologic treatment of acute
spinal cord injury. Spine J 2004;4:451-464.
57. PREFERRED RESPONSE: 5

DISCUSSION: The patient has obvious signs of progressive myelopathy. Based on her
significant physical examination findings, nonsurgical management will not significantly impact
her outcome. Cervical decompression alone is contraindicated in patients with cervical kyphosis
such as seen here. Anterior cervical fusion is the best option.

REFERENCES: Emery SE, Bohlman HH, Bolesta MJ, et al: Anterior cervical decompression
and arthrodesis for the treatment of cervical spondylotic myelopathy: Two to seventeen-year
follow-up. J Bone Joint Surg Am 1998;80:941-951.
Ferguson RJ, Caplan LR: Cervical spondylotic myelopathy. Neurol Clin 1985;3:373-382.
Herkowitz HN: A comparison of anterior cervical fusion, cervical laminectomy, and cervical
laminoplasty for the surgical management of multiple level spondylotic radiculopathy. Spine
1988;13:774-780.

58. PREFERRED RESPONSE: 2

DISCUSSION: The structure illustrated is the sympathetic chain viewed from an anterolateral
view of the lower lumbar spine. It descends along the anterolateral aspect of the spine into the
pelvis closely adherent to the vertebral column. The spinal nerves, including L5, can be seen
exiting from the foramen. The ureters descend from the kidneys and cross anterior to the iliac
vessels to the bladder.

REFERENCES: Onibokun A, Khoo LT, Holly L: Anterior retroperitoneal approach to the


lumbar spine, in Kim DH, Henn JS, Vaccaro AR, et al (eds): Surgical Anatomy and Techniques
to the Spine. Philadelphia, PA, Saunders Elsevier, 2006, pp 101-105.
Netter GH: Atlas of Human Anatomy. Summit, NJ, Ciba-Geigy Corporation, 1989.

59. PREFERRED RESPONSE: 4

DISCUSSION: The most predictable positive outcome from spinal surgery due to a cauda equina
syndrome is early surgical intervention before any significant neurologic deficit develops. Meta-
analysis studies demonstrate that surgical intervention more than 48 hours after the onset of
cauda equina syndrome show an increased risk for poor outcomes.

REFERENCES: Ahn UM, Ahn NU, Buchowski JM, et al: Cauda equina syndrome secondary to
lumbar disc herniation: A meta-analysis of surgical outcomes. Spine 2000;25:1515-1522.
Kohles SS, Kohles JD, Karp AP, et al: Time-dependent surgical outcomes following cauda
equina syndrome diagnosis: Comments on meta-analysis. Spine 2004;29:1281-1287.

60. PREFERRED RESPONSE: 1

DISCUSSION: The radiograph shows significant prevertebral soft-tissue swelling following a


two-level anterior cervical diskectomy and fusion. The incidence of dysphagia 2 years after
anterior cervical spine surgery is 13.6%. Risk factors for long-term dysphagia after anterior
cervical spine surgery include gender, revision surgeries, and multilevel surgeries. The use of
instrumentation, higher levels, or corpectomy versus diskectomy did not significantly increase
the prevalence of dysphagia. Lee and associates demonstrated that while dysphagia after anterior
cervical spine surgery is a common early finding, it generally decreases significantly by
6 months with nonsurgical management. A minority of patients experience moderate or severe
symptoms by 6 months after the procedure. Female gender and multiple surgical levels have
been identified as risk factors for the development of postoperative dysphagia.

REFERENCES: Lee MJ, Bazaz R, Furey CG, et al: Risk factors for dysphagia after anterior
cervical spine surgery: A two-year prospective cohort study. Spine J 2007;7:141-147.
Bazaz R, Lee MJ, Yoo JU: Incidence of dysphagia after anterior cervical spine surgery:
A prospective study. Spine 2002;27:2453-2458.

61. PREFERRED RESPONSE: 3

DISCUSSION: The proposed mechanisms by which steroids such as methylprednisolone are


thought to prevent neurologic deterioration by limiting secondary insult, include: decreasing the
area of ischemia in the cord, reducing TNF-alpha expression and NF-kB binding activity,
decreasing free radical oxidation and thus stabilizing cell and lysosomal membranes, and
checking the influx of calcium into the injured cells, thus reducing cord edema.

REFERENCES: Slucky AV: Pathomechanics of spinal cord injury. Spine: State Art Rev
1999;13:409-417.
Torg JS, Thibault L, Sennett B, et al: The Nicolas Andry Award. The pathomechanics and
pathophysiology of cervical spinal cord injury. Clin Orthop Relat Res 1995;321:259-269.

62. PREFERRED RESPONSE: 1

DISCUSSION: Cigarette smoking has been directly linked to pseudarthrosis in spinal fusions.
The direct mechanism of action is diminished revascularization of cancellous bone graft.
Additionally, a smaller area of revascularization is seen in these grafts, as well as an increased
area of necrosis. Increased activity of osteoblasts would result in more bone production.
Increased activity of osteocytes would not affect the fusion because osteocytes are mature bone
cells.

REFERENCE: Daftari TK, Whitesides TE Jr, Heller JG, et al: Nicotine on the revascularization
of bone graft: An experimental study in rabbits. Spine 1994;19:904-911.

63. PREFERRED RESPONSE: 4

DISCUSSION: In a study of 427 patients undergoing cervical spine surgery, 12 patients


demonstrated substantial or complete loss of amplitude of the tceMEPs. Ten of those patients
had complete reversal of the loss following prompt intraoperative intervention. SSEP
monitoring failed to identify any changes in one of the two patients that awoke with a new motor
deficit. SSEP changes lagged behind the tceMEP changes in patients in which major changes
were detected by both modalities. TceMEP monitoring was 100% sensitive and 100% specific.
SSEP monitoring was only 25% sensitive and 100% specific.
REFERENCE: Hilibrand AS, Schwartz DM, Sethuraman V, et al: Comparison of transcranial
electric motor and somatosensory evoked potential monitoring during cervical surgery. J Bone
Joint Surg Am 2004;86:1248-1253.

64. PREFERRED RESPONSE: 4

DISCUSSION: In general, a previously healthy patient with an acute onset of nontraumatic


lower back pain does not need diagnostic imaging before proceeding with therapeutic treatment.
In the absence of any red flags during the history and physical examination, such as trauma or
constitutional symptoms (ie, fevers, chills, weight loss), the appropriate treatment for acute onset
lower back pain is purely symptomatic treatment including limited analgesics and early range of
motion. Diagnostic imaging is not necessary unless the initial treatment is unsuccessful and
symptoms are prolonged. Miller and associates suggested that the use of radiographs can lead to
better patient satisfaction but not necessarily better outcomes.

REFERENCES: Miller P, Kendrick D, Bentley E, et al: Cost effectiveness of lumbar spine


radiographs in primary care patients with low back pain. Spine 2002;27:2291-2297.
Wong DA, Transfeldt E: Macnabs Backache, ed 4. Philadelphia, PA, Lippincott Williams and
Wilkins 2007, pp 298-338.

65. PREFERRED RESPONSE: 3

DISCUSSION: Denis divided the sacrum into three zones: zone 1 represents the lateral ala, zone
2 represents the foramina, and zone 3 represents the central canal. A fracture is classified
according to its most medial extension. Those in zone 3 are typically bursting-type fractures or
fracture-dislocations and are most prone to neurologic sequelae.

REFERENCES: Denis F, Davis S, Comfort T: Sacral fractures: An important problem.


A retrospective analysis of 236 cases. Clin Orthop Relat Res 1988;227:67-81.
Wood KB, Denis F: Fractures of the sacrum and coccyx, in Vacarro AR (ed): Fractures of the
Cervical, Thoracic and Lumbar Spine. New York, NY, Marcel Dekker, 2003, pp 473-488.

66. PREFERRED RESPONSE: 1

DISCUSSION: The indications of bisphosphonate therapy in breast cancer patients range from
the correction of hypercalcemia to the prevention of cancer treatment-induced bone loss.
Bisphosphonates reduce metastatic bone pain in at least 50% of patients and can reduce the
frequency of skeletal-related events by 30% to 40%. Osteonecrosis of the jaw could occur in up
to 2.5% of breast cancer patients during long-term bisphosphonate therapy.

REFERENCE: Body JJ: Breast cancer: Bisphosphonate therapy for metastatic bone disease.
Clin Cancer Res 2006;12:6258s-6263s.
67. PREFERRED RESPONSE: 3

DISCUSSION: The patient sustained an incomplete spinal cord injury known as central cord
syndrome. Central cord syndrome characteristically has disproportionate involvement of the
upper extremities with the lower extremities being relatively spared. It is most commonly seen
after cervical injuries in elderly patients with spondylosis and spinal stenosis, often without
fracture. Penrod and associates noted that 23 of 59 patients with central cord syndrome
(ASIA C and D) ultimately walked. The poorest prognosis, however, was in ASIA C patients
older than age 50, in which only 40% walked.

REFERENCES: Penrod LE, Hegde SK, Ditunno JF Jr: Age effect on prognosis for functional
recovery in acute, traumatic central cord syndrome. Arch Phys Med Rehab 1990;71:963-968.
Northrup BE: Acute injuries to the spine and spinal cord: Evaluation and early treatment, in
Clark CR (ed): The Cervical Spine, ed 4. Philadelphia, PA, Lippincott Williams & Wilkins,
2005, p 735.

68. PREFERRED RESPONSE: 2

DISCUSSION: In the patient who is neurologically intact or has an incomplete injury from a
cervical facet dislocation, a closed reduction with weighted tong traction is appropriate when the
patient is awake, alert, and cooperative. Although there is a risk that a cervical facet dislocation
could occur with an underlying cervical disk herniation, Vaccaro and associates have shown that
closed reduction can be safely carried out in the awake, responsive patient. Closed reduction can
be performed in the emergency department with traction with skull tongs or a halo ring. A slow
stepwise application of weight is added until a reduction is achieved. Any worsening of the
neurologic status of the patient requires immediate termination of the closed reduction and
further diagnostic imaging before proceeding with further treatment.

REFERENCES: Vaccaro AR, Falatyn SP, Flanders AE, et al: Magnetic resonance evaluation of
the intervertebral disc, spinal ligaments, and spinal cord before and after closed traction
reduction of cervical spine dislocations. Spine 1999;24:1210-1217.
Hart RA: Cervical facet dislocation: When is magnetic resonance imaging indicated? Spine
2002;27:116-117.
Cotler JM, Herbison GJ, Nasuti JF, et al: Closed reduction of traumatic cervical spine dislocation
using traction weights up to 140 pounds. Spine 1993;18:386-390.

69. PREFERRED RESPONSE: 2

DISCUSSION: In the initial assessment of acute low back pain in adults, no diagnostic testing is
indicated during the first 4 weeks in the absence of red flags for a serious underlying
condition. The purpose of the initial assessment of acute low back pain in adults is to rule out
serious underlying conditions presenting as low back pain. The Agency for Healthcare Policy
and Research, in its 1994 clinical practice guideline, identified four serious conditions that may
present with low back pain, including fracture, tumor, infection, and cauda equina syndrome.
This patient has five red flags for a spinal tumor as a possible etiology of his low back pain,
including age of older than 50 years, constitutional symptoms (recent weight loss), pain worse
when supine, severe nighttime pain, and a history of cancer. Of these, his history of cancer is
most significant, as greater than 90% of spinal tumors are metastatic. In order of frequency,
breast, prostate, lung, and kidney make up approximately 80% of all secondary spread to the
spine. In the presence of red flags for tumor or infection, it is recommended that the clinician
obtain a CBC count, ESR, and a urinalysis. If these are within normal limits and suspicions still
remain, consider consultation or seek further evidence with a bone scan, radiographs, or
additional laboratory studies. Negative radiographs alone are insufficient to rule out disease. If
radiographs are positive, the anatomy can be better defined with MRI.

REFERENCES: Agency for Health Care Policy and Research, Bigos SJ (ed): Acute Low Back
Problems in Adults. Rockville, MD, US Department of Health and Human Services, AHCPR
Publication 95-0642, Clinical Practice Guideline #14, 1994.
Gertzbein SD: Metastatic spine tumors, in Herkowitz HN, Dvorak J, Bell G, et al (eds): The
Lumbar Spine, ed 3. Philadelphia, PA, Lippincott Williams & Wilkins, 2004, pp 792-802.

70. PREFERRED RESPONSE: 4

DISCUSSION: Continuous fluoroscopy and cine radiography expose the patient and personnel
to markedly increased levels of direct and scatter radiation exposure. Continuous fluoroscopy
should be limited to only what is absolutely needed for safe completion of the procedure. By
orienting the cathode ray tube beneath the patient and placing the image intensifier as close as
clinically possible to the patient, scatter radiation exposure to the personnel is minimized.

REFERENCE: Wagner L, Archer B: Minimizing Risks from Fluoroscopic X-rays: A


Credentialing Program for Anesthesiologists, Cardiologists, Surgeons, Radiologists, and
Urologists, ed 3. The Woodlands, TX, Partners in Radiation Management, 2000.

71. PREFERRED RESPONSE: 3

DISCUSSION: The MRI scans reveal a large postoperative hematoma causing significant thecal
compression. An epidural hematoma with neurologic deficit is a surgical emergency requiring
immediate evacuation of the hematoma. Although the incidence of postoperative epidural
hematomas is rare, the consequences of a missed diagnosis can be catastrophic. Early
recognition and evacuation are essential in preserving or restoring neurologic function. Uribe
and associates attributed delayed postoperative hematomas to previous multiple lumbar surgeries
as a possible contributing factor.

REFERENCES: Yi S, Yoon do H, Kim KN, et al: Postoperative spinal epidural hematoma: Risk
factor and clinical outcome. Yonsei Med J 2006;47:326-332.
Uribe J, Moza K, Jimenez O, et al: Delayed postoperative spinal epidural hematomas. Spine J
2003;3:125-129.
72. PREFERRED RESPONSE: 5

DISCUSSION: Although the spinal canal has been penetrated, the lateral masses are intact
bilaterally with only partial destruction of the vertebral body and penetration of the lamina on
one side, thus the cervical spine is not unstable and surgical stabilization is not indicated. Dural
repair is not indicated since there is no external cerebrospinal fluid leakage. Surgical treatment
should be based on the need to treat extraspinal pathology only.

REFERENCES: Bono CM, Heary RF: Gunshot wounds to the spine. Spine J 2004;4:230-240.
Punjabi MM, Jue JJ, Dvorak J, et al: Cervical spine kinematics and clinical instability, in Clark
CR (ed): The Cervical Spine, ed 4. Philadelphia, PA, Lippincott Williams & Wilkins, 2005,
pp 55-87.

73. PREFERRED RESPONSE: 4

DISCUSSION: Symptomatic herniations of the thoracic spine are much less common than those
of the cervical or lumbar region. They tend to occur most commonly during the third to fifth
decades of life and although they can be found at all levels, they are most common in the lower
third near the thoracolumbar region. Posterior laminectomy and disk excision has the highest
rate of neurologic deterioration and is not recommended. Multiple studies have shown that
herniated thoracic disks can be found at one or more levels in 40% of asymptomatic individuals.

REFERENCES: Shah RP, Grauer JN: Thoracoscopic excision of thoracic herniated disc, in
Vaccaro AR, Bono CM (eds): Minimally Invasive Spine Surgery. New York, NY, Informa
Healthcare, 2007, pp 73-80.
Bohlman HH, Zdeblick TA: Anterior excision of herniated thoracic discs. J Bone Joint Surg Am
1988;70:1038-1047.

74. PREFERRED RESPONSE: 5

DISCUSSION: Patient safety and prevention of medical errors is a major focus of recent national
advocacy groups. Analysis has shown that the most common sentinel event in spine surgery is
surgery on the wrong level. Therefore, it is recommended that every patient have the surgical
site signed, the level of surgery marked intraoperatively, and a radiograph taken. Surgery on the
wrong level is most likely to occur in single-level decompressive procedures.

REFERENCES: Wong DA, Watters WC III: To err is human: Quality and safety issues in spine
care. Spine 2007;32:S2-S8.
Wong DA: Spinal surgery and patient safety: A systems approach. J Am Acad Orthop Surg
2006;14:226-232.

75. PREFERRED RESPONSE: 3

DISCUSSION: Posterior screw fixation of the upper cervical spine has gained a great deal of
popularity due to its stable fixation, obviating the use of halo vest immobilization, and its high
fusion rates. The use of screws in this location, however, has introduced a whole new set of
potential complications. Vertebral artery injury is one of the most feared complications
associated with screws in the C1/C2 region. This structure, however, is lateral and posterior at
the C2 level and then penetrates the foramen transversarium of C1 to lie cephalad to the arch of
C1 before entering the foramen magnum. It is the internal carotid artery that lies immediately
anterior to the arch of C1 that is particularly at risk by anterior penetration of C1 lateral mass or
C1-C2 transarticular screws as demonstrated by Currier and associates. The internal carotid
artery lies posterior to the pharynx. The external carotid artery and the glossopharyngeal nerve
are not at risk with this method of fixation.

REFERENCES: Currier BL, Todd LT, Maus TP, et al: Anatomic relationship of the internal
carotid artery to the C1 vertebra: A case report of cervical reconstruction for chordoma and pilot
study to assess the risk of screw fixation of the atlas. Spine 2003;28:E461-E467.
Grant JC: Grants Atlas of Anatomy, ed 6. Baltimore, MD, Williams & Wilkins, 1972.
Harms J, Melcher RP: Posterior C1-C2 fusion with polyaxial screw and rod fixation. Spine
2001;26:2467-2471.

76. PREFERRED RESPONSE: 2

DISCUSSION: Halo fixation is the most rigid form of cervical orthosis, but complications can
arise from improper placement of the initial halo ring. A relatively safe zone for anterior pin
placement is located 1 cm above the orbital rim and superior to the lateral two thirds of the orbit.
This position avoids the supraorbital and supratrochlear nerves and arteries over the medial one
third of the orbit. The more lateral positions in the temporal fossa have very thin bone and can
interfere with the muscles of mastication. Posterior pin site locations are less critical; positioning
on the posterolateral aspect of the skull, diagonal to the contralateral anterior pins, is generally
desirable.

REFERENCES: Botte MJ, Byrne TP, Abrams RA, et al: Halo skeletal fixation: Techniques of
application and prevention of complications. J Am Acad Orthop Surg 1996;4:44-53.
Garfin SR, Botte MJ, Nickel VL: Complications in the use of the halo fixation device. J Bone
Joint Surg Am 1987;69:954.

77. PREFERRED RESPONSE: 3

DISCUSSION: The patient has lumbar spinal stenosis and the MRI scans reveal the pathology at
L4-5, which is secondary to posterior disk bulging and hypertrophy and infolding of the
ligamentum flavum, as well as degenerative facet arthrosis. The degree of spinal stenosis is
moderate and his symptoms are positional in nature. Tadokoro and associates reported on a
prospective study of 89 patients older than 70 years of age who underwent nonsurgical
management for lumbar spinal stenosis. They found the prognosis to be relatively good with
patients scoring at excellent or good for activities of daily living at final follow-up.
However, they did note that patients with a complete block on myelography did not respond
favorably to nonsurgical management. Amundsen and associates reported on a 10-year
prospective study comparing surgical care to nonsurgical management. They concluded that,
while the long-term results largely favored surgical treatment, more than half of the
nonsurgically managed patients had a satisfactory outcome. They also concluded that a delay of
surgery for some months did not worsen the prognosis. Therefore, their recommendation was for
an initial primarily nonsurgical approach.
REFERENCES: Amundsen T, Weber H, Nordal HJ, et al: Lumbar spinal stenosis: Conservative
or surgical management? A prospective 10-year study. Spine 2000;25:1424-1435.
Hilibrand AS, Rand N: Degenerative lumbar stenosis: Diagnosis and management. J Am Acad
Orthop Surg 1999;7:239-249.
Tadokoro K, Miyamoto H, Sumi M, et al: The prognosis of conservative treatments for lumbar
spinal stenosis: Analysis of patients over 70 years of age. Spine 2005;30:2458-2463.

78. PREFERRED RESPONSE: 4

DISCUSSION: The smallest pedicle isthmic width is at L1, whereas T12 has the largest pedicle
width in the upper lumbar and lower thoracic spine. Although smaller in diameter than T12,
both T10 and T11 have larger pedicle widths than L1.

REFERENCE: Ofiram E, Polly DW, Gilbert TJ Jr, et al: Is it safe to place pedicle screws in the
lower thoracic spine than in the upper lumbar spine? Spine 2007;32:49-54.

79. PREFERRED RESPONSE: 3

DISCUSSION: Kondrashov and associates noted stable good outcomes at 4 years in 14 of 18


patients treated with X-STOP interspinous process decompression as defined as an improvement
over preoperative Oswestry scores of 15 points or more. Similar results were seen after 1 year in
a European study by Siddiqui and associates. Exclusion and inclusion criteria for these studies
varied somewhat, but cauda equina syndrome was the only exclusion criteria listed in both
studies. All of the other choices did not represent exclusion criteria in either study.

REFERENCES: Kondrashov DG, Hannibal M, Hsu KY, et al: Interspinous process


decompression with the X-STOP device for lumbar spinal stenosis: A 4-year follow-up study.
J Spinal Disord Tech 2006;19:323-327.
Siddiqui M, Smith FW, Wardlaw D: One-year results of X Stop interspinous implant for the
treatment of lumbar spinal stenosis. Spine 2007;32:1345-1348.

80. PREFERRED RESPONSE: 3

DISCUSSION: It has been traditionally taught that a left-sided approach to the anterior cervical
spine is associated with a lower incidence of injury compared to the right-sided approach. This
is due in part to the anatomic differences in the path the recurrent laryngeal nerve (RLN) takes
on the right as compared to the left. Both nerves ascend in the tracheoesophageal groove after
branching off the vagus nerve in the upper thorax. The left-sided RLN loops around the aortic
arch and stays relatively medial as compared to the right-sided RLN which loops around the
right subclavian artery and is somewhat more lateral at this point, and therefore is theoretically
more vulnerable as it ascends toward the larynx before becoming protected in the
tracheoesophageal groove. Furthermore, the variant of a nonrecurrent inferior laryngeal nerve
branching directly off the vagus nerve at the level of the midcervical spine is much more
common on the right than the left. Despite this reasoning, there has been no clinical evidence to
suggest that laterality of approach for anterior cervical surgery makes any difference in the
incidence of vocal cord paralysis. Furthermore, two recent studies have shown that the incidence
of RLN injury and vocal cord paralysis is equal with either side of approach.
REFERENCES: Beutler WJ, Sweeney CA, Connolly PJ: Recurrent laryngeal nerve injury with
anterior cervical spine surgery risk with laterality of surgical approach. Spine 2001;26:1337-
1342.
Kilburg C, Sullivan HG, Mathiason MA: Effect of approach side during anterior cervical
discectomy and fusion on the incidence of recurrent laryngeal nerve injury. J Neurosurg Spine
2006;4:273-277.

81. PREFERRED RESPONSE: 2

DISCUSSION: The imaging studies show a fracture-dislocation. Surgical treatment of this


injury consists of a decompression reduction, stabilization, and fusion. A posterolateral
decompression can also be performed as necessary. An isolated anterior procedure in this type
of injury is contraindicated. The anterior longitudinal ligament is most likely intact; therefore, an
anterior procedure further destabilizes the spine. Reduction by an anterior approach would also
be difficult. Nonsurgical management of the neurologic injury in this patient is not indicated.

REFERENCES: Theiss SM: Thoracolumbar and lumbar spine trauma, in Stannard JP, Schmidt
AH, Kregor PJ (eds): Surgical Treatment of Orthopaedic Trauma. New York, NY, Thieme,
2007, pp 179-207.
Spivak JM, Connolly PJ (eds): Orthopaedic Knowledge Update: Spine 3. Rosemont, IL,
American Academy of Orthopaedic Surgeons, 2006, pp 201-216.

82. PREFERRED RESPONSE: 2

DISCUSSION: The recently published SPORT trial verifies that surgical treatment of
symptomatic disk herniations is associated with early and sustained pain relief. The trial also
verifies that nonsurgical management is associated with improved symptoms as well. Nerve root
injury, recurrent herniation, and diskitis are known complications of surgery, but all are less
common than described above.

REFERENCE: Weinstein JN, Lurie JD, Tosteson TD, et al: Surgical vs nonoperative treatment
for lumbar disk herniation: The Spine Patient Outcomes Research Trial (SPORT) observational
cohort. JAMA 2006;296:2451-2459.

83. PREFERRED RESPONSE: 2

DISCUSSION: Clearance of the cervical spine can be difficult in the obtunded or unresponsive
patient. Various trauma series have been reported to detect up to 95% of cervical fractures but
only when ideal imaging views have been obtained, which is not often possible in the
unresponsive or uncooperative patient. Passively performed cervical flexion-extension under
live fluoroscopy has been suggested but is not without inherent risk in the potentially unstable
cervical spine. CT of the cervical spine has gained acceptance for the evaluation of these
patients given the excellent evaluation of the osseous anatomy and for the common availability
in most emergency departments. Sanchez and associates, using a protocol to evaluate for
cervical spine injuries after blunt trauma, were able to detect 99% of cervical fractures with
100% specificity.
REFERENCES: Chiu WC, Haan JM, Cushing BM, et al: Ligamentous injuries of the cervical
spine in unreliable blunt trauma patients: Incidence, evaluation, and outcome. J Trauma
2001;50:457-463.
Sanchez B, Waxman K, Jones T, et al: Cervical spine clearance in blunt trauma: Evaluation of a
computed tomography-based protocol. J Trauma 2005;59:179-183.
Nunez D Jr: Value of complete cervical helical computed tomographic scanning in identifying
cervical spine injury in the unevaluable blunt trauma patient with multiple injuries:
A prospective study. J Trauma 2000;48:988-989.

84. PREFERRED RESPONSE: 5

DISCUSSION: When attempting a revision anterior cervical approach from the side opposite the
original approach, it is important to evaluate the function of the vocal cords. If this evaluation
reveals dysfunction of the vocal cord on the side of the original approach, then an approach on
the contralateral side should not be attempted. Injury to the stellate ganglion, which causes a
Horners syndrome, should not preclude an approach on the contralateral side. While the side of
the symptomatology can influence the surgeons choice as to the side of an anterior approach, it
does not preclude a certain approach. When approaching the lower cervical spine from the right
side, the recurrent laryngeal nerve can cross the surgical field and should be preserved.
Excessive intraoperative pressure on the esophagus can increase the incidence of dysphagia, but
its incidence is no different with either approach.

REFERENCES: Spivak JM, Connolly PJ (eds): Orthopaedic Knowledge Update: Spine 3.


Rosemont, IL, American Academy of Orthopaedic Surgeons, 2006, pp 387-394.
Edwards CC II, Riew KD, Anderson PA, et al: Cervical myelopathy: Current diagnostic and
treatment strategies. Spine J 2003;3:68-81.

85. PREFERRED RESPONSE: 5

DISCUSSION: The CT scans show a burst fracture that results from an axial load injury. The
radiographic hallmark of a burst fracture is compression of the posterior cortex of the vertebral
body with retropulsion of bone into the spinal canal. AP radiographs often show widening of the
interpedicular distance with a fracture of the lamina.

REFERENCES: Theiss SM: Thoracolumbar and lumbar spine trauma, in Stannard JP, Schmidt
AH, Kregor PJ (eds): Surgical Treatment of Orthopaedic Trauma. New York, NY, Thieme,
2007, pp 179-207.
Spivak JM, Connolly PJ (eds): Orthopaedic Knowledge Update: Spine 3. Rosemont, IL,
American Academy of Orthopaedic Surgeons, 2006, pp 201-216.

86. PREFERRED RESPONSE: 2

DISCUSSION: In examining a traditional Muslim woman, a male physician should have another
woman present, and the patients husband, if possible. Only the affected limb or area needing
examination should be exposed.

REFERENCE: Jimenez R, Lewis VO (eds): Culturally Competent Care Guidebook. Rosemont,


IL, American Academy of Orthopaedic Surgeons, 2007.
87. PREFERRED RESPONSE: 4

DISCUSSION: The hallmark of neurogenic shock is hypotension without tachycardia. It is


associated most commonly with high cervical spinal cord injuries and results from loss of
function of the sympathetic nervous system. Because the peripheral vasculature is dilated due to
loss of its sympathetic tone, continued rapid administration of fluid corrects the hypotension and
can quickly lead to fluid overload and congestive heart failure. Therefore, neurogenic shock is
best treated by the use of pressors. Cardioversion or administration of antibiotics or systemic
steroids is not appropriate treatment for this patients hypotension.

REFERENCES: Spivak JM, Connolly PJ (eds): Orthopaedic Knowledge Update: Spine 3.


Rosemont, IL, American Academy of Orthopaedic Surgeons, 2006, pp 179-187.
Nockels RP: Nonoperative management of acute spinal cord injury. Spine 2001;26:S31-S37.

88. PREFERRED RESPONSE: 5

DISCUSSION: Thoracic pedicles typically are angled 25 degrees medially at T 1 so the


starting point is more lateral. T 2 angles about 15 degrees, and then the pedicles average about
5 to 7 degrees down to T10. At T11 and 12, the angulation is minimal.

REFERENCES: Weinstein L: Pediatric Spine Principles and Practice. New York, NY, Raven
Press, 1994, pp 1659-1681.
Lenke LG, Orchowski J: Segmental posterior spinal instrumentation: Thoracic spine to sacrum,
in Frymoyer JW, Wiesel SW (eds): The Adult and Pediatric Spine, ed 3. Philadelphia, PA,
Lippincott Williams and Wilkins, 2004, pp 537-552.

89. PREFERRED RESPONSE: 1

DISCUSSION: The omohyoid muscle crosses the surgical field from inferior lateral to anterior
superior traveling from the scapula to the hyoid bone and may need to be transected. The
posterior digastric crosses the field as well but higher near C3-4. The other muscles run
longitudinally.

REFERENCES: Chang U, Lee MC, Kim DH: Anterior approach to the midcervical spine, in
Kim DH, Henn JS, Vaccaro AR, et al (eds): Surgical Anatomy and Techniques to the Spine.
Philadelphia, PA, Saunders Elsevier, 2006, pp 45-56.
Netter GH: Atlas of Human Anatomy. Summit, NJ, Ciba-Geigy Corporation, 1989.

90. PREFERRED RESPONSE: 4

DISCUSSION: The annulus fibrosis has a multilayer lamellar architecture mode of type I
collagen fibers. Each successive layer is oriented at 30 degrees to the horizontal in the opposite
direction, leading to a criss-cross type pattern. This composition allows the annulus, which
has the highest tensile modulus, to resist torsional, axial, and tensile loads.
REFERENCE: Rhee JM, Schaufele M, Abdu WA: Radiculopathy and the herniated lumbar disc:
Controversies regarding pathophysiology and management. J Bone Joint Surg Am
2006;88:2070-2080.

91. PREFERRED RESPONSE: 4

DISCUSSION: When patients are compared at 5 years follow-up, there are no statistically
significant differences between the two groups with respect to kyphosis, the degree of
retropulsed bone resorption, pain and function levels, or the ability to return to work.
Nonsurgical management of stable neurologically intact burst fractures has a very low incidence
of complications.

REFERENCES: Wood K, Butterman G, Mehbod A, et al: Operative compared with


nonoperative treatment of a thoracolumbar burst fracture without neurological deficit: A
prospective, randomized study. J Bone Joint Surg Am 2003;85:773-781.
Shen WJ, Liu TJ, Shen YS: Nonoperative treatment versus posterior fixation for thoracolumbar
junction burst fractures without neurologic deficit. Spine 2001;26:1038-1045.

92. PREFERRED RESPONSE: 5

DISCUSSION: The MRI scans reveal advanced degenerative disk disease at L5-S1.
Nonsurgical management has failed to provide relief and the patient is quite debilitated as a
result of her back pain. Fritzell and associates demonstrated that in a well-informed and selected
group of patients with severe low back pain, lumbar fusion can diminish pain and decrease
disability more efficiently than commonly used nonsurgical treatments. In a recent updated
Cochrane Review of surgery for degenerative lumbar spondylosis, it was noted that while Fritzell
and associates appeared to provide strong evidence in favor of fusion, a more recent trial by Brox
and associates demonstrated no difference between those patients undergoing lumbar fusion
compared to those receiving cognitive intervention and exercise. The Cochrane Review suggests
that this may reflect a difference between the control groups. Fritzell and associates compared
lumbar fusion to standard 1990s usual care, whereas Brox and associates compared lumbar
fusion to a modern rehabilitation program. Bear in mind that this patient is a certified athletic
trainer and runs a hospital health fitness department; therefore, at least for purposes of this
question, it can be assumed that she has participated in a modern rehabilitation program. The
Cochrane Review goes on to state that preliminary results of three small trials of intradiskal
electrotherapy suggest that it is ineffective and that preliminary data from three trials of disk
arthroplasty do not permit firm conclusions.

REFERENCES: Gibson JN, Waddell G: Surgery for degenerative lumbar spondylosis: Updated
Cochrane Review. Spine 2005;30:2312-2320.
Fritzell P, Hagg O, Wessberg P, et al: 2001 Volvo Award Winner in Clinical Studies: Lumbar
fusion versus nonsurgical treatment for chronic low back pain: A multicenter randomized
controlled trial from the Swedish Lumbar Spine Study Group. Spine 2001;26:2521-2532.
Brox JI, Sorensen R, Friis A, et al: Randomized clinical trial of lumbar instrumented fusion and
cognitive intervention and exercises in patients with chronic low back pain and disc
degeneration. Spine 2003;28:1913-1921.
93. PREFERRED RESPONSE: 1

DISCUSSION: The MRI scan reveals a L4-L5 foraminal disk herniation originating from the
L4-5 disk space that has migrated up into the foramen, compressing the left L4 nerve root. There
is normal distribution of the roots in the cerebrospinal fluid, excluding arachnoiditis as a
diagnosis, and disk herniation in this location would not result in cauda equina syndrome or
myelopathy.

REFERENCE: McCullouch JA, Transfeldt EE: Macnabs Backache, ed 3. Philadelphia, PA,


Williams and Wilkins, 1997, pp 569-608.

94. PREFERRED RESPONSE: 3

DISCUSSION: Intradiskal electrothermal therapy (IDET) initial clinical results were reported in
2000. The early case series were quite encouraging with reported therapeutic success rates of
60% to 80%. Early enthusiasm was high as IDET provided a nonsurgical treatment option for an
otherwise complex and difficult clinical entity, chronic diskogenic low back pain. The actual
mechanism of action was not well understood, and while the theoretic explanation made good
sense, it did not hold up under laboratory testing. Soon clinical results from the field did not
meet the high expectations set by the developers of the technique. Since those early case studies,
a few level I evidence studies have been conducted, one by Freeman and associates and one by
Pauza and associates. These randomized, placebo-controlled trials demonstrated no significant
benefit of IDET over the placebo.

REFERENCES: Freeman BJ, Fraser RD, Cain CM, et al: A randomized, double-blind,
controlled trial: Intradiscal electrothermal therapy versus placebo for the treatment of chronic
discogenic low back pain. Spine 2005;30:2369-2377.
Pauza KJ, Howell S, Dreyfuss P, et al: A randomized, placebo-controlled trial of intradiscal
electrothermal therapy for the treatment of discogenic low back pain. Spine J 2004;4:27-35.
Wetzel FT, McNally TA: Treatment of chronic discogenic low back pain with intradiskal
electrothermal therapy. J Am Acad Orthop Surg 2003;11:6-11.

95. PREFERRED RESPONSE: 1

DISCUSSION: Hypothenar atrophy is a nonspecific sign that can be seen in ulnar neuropathy,
C8 radiculopathy, or even cervical myelopathy; however, the atrophy usually is not unilateral
and includes other muscle groups. The Spurling test is an excellent method of eliciting cervical
radicular pain but involves hyperextension and ipsilateral rotation of the cervical spine, resulting
in nerve root compression by reducing the cross-sectional area of the ipsilateral neuroforamen.
Tinels sign at the levator scapulae, if present, is indicative of an upper cervical (C3 or C4)
radiculopathy. A subluxable ulnar nerve at the cubital tunnel, while often asymptomatic, points
toward cubital tunnel syndrome as an etiology for this patients pain. The shoulder abduction
relief (SAR) sign (relief of upper extremity pain with shoulder abduction) is virtually
pathognomic of cervical radiculopathy because this maneuver results in relaxation of a
compressed and/or inflamed cervical nerve root. The SAR sign is the converse analog of the
straight leg raising sign in the lumbar examination for lumbar radiculopathy, as it relieves
tension in the nerve root, thereby relieving symptoms.
REFERENCES: Ducker TB, Zeidman SM: Neurologic and functional evaluation, in Clark CR
(ed): The Cervical Spine, ed 3. Philadelphia, PA, Lippincott Raven, 1998, pp 143-161.
An HS: Clinical presentation of discogenic neck pain, radiculopathy, and myelopathy, in Clark
CR (ed): The Cervical Spine, ed 3. Philadelphia, PA, Lippincott Raven, 1998, pp 755-764.
Hoppenfeld S: Physical examination of the cervical spine and temporomandibular joint, in
Physical Examination of the Spine and Extremities. New York, NY, Appleton-Century-Crofts,
1976, pp 105-132.

96. PREFERRED RESPONSE: 4

DISCUSSION: The patients neurologic examination is consistent with a C7 radiculopathy on


the right side. In a patient with this symptom complex in the absence of trauma, a cervical disk
herniation is the most common etiology for a C7 radiculopathy. There are eight cervical nerve
roots and the C7 nerve exits at the C6-7 disk space and is most frequently impinged by a disk
herniation at this level.

REFERENCES: Houten JK, Errico TJ: Cervical spondylotic myelopathy and radiculopathy:
Natural history and clinical presentation, in Clark CR (ed): The Cervical Spine, ed 4.
Philadelphia, PA, Lippincott Williams & Wilkins, 2005, pp 985-990.
Hoppenfeld S: Orthopaedic Neurology: A Diagnostic Guide to Neurologic Levels. Philadelphia,
PA, JB Lippincott, 1977, pp 7-43.

97. PREFERRED RESPONSE: 2

DISCUSSION: Patient positioning that results in local nerve compression, plexus traction, or
improper neck alignment is the most common nonanesthetic-related cause of changes in
intraoperative neurophysiologic monitoring data during spinal surgery.

REFERENCES: Jones SC, Fernau R, Woeltjen BL: Use of somatosensory evoked potentials to
detect peripheral ischemia and potential injury resulting from positioning of the surgical patient:
Case reports and discussion. Spine J 2004;4:360-362.
Schwartz DM, Sestokas AK, Hilibrand AS, et al: Neurophysiological identification of position-
induced neurologic injury during anterior cervical spine surgery. J Clin Monit Comput
2006;20:437-444.

98. PREFERRED RESPONSE: 2

DISCUSSION: The surgeon perforated the anterior longitudinal ligament and injured the
common iliac artery. Bingol and associates described injuries to the vascular structures during
lumbar disk surgery. The common iliac artery was most commonly affected and constituted
76.9% of injuries.

REFERENCE: Bingol H, Cingoz F, Yilmaz AT, et al: Vascular complications related to lumbar
disc surgery: J Neurosurg 2004;100:249-253.
99. PREFERRED RESPONSE: 3

DISCUSSION: In the absence of a cauda equina syndrome or progressive weakness, the best
indication for surgical management is refractory radicular pain. Surgical decision-making should
not be based on the size of the herniation. Large extruded herniations tend to resolve more
predictably than smaller herniations. Stable motor weakness and numbness resolve similarly in
both surgical and nonsurgical management, although surgery hastens the process. When
intractable radicular pain is the strict indication for surgery, surgical intervention provides
substantial and more rapid pain relief than nonsurgical care.

REFERENCES: Rhee JM, Schaufele M, Abdu WA: Radiculopathy and the herniated lumbar
disc: Controversies regarding pathophysiology and management. J Bone Joint Surg Am
2006;88:2070-2080.
Atlas SJ, Keller RB, Wu YA, et al: Long-term outcomes of surgical and nonsurgical
management of sciatica secondary to a lumbar disc herniation: 10 year results from the Maine
lumbar spine study. Spine 2005;30:927-935.

100. PREFERRED RESPONSE: 5

DISCUSSION: A thoracotomy in an adult with idiopathic scoliosis causes a reduction in


pulmonary function that often does not return to preoperative levels. What pulmonary function
that does recover, recovers over many months. Long-term improvement in pulmonary function,
compared to preoperative function, is rarely seen. This should be considered in planning surgical
intervention in adults with scoliosis.

REFERENCES: Graham EJ, Lenke LG, Lowe TG, et al: Prospective pulmonary function
evaluation following open thoracotomy for anterior spinal fusion in adolescent idiopathic
scoliosis. Spine 2000;25:2319-2325.
Kishan S, Bastrom T, Betz RR, et al: Thoracoscopic scoliosis surgery affects pulmonary function
less than thoracotomy at 2 years postsurgery. Spine 2007;32:453-458.
1. Which is the best initial study for the diagnostic evaluation of diskogenic low back pain?

1- MRI
2- Diskography
3- CT-diskography
4- Radiography
5- CT

PREFERRED RESPONSE: 4

DISCUSSION: Radiography is the best initial study for the evaluation of diskogenic low back
pain. The normal degenerative process can be evaluated. Vacuum phenomenon may be found
within the disk space. Other possible sources for back pain should also be evaluated. The other
tests may be beneficial but represent later imaging options.

REFERENCE: Spivak JM, Connolly PJ (eds): Orthopaedic Knowledge Update: Spine 3.


Rosemont, IL, American Academy of Orthopaedic Surgeons, 2006, pp 319-329.
2. A patient who is an observant Jehovahs Witness requires major surgery for scoliosis that
will likely result in significant blood loss. Which of the following might the patient
consider allowing the surgical team to use?

1- Transfusion of whole blood


2- Transfusion of packed red blood cells
3- A cell saver with continuity maintained in a closed circuit
4- Transfusion of plasma
5- Transfusion of platelets

PREFERRED RESPONSE: 3

DISCUSSION: Jehovahs Witnesses will not accept the transfusion of blood or blood products
such as packed red or white cells, platelets, or plasma. However, many Jehovahs Witnesses will
accept the use of a cell saver in a closed circuit.

REFERENCES: Jimenez R, Lewis VO (eds): Culturally Competent Care Guidebook. Rosemont,


IL, American Academy of Orthopaedic Surgeons, 2007.
www.watchtower.org. Official Web Site of Jehovahs Witnesses. Link verified as active as of
August 8, 2008.
3. Which of the following is a contraindication to laminoplasty in a patient with cervical
spondylotic myelopathy?

1- Space available for the cord of less than 8 mm


2- Ossification of the posterior longitudinal ligament
3- Fixed cervical kyphosis
4- Previous posterior surgery
5- Concomitant cervical radiculopathy

PREFERRED RESPONSE: 3

DISCUSSION: Laminoplasty or any posterior decompressive procedure is contraindicated in


patients with cervical spondylotic myelopathy and cervical kyphosis. The residual kyphotic
posture of the cervical spine results in persistent spinal cord compression. The other choices are
not contraindications for laminoplasty. Concomitant cervical radiculopathy can be addressed at
the time of laminoplasty with a keyhole foraminotomy.

REFERENCES: Emery SE: Cervical spondylotic myelopathy: Diagnosis and treatment. J Am


Acad Orthop Surg 2001;9:376-388.
Spivak JM, Connolly PJ (eds): Orthopaedic Knowledge Update: Spine 3. Rosemont, IL,
American Academy of Orthopaedic Surgeons, 2006, pp 235-247.
4. A 44-year-old man reports persistent left leg pain following a L5-S1 hemilaminotomy
and partial diskectomy. Examination shows a grade 4 weakness of the left extensor
hallucis longus and a positive left straight leg raise. A radiograph is shown in Figure 1a,
and sagittal and axial MRI scans are shown in Figures 1b and 1c. Nonsurgical
management consisting of medication, physical therapy, and injections has failed to
provide relief. Surgical management should consist of

1- revision L5-S1 hemilaminotomy.


2- L5-S1 total disk arthroplasty.
3- L5 Gill laminectomy.
4- posterior foraminal decompression and fusion at L5-S1 with instrumentation and
bone graft.
5- stand-alone posterior lumbar interbody fusion.

PREFERRED RESPONSE: 4

DISCUSSION: The patient has a grade I isthmic spondylolisthesis at L5-S1. He has an L5


radiculopathy with foraminal stenosis. Any further treatment needs to include an arthrodesis and
foraminal decompression. Isolated interbody fusion is contraindicated in patients with
spondylolisthesis, as is total disk arthroplasty. Therefore, the best procedure is a posterior fusion
with instrumentation and bone graft along with a foraminal decompression.

REFERENCES: Spivak JM, Connolly PJ (eds): Orthopaedic Knowledge Update: Spine 3.


Rosemont, IL, American Academy of Orthopaedic Surgeons, 2006, pp 311-317.
Moller H, Hedlund R: Instrumented and noninstrumented posterolateral fusion in adult
spondylolisthesis: A prospective randomized study: Part 2. Spine 2000;25:1716-1721.
5. Bisphosphonates are indicated in the treatment of osteoporosis in patients who have a
DEXA T-score of

1- between 0 and 1.
2- between 0 and -1.
3- -3.5 and are already on teriparatide.
4- within one standard deviation from the mean.
5- less than -1.

PREFERRED RESPONSE: 5

DISCUSSION: Bisphosphonates are indicated in the treatment of osteoporosis. They have been
shown to reduce the incidence of vertebral and extremity fractures in patients with a T-score of
less than -1.

REFERENCE: Gass M, Dawson-Hughs B: Preventing osteoporosis-related fractures: An


overview. Am J Med 2006;119:S3-S11.
6. A 45-year-old man reports that he awoke 2 weeks ago with severe pain in his right arm.
Examination reveals weakness in the biceps, brachialis, and wrist extensors. There is
decreased sensation in the thumb and index finger and a diminished brachioradialis
reflex. Assuming this patient has a posterolateral herniated nucleus pulposus, what level
is involved?

1- C2-3
2- C3-4
3- C4-5
4- C5-6
5- C6-7

PREFERRED RESPONSE: 4

DISCUSSION: This is a classic C6 nerve injury, and it is most likely the result of a herniated
nucleus pulposus at C5-6. The C5 nerve root controls the elbow flexors, shoulder abductors, and
external rotators. The C7 nerve root controls the elbow extensors, wrist pronators, and the
triceps reflex.

REFERENCES: Standaert CJ: The patient history and physical examination: Cervical, thoracic
and lumbar, in Herkowitz HN, Garfin SR, Eismont FJ, et al (eds): Rothman-Simeone The Spine,
ed 5. Philadelphia, PA, Saunders Elsevier, 2006, vol 1, pp 171-186.
Bates B: A Guide to Physical Examination and History Taking, ed 5. Philadelphia, PA,
JB Lippincott, 1991.
7. A 42-year-old woman underwent an instrumented posterior spinal fusion at L3-S1 with
transforaminal lumbar interbody fusion. She had an excellent clinical result with
complete resolution of leg pain. Three months later she now reports increasing back pain
and weakness in her legs. Examination reveals weakness in the quadriceps and tibialis
anterior. Radiographs show no interval changes in the position of the hardware. MRI
scans are shown in Figures 2a through 2c. What is the next most appropriate step in
management?

1- Observation
2- Oral antibiotics only
3- IV antibiotics only
4- Irrigation and debridement of the surgical site
5- Irrigation and debridement of the surgical site with hardware removal

PREFERRED RESPONSE: 4

DISCUSSION: The MRI scans reveal a postoperative infection. Observation and antibiotics are
not appropriate choices. There is a large fluid collection and this requires decompression
because the patient has neurologic changes. There is considerable debate regarding the removal
of hardware. Many contend that biofilm on the implants can harbor the infection. However,
these complications usually can be treated with serial irrigations, debridements, and IV
antibiotics. The incidence of infection has been widely studied with varying rates in fusions with
instrumentation. Rates appear to be increased with instrumentation, yet these infections usually
can be managed without hardware removal.

REFERENCES: Glassman SD, Dimar JR, Puno RM, et al: Salvage of instrumental lumbar
fusions complicated by surgical wound infection. Spine 1996;21:2163-2169.
Fang A, Hu SS, Endres N, et al: Risk factors for infection after spinal surgery. Spine
2005;30:1460-1465.
8. What is the primary reason for including the ilium in the distal fixation of long
instrumentation constructs in adult scoliosis?

1- Better coronal balance


2- Better pelvic balance
3- Reduced fretting and corrosion
4- Improved curve correction
5- Improved fusion success

PREFERRED RESPONSE: 5

DISCUSSION: Studies have shown that when compared with fixation to the sacrum alone, the
success rate of fusion across the lumbosacral junction increases when both the sacrum and ilium
are included in the posterolateral construct. Curve correction, coronal balance, and pelvic
balance are all attended to within the thoracolumbar spine and are not directly related to the
pelvic fixation. Fretting and corrosion are a byproduct of metal-to-metal connections.

REFERENCES: Islam NC, Wood KB, Transfeldt EE, et al: Extension of fusions to the pelvis in
idiopathic scoliosis. Spine 2001;26:166-173.
Emami A, Deviren V, Berven S, et al: Outcome and complications of long fusions to the sacrum
in adult spine deformity: Luque-Galveston, combined iliac and sacral screws, and sacral fixation.
Spine 2002;27:776-786.
9. A 60-year-old man is evaluated in the ICU after a rollover motor vehicle accident 3 days
ago. He has multiple upper and lower extremity trauma and was found unresponsive at
the accident scene. Surgery is planned for the extremity trauma once the patient is
medically stable. He remains intubated and the cervical spine is immobilized in a semi-
rigid collar. Examination reveals mild erythema in the posterior occipital cervical region.
Initial AP and lateral radiographs of the cervical spine have not revealed any obvious
fracture. What is the most appropriate treatment option at this time?

1- Continued semi-rigid immobilization until the extremity surgeries are completed


2- Halo skeletal fixation prior to the extremity surgery
3- Definitive clearance of the cervical spine with CT and/or MRI
4- Removal of the semi-rigid collar and physical examination when the patient is
responsive
5- Soft collar immobilization and local wound care

PREFERRED RESPONSE: 3

DISCUSSION: Ackland and associates demonstrated that the failure to achieve early spinal
clearance in an unconscious blunt trauma patient predisposed the patient to increased morbidity
secondary to the prolonged used of cervical immobilization. They demonstrated that the four
significant predictors of collar-related ulcers were ICU admission, mechanical ventilation, the
necessity for cervical MRI, and the time to cervical spine clearance and collar removal. The risk
of pressure-related ulceration increased by 66% for every 1-day increase in Philadelphia collar
time and this highlights the need for definitive C-spine clearance.

REFERENCES: Ackland HM, Cooper DJ, Malham GM, et al: Factors predicting cervical collar-
related decubitus ulceration in major trauma patients. Spine 2007;32:423-428.
Hewitt S: Skin necrosis caused by semi-rigid cervical collar in a ventilated patient with multiple
injuries. Injury 1994;25:323-324.
10. A 46-year-old woman who was involved in a motor vehicle accident reports a 4-month
history of right-sided lower back pain and pain radiating into the right thigh. The patient
underwent an extensive 3-month course of physical therapy and now is dependent on
narcotic medication for pain control. Epidural injection therapy has failed to improve her
symptoms. Examination is significant for weakness of hip flexion in the seated position
and for decreased sensation to light touch in the medial anterior thigh region. Straight leg
raise is negative, but the femoral stretch test reproduces anterior thigh pain. A CT
myelogram image, at L3-L4, is shown in Figure 3. What is the most appropriate
management at this time?

1- Repeat epidural steroid injections


2- Wide lumbar laminectomy
3- Microdiskectomy from either a midline approach or far lateral approach
4- Referral to pain management
5- Minimally invasive posterior lumbar interbody fusion

PREFERRED RESPONSE: 3

DISCUSSION: The CT scan reveals a right-sided lateral disk protrusion at L3-4 that has been
symptomatic for more than 4 months despite appropriate nonsurgical management. Relative
surgical indications include persistent radiculopathy despite an adequate trial of nonsurgical
management, recurrent episodes of sciatica, persistent motor deficit with tension signs and pain,
and pseudoclaudication caused by underlying stenosis. Whereas studies have shown
improvement in patients with sciatica from a lumbar disk herniation treated either nonsurgically
or surgically, those undergoing surgical treatment had an overall greater improvement of
symptoms.

REFERENCES: Weinstein JN, Lurie JD, Tosteson TD, et al: Surgical vs nonoperative treatment
for lumbar disk herniation: The Spine Patient Outcomes Research Trial (SPORT) observational
cohort. JAMA 2006;296:2451-2459.
Yorimitsu E, Chiba K, Toyama Y, et al: Long-term outcomes of standard discectomy for lumbar
disc herniation: A follow-up study of more than 10 years. Spine 2001;26:652-657.
11. A 73-year-old woman reports a 4-month history of severe left-sided posterior buttock
pain and left leg pain. The leg pain radiates into the left lateral thigh and posterior calf
with cramping. Examination reveals mild difficulty with a single-leg toe raise on the left
side and a diminished ankle reflex. There is also a significant straight leg raise test at 45
degrees which exacerbates symptoms. An MRI scan is shown in Figure 4. What is the
most appropriate treatment at this time?

1- Lumbar laminectomy with synovial cyst excision


2- Repeat epidural steroid injection
3- Microdiskectomy at L4-5
4- Nonsteroidal medication and outpatient physical therapy
5- Left-sided facet blocks at L4-5 and L5-S1

PREFERRED RESPONSE: 1

DISCUSSION: Lumbar spinal stenosis with lumbar radiculopathy can be commonly caused by a
synovial cyst arising from the facet joints. Lyons and associates reported on the surgical
treatment of synovial cysts in 194 patients. Of the 147 with follow-up data, 91% reported good
pain relief and 82% had improvement of their motor deficits. Epstein reported a 58% to 63%
incidence of good/excellent results and a 38 to 42 point improvement on the SF-36 Physical
Function Scale. It was also suggested that since the presence of a synovial cyst indicates facet
pathology, possible fusion should be considered in these patients, especially those with
underlying spondylolisthesis.

REFERENCES: Lyons MK, Atkinson JL, Wharen RE, et al: Surgical evaluation and
management of lumbar synovial cysts: The Mayo Clinic Experience. J Neurosurg
2000;93:53-57.
Khan AM, Synnot K, Cammisa FP, et al: Lumbar synovial cysts of the spine: An evaluation of
surgical outcome. J Spinal Disord Tech 2005;18:127-131.
Epstein NE: Lumbar laminectomy for the resection of synovial cysts and coexisting lumbar
spinal stenosis or degenerative spondylolisthesis: An outcome study. Spine 2004;29:1049-1055.
12. Osteoporotic vertebral compression fractures are associated with

1- neurologic deterioration in 33% of patients.


2- osteomalacia in 50% of patients.
3- a further fracture risk rate of 20%.
4- chronic pain in 75% of patients.
5- a 2-year mortality rate that is less than that associated with hip fractures.

PREFERRED RESPONSE: 3

DISCUSSION: Osteoporotic vertebral compression fractures are associated with neurologic


complications in less than 1% of patients. After the initial fracture however, patients have a 20%
risk of further fractures. The mortality rate of patients with vertebral fractures exceeds that of
patients with hip fractures when they are followed beyond 6 months.

REFERENCES: Gass M, Dawson-Hughs B: Preventing osteoporosis-related fractures: An


overview. Am J Med 2006;119:S3-S11.
Lindsay R, Silverman SL, Cooper C, et al: Risk of new vertebral fracture in the year following a
fracture. JAMA 2001;285:320-323.
Kado DM, Duong T, Stone KL, et al: Incident vertebral fractures and mortality in older women:
A prospective study. Osteoporos Int 2003;14:589-594.
13. When compared to smokers who do not quit, an improvement in the rate of lumbar fusion
is seen in patients who cease smoking for at least how many months postoperatively?

1- 1 month
2- 2 months
3- 4 months
4- 6 months
5- 12 months

PREFERRED RESPONSE: 4

DISCUSSION: The effects of cigarette smoking and smoking cessation on spinal fusion have
been studied extensively. Although permanent smoking cessation is ideal, significant
improvements in fusion rates are seen in patients who avoid smoking for greater than 6 months
postoperatively.

REFERENCE: Glassman SD, Anagnost SC, Parker A, et al: The effect of cigarette smoking and
smoking cessation on spinal fusion. Spine 2000;25:2608-2615.
14. A 19-year-old woman reports persistent neck pain for 2 years. Pain is relieved with
aspirin. A bone scan shows intense uptake in the superior, posterior portion of the C3
vertebral body. A sagittal CT reconstruction is shown in Figure 5. Treatment should
consist of

1- radiation therapy.
2- en bloc excision.
3- posterior fusion at C2-C3 with instrumentation.
4- CT-guided aspiration followed by IV antibiotics.
5- radiofrequency ablation.

PREFERRED RESPONSE: 2

DISCUSSION: The CT scan shows an osteoblastic nidus pathognomic for an osteoid osteoma.
Surgical treatment should include an en bloc excision of the lesion. Surgical treatment is not
mandatory because the lesion often becomes asymptomatic over time. This lesion is not
amenable to radiofrequency ablation due to its proximity to the spinal cord. A complete
corpectomy is not necessary to adequately resect the lesion, as only the nidus needs to be
removed. Radiation therapy and antibiotics are not appropriate treatments for an osteoid
osteoma. Posterior C2-C3 fusion will not address the pathology.

REFERENCES: Spivak JM, Connolly PJ (eds): Orthopaedic Knowledge Update: Spine 3.


Rosemont, IL, American Academy of Orthopaedic Surgeons, 2006, pp 351-366.
Hadjipavlou AG, Lander PH, Marchesi D, et al: Minimally invasive surgery for ablation of
osteoid osteoma of the spine. Spine 2003;28:E472-E477.
15. A 56-year-old man with a history of chronic lower back pain from lumbar spondylosis
reports a 2-day history of acute incapacitating back pain. He denies any history of acute
trauma, although he reports the pain starting after a coughing spell. He also reports
difficulty urinating and some fecal incontinence. Examination reveals generalized lower
extremity weakness, saddle paresthesia, hyporeflexia in the lower extremities, and loss of
rectal tone. What is the most appropriate management at this time?

1- Immediate MRI of the lumbar spine


2- General reassurance, anti-inflammatory drugs, and an early home exercise
program
3- Immediate radiographs of the lumbar spine and pain medications with 2 days of
bed rest if the radiographs are normal
4- Office caudal epidural steroid injection with follow-up in 1 week
5- Outpatient MRI of the lumbar spine with follow-up in 1 week for test results

PREFERRED RESPONSE: 1

DISCUSSION: Cauda equina syndrome is a medical emergency that must be quickly diagnosed
and treated to avoid long-term complications. Cauda equina syndrome typically presents with
low back pain, unilateral or usually bilateral sciatica, saddle sensory disturbances, bladder and
bowel dysfunction, and variable lower extremity motor and sensory loss. Although a number of
pathologies can cause cauda equina syndrome, in a patient with a history of chronic back pain,
disk pathology is the most common cause of acute onset cauda equina syndrome. Whereas
radiographs may be useful in a traumatic onset of symptoms, MRI is the most appropriate study.
Cauda equina syndrome should be evaluated on an emergent basis and admission for work-up is
appropriate.

REFERENCES: Ahn UM, Ahn NU, Buchowski JM, et al: Cauda equina syndrome secondary to
lumbar disc herniation: A meta-analysis of surgical outcomes. Spine 2000;25:1515-1522.
Small SA, Perron AD, Brady WJ: Orthopedic pitfalls: Cauda equina syndrome. Am J Emerg
Med 2005;23:159-163.
16. A 55-year-old woman with a long history of low back and left lower extremity pain has
failed to respond to exhaustive nonsurgical management. MRI scans show bulging and
degeneration at L3-4 and L4-5 as well as a normal disk at L2-3 and L5-S1. She
undergoes provocative lumbar diskography at L3-4, L4-5, and L5-S1. Post-diskography
axial CT images of L3-4 and L4-5 are shown in Figures 6a and 6b, respectively. The
injections at L3-4 and L4-5 produce no pain. The injection at L5-S1 produces 10/10
concordant back pain with radiation to the lower extremity. What is the most appropriate
recommendation at this time?

1- Consider fusion surgery


2- Intradiskal ozone therapy
3- Lumbar laminectomy
4- Vertebral augmentation
5- Cognitive intervention and exercise

PREFERRED RESPONSE: 5

DISCUSSION: The results of this patients lumbar diskography are equivocal at best. The two
disks most likely to be her pain generators, based on their MRI appearance, produced 10/10 pain,
however it was nonconcordant and did not reproduce any of her typical left-sided radicular
symptoms. The only disk that produced concordant back pain was the normal disk at the L5-S1
level and it reproduced radicular symptoms on the side opposite of her typical pain. Based on
these findings, it would be difficult to select a level or levels to include in a lumbar fusion. As
such, continued nonsurgical management is the safest treatment option at the current time. Brox
and associates reported on a randomized clinical trial comparing lumbar fusion to cognitive
intervention and exercise and found similar results in both groups, with significantly less risk in
the latter.

REFERENCES: Brox JI, Sorensen R, Friis A, et al: Randomized clinical trial of lumbar
instrumented fusion and cognitive intervention and exercises in patients with chronic low back
pain and disc degeneration. Spine 2003;28:1913-1921.
Carragee EJ: Clinical practice: Persistent low back pain. N Engl J Med 2005;352:1891-1898.
17. A 36-year-old woman is brought to the emergency department intubated and sedated
following a motor vehicle accident. She is moving her upper and lower extremities
spontaneously. She cannot follow commands. CT scans are shown in Figures 7a through
7c. The initial survey does not reveal any other injuries. Initial management of the
cervical injury should consist of immediate

1- immobilization with a halo ring and vest with reduction when medically stable.
2- closed traction reduction using Gardner-Wells tongs.
3- posterior open reduction, stabilization, and fusion.
4- cervical MRI followed by reduction.
5- anterior open reduction, stabilization, and fusion.

PREFERRED RESPONSE: 4

DISCUSSION: The patient has a bilateral facet dislocation of C6-C7 with preservation of at least
some neurologic function. Urgent reduction is necessary. However, because she is sedated and
unable to follow commands, an MRI scan is necessary before any closed or open posterior
reduction to look for an associated disk herniation. If a disk herniation is present, it must be
removed prior to any reduction maneuver to prevent iatrogenic neurologic injury. It is very
unlikely that this injury can be reduced with an open anterior procedure alone.

REFERENCES: Spivak JM, Connolly PJ (eds): Orthopaedic Knowledge Update: Spine 3.


Rosemont, IL, American Academy of Orthopaedic Surgeons, 2006, pp 189-199.
Kwon BK, Vaccaro AR, Grauer JN, et al: Subaxial cervical spine trauma. J Am Acad Orthop
Surg 2006;14:78-89.
18. A 51-year-old woman with no preoperative neurologic deficit is undergoing elective
anterior cervical diskectomy and fusion (ACDF) with plating and fusion for a C5-6 disk
herniation with right-sided neck pain. Thirty minutes into the surgery the
neurophysiologic monitoring shows a rapid drop and then loss of amplitude in the right
cortical somatosensory-evoked potential waveform. All other waveforms remained
normal and unchanged, including right-sided cervical (subcortical) and peripheral (Erbs
point), and those from the left-sided upper extremity and both lower extremities. What is
the most likely cause of the change?

1- Electrode placement
2- Stimulation failure
3- Anesthetic effect
4- Cord ischemia from retraction
5- Cerebral ischemia from retraction

PREFERRED RESPONSE: 5

DISCUSSION: The change noted is focal and confined to the cortex, sparing the opposite side,
both lower extremities, and the subcortical waveforms, making all the choices unlikely with the
exception of carotid compression with focal cortical ischemia. This may be associated with poor
collateral flow from the opposite hemisphere due to an incomplete circle of Willis.

REFERENCES: Drummond JC, Englander RN, Gallo CJ: Cerebral ischemia as an apparent
complication of anterior cervical discectomy in a patient with an incomplete circle of Willis.
Anesth Analg 2006;102:896-899.
Yeh YC, Sun WZ, Lin CP, et al: Prolonged retraction on the normal common carotid artery
induced lethal stroke after cervical spine surgery. Spine 2004;29:E431-E434.
19. A 68-year-old woman undergoes a complicated four-level anterior cervical diskectomy
and fusion at C3-7 with iliac crest bone graft and instrumentation for multilevel cervical
stenosis. Surgical time was approximately 6 hours and estimated blood loss was 800 mL.
Neuromonitoring was stable throughout the procedure. The patients history is
significant for smoking. The most immediate appropriate postoperative management for
this patient should include

1- normal postoperative orders with frequent neurologic evaluations for the first 24
hours.
2- administration of IV steroids and placement of a soft cervical collar for 24 hours.
3- placement of both deep and superficial surgical drains prior to wound closure.
4- administration of IV mannitol and placement of a soft collar.
5- maintaining intubation for up to 24 to 48 hours.

PREFERRED RESPONSE: 5

DISCUSSION: Airway complications after anterior cervical surgery can be a catastrophic event
necessitating emergent intubation for airway protection. Multilevel surgeries requiring long
intubation and prolonged soft-tissue retraction as well as preexisting comorbidities may
predispose a patient to postoperative airway complications. Sagi and associates reported that
surgical times greater than 5 hours, blood loss greater than 300 mL, and multilevel surgery at or
above C3-4 are risk factors for airway complications. In surgical procedures with the
aforementioned factors, serious consideration should be given to elective intubation for 1 to 3
days to avoid urgent reintubation.

REFERENCES: Sagi HC, Beutler W, Carroll E, et al: Airway complications associated with
surgery on the anterior cervical spine. Spine 2002;27:949-953.
Epstein NE, Hollingsworth R, Nardi D, et al: Can airway complications following multilevel
anterior cervical surgery be avoided? J Neurosurg 2001;94:185-188.
Emery SE, Smith MD, Bohlman HH: Upper-airway obstruction after multi-level cervical
corpectomy for myelopathy. J Bone Joint Surg Am 1991;73:544-551.
20. A 22-year-old woman reports a 4-year history of worsening low back and left lower
extremity pain following a motor vehicle accident. Management consisting of physical
therapy, chiropractic manipulation, and interventional pain management, including
sacroiliac joint injections and epidural steroid injections, has failed to provide relief. A
sagittal T 2 -weighted MRI scan is shown in Figure 8. No nerve root compression is seen
on axial images. She is currently working and lives with her fianc. She smokes half a
pack of cigarettes per day and reports depression on her health history. She is being
maintained on narcotic analgesics and is having increasing difficulty performing her
activities of daily living secondary to pain. What is the most appropriate management at
this time?

1- Provocative lumbar diskography


2- Laboratory studies, including a complete blood cell (CBC) count, erythrocyte
sedimentation rate (ESR), and urinalysis
3- Cognitive intervention, exercise, and smoking cessation
4- Bilateral lower extremity electromyography and nerve conduction velocity studies
5- Lumbar myelogram with a postmyelography CT scan of the lumbar spine

PREFERRED RESPONSE: 3

DISCUSSION: The MRI scan reveals a rudimentary disk at the L5-S1 level, suggesting
transitional anatomy. There is a posterior disk bulge at L3-4. At L4-5, there is disk desiccation
and loss of disk height, with a posterior disk bulge and a high intensity zone in the posterior
annulus, suggesting an annular tear. While these and similar radiographic findings have been
associated with the severity of a patients pain, they are also commonly found in cross-sectional
studies of asymptomatic subjects. Carragee and associates found 59% of symptomatic patients
undergoing diskography have high intensity zones as compared to 25% of asymptomatic subjects
of a similar patient profile. Diskographic injections provoked pain in disks with high intensity
zones approximately 70% of the time whether the individual was previously symptomatic or not.
This patients non-specific pain pattern does not require further work-up as she is not a surgical
candidate.

REFERENCES: Carragee EJ, Paragioudakis SJ, Khurana S: 2000 Volvo Award winner in
clinical studies: Lumbar high-intensity zone and discography in subjects without low back
problems. Spine 2000;25:2987-2992.
Pneumaticos SG, Reitman CA, Lindsey RW: Diskography in the evaluation of low back pain.
J Am Acad Orthop Surg 2006;14:46-55.
Brox JI, Sorensen R, Friis A, et al: Randomized clinical trial of lumbar instrumented fusion and
cognitive intervention and exercises in patients with chronic low back pain and disc
degeneration. Spine 2003;28:1913-1921.
Carragee EJ: Clinical practice: Persistent low back pain. N Engl J Med 2005;352:1891-1898.
21. A 42-year-old man with a history of renal cell carcinoma has progressive weakness in the
lower extremities for the past 3 weeks. The patient desires intervention. A sagittal T 2 -
weighted MRI scan is shown in Figure 9a, and a sagittal contrast enhanced T 1 -weighted
MRI scan is shown in Figure 9b. He currently ambulates minimal distances with a
walker. His life expectancy is 8 months. Treatment of the spine lesion should consist of

1- radiation therapy.
2- posterior laminectomy.
3- anterior corpectomy and reconstruction.
4- posterior laminectomy and fusion.
5- kyphoplasty.

PREFERRED RESPONSE: 3

DISCUSSION: The MRI scans show a metastatic lesion in two contiguous vertebral bodies in
the lower thoracic spine. Posterior laminectomy is not indicated because this does not
adequately decompress the neural elements and will lead to progressive kyphosis. A posterior
fusion may prevent progressive kyphosis but will not decompress the spinal cord. Renal cell
carcinoma is not radiosensitive; therefore, radiation therapy would not be helpful in relieving
neurologic compression. The lesion should be treated by an anterior corpectomy and
reconstruction. This will allow for complete decompression as well as reconstruction of the
anterior column. Kyphoplasty is not indicated in a lesion with disruption of the posterior cortex
and neurologic impairment.

REFERENCES: Spivak JM, Connolly PJ (eds): Orthopaedic Knowledge Update: Spine 3.


Rosemont, IL, American Academy of Orthopaedic Surgeons, 2006, pp 351-366.
White AP, Kwon BK, Lindskog DM, et al: Metastatic disease of the spine. J Am Acad Orthop
Surg 2006;14:587-598.
22. A 40-year-old man has intractable pain following 2 years of nonsurgical management for
high-grade spondylolisthesis. What is the best surgical option?

1- Posterolateral fusion
2- Posterolateral fusion with instrumentation
3- Circumferential fusion
4- Transforaminal lumbar interbody fusion
5- Anterior lumbar interbody fusion

PREFERRED RESPONSE: 3

DISCUSSION: Circumferential fusion is the preferred choice for patients undergoing revision
surgery following failed posterolateral fusions for isthmic spondylolisthesis as well as for those
patients having primary surgery for high-grade isthmic spondylolisthesis.

REFERENCE: Spivak JM, Connolly PJ (eds): Orthopaedic Knowledge Update: Spine 3.


Rosemont, IL, American Academy of Orthopaedic Surgeons, 2006, pp 311-317.
23. An adult patient with a grade I isthmic spondylolisthesis at L5-S1 is most likely to have
weakness of the

1- anterior tibialis.
2- quadriceps.
3- gastrocsoleus.
4- extensor hallucis longus.
5- iliopsoas.

PREFERRED RESPONSE: 4

DISCUSSION: Adult patients with isthmic spondylolisthesis most commonly have neurologic
symptoms due to foraminal stenosis at the level of the spondylolisthesis. In this scenario, the
patient is most likely to have weakness of the L5 myotome, which would cause weakness of the
extensor hallucis longus.

REFERENCES: Spivak JM, Connolly PJ (eds): Orthopaedic Knowledge Update: Spine 3.


Rosemont, IL, American Academy of Orthopaedic Surgeons, 2006, pp 311-317.
Lauerman WC, Cain JE: Isthmic spondylolisthesis in the adult. J Am Acad Orthop Surg
1996;4:201-208.
24. When performing a long fusion to the sacrum in an osteopenic patient in whom optimal
sagittal balance is restored, which of the following is a benefit of extending the distal
fixation to the pelvis, rather than the sacrum alone?

1- Decreased risk of sacral fractures


2- Decreased risk of proximal functional kyphosis
3- Easier contouring of the instrumentation
4- Reduced risk of late pubic ramus fractures
5- Improved coronal plane correction

PREFERRED RESPONSE: 1

DISCUSSION: In osteopenic individuals, even those with excellent obtained or maintained


balance, long instrumented fusions to the sacrum impart a high degree of strain, and the sacrum
may fail in a transverse fracture or fracture-dislocation pattern. The risk of proximal functional
kyphosis is unrelated to distal fixation as are coronal plane correction and rod contouring. Pubic
ramus fractures have been shown to be associated with both fixation to the sacrum alone as well
as to the ilium.

REFERENCE: Hu SS, Berven SH, Bradford DS: Adult spinal deformity, in Frymoyer JW,
Wiesel SW (eds): The Adult and Pediatric Spine, ed 3. Philadelphia, PA, Lippincott Williams
and Wilkins, 2004, pp 465-477.
25. Which of the following statements describing chordomas is false?

1- Treatment consisting of complete surgical resection with clean margins offers the
best survival.
2- They occur in the clivus and sacrum and occur only 15% of the time in the rest of
the spine.
3- They are locally aggressive and invasive tumors.
4- They are highly radiosensitive.
5- They have the ability to become malignant.

PREFERRED RESPONSE: 4

DISCUSSION: Casali and associates provided a recent review of the treatment options for
chordomas. These tumors are not radiosensitive; however, modern intensity modulated
radiosurgery techniques may be of value. The combination of surgery and radiotherapy
compared to surgery alone results in the same disease-free survival time. Complete surgical
resection of the chondroma with clean margins offers the best survival; however, its location
may make total removal impossible. Thus subtotal resection followed by radiotherapy results in
better survival despite the tumors lack of radiosensitivity.

REFERENCE: Casali PG, Stacchiotti S, Sangalli C, et al: Chordoma. Curr Opin Oncol
2007;19:367-370.
26. A previously healthy 35-year-old man was involved in a rollover motor vehicle accident
2 days ago. He was placed in a semi-rigid cervical orthosis. He now reports mostly axial
neck pain with attempted range of motion. Examination reveals the mechanical neck
pain but no obvious neurologic deficits. AP, flexion, and extension radiographs are
shown in Figures 10a through 10c, and sagittal and coronal CT scans are shown in
Figures 10d and 10e. What is the most appropriate management at this time?

1- Continued immobilization in a semi-rigid cervical orthosis for 6 to 8 weeks


2- Posterior occipital-cervical fusion with iliac crest bone graft
3- Open reduction and internal fixation of the odontoid process with an anterior
odontoid screw
4- Resection of the odontoid process through a transoral approach
5- Reduction with Gardner-Wells tong traction and 6 weeks of skeletal traction

PREFERRED RESPONSE: 3

DISCUSSION: Odontoid fractures can be classified based on the anatomic position of the
fracture within the dens itself. Type I is an oblique fracture through the upper part of the
odontoid process. Type II is a fracture that occurs at the base of the odontoid as it attaches to the
body of C2; type III occurs when the fracture line extends through the body of the axis. Type 1
fractures typically can be treated nonsurgically with 6 to 8 weeks of immobilization with a semi-
rigid cervical orthosis. Nondisplaced, deep type III fractures generally are treated with skeletal
halo fixation. Deep, displaced, and angled type III fractures can be treated with closed reduction
and skeletal halo fixation. Shallow type III fractures are sometimes amenable to anterior
odontoid screw fixation. Type II fractures can be managed nonsurgically or surgically.
Treatment options include halo immobilization, internal fixation (odontoid screw fixation), and
posterior atlantoaxial arthrodesis. Management with the halo vest usually is considered if the
initial dens displacement is less than 6 mm, the reduction is performed within 1 week of the
injury and is able to be maintained, and the patient is younger than age 60 years. Halo vest
immobilization can lead to a healing rate of more than 90%. Posterior surgical fusion techniques
provide high fusion success rates but do so at the expense of cervical rotation. Up to 50% of
rotation is lost with these techniques. Anterior odontoid single screw fixation is often tolerated
better than skeletal halo fixation and also is noted to preserve the normal rotation at C1/C2.
Studies have shown less of a malunion and nonunion rate in the treatment of type II odontoid
fractures with anterior odontoid screw fixation. Osteoporosis, short neck and barrel-chested
anatomy, and fractures that are more than 4 weeks old preclude anterior odontoid fixation.

REFERENCES: Shilpakar S, McLaughlin MR, Haid RW Jr, et al: Management of acute


odontoid fractures: Operative techniques and complication avoidance. Neurosurg Focus
2000;8:e3.
Subach BR, Morone MA, Haid RW Jr, et al: Management of acute odontoid fractures with
single-screw anterior fixation. Neurosurgery 1999;45:812-819.
Fountas KN, Kapsalaki EZ, Karampelas I, et al: Results of long-term follow-up in patients
undergoing anterior screw fixation for type II and rostral type III odontoid fractures. Spine
2005;30:661-669.
27. Which of the following palpable bony landmarks is correctly matched with its
corresponding vertebral level?

1- Angle of the mandible and the C2-C3 interspace


2- Hyoid bone and C6
3- Carotid tubercle and C6
4- Superior portion of the thyroid cartilage and the C3 vertebral body
5- Cricoid cartilage and C7-T 1

PREFERRED RESPONSE: 3

DISCUSSION: The carotid tubercle is usually located at the level of C6. The angle of the
mandible is at C1-C2; the hyoid is at C4; the superior portion of the thyroid cartilage is C4-C5;
and the cricoid cartilage is at C6.

REFERENCES: Smith GW, Robinson RA: The treatment of certain cervical-spine disorders by
anterior removal of the intervertebral disc and interbody fusion. J Bone Joint Surg Am
1958;40:607.
An HS: Surgical Exposure and Fusion Techniques of the Spine: Principles and Techniques of
Spine Surgery. Baltimore, MD, Williams and William, 1998, pp 31-62.
28. What root is most commonly involved with a segmental root level palsy after
laminoplasty?

1- C3
2- C4
3- C5
4- C6
5- C7

PREFERRED RESPONSE: 3

DISCUSSION: The postoperative incidence of C5 root palsy after laminoplasty ranges from 5%
to 12%. Other roots also may be affected. The palsies tend to be motor dominant, although
sensory dysfunction and radicular pain are also possible. The palsy may arise during the
immediate postoperative period or up to 20 days later. C5 may be preferentially involved
because it is at the apex of the cervical lordosis. Recovery usually occurs over weeks to months.

REFERENCES: Spivak JM, Connolly PJ (eds): Orthopaedic Knowledge Update: Spine 3.


Rosemont, IL, American Academy of Orthopaedic Surgeons, 2006, pp 235-249.
Uematsu Y, Tokuhashi Y, Matsuzaki H: Radiculopathy after laminoplasty of the cervical spine.
Spine 1998;23:2057-2062.
29. Up to what time frame are the risks minimized in anterior revision disk replacement
surgery?

1- 3 days
2- 1 week
3- 10 days
4- 2 weeks
5- 6 weeks

PREFERRED RESPONSE: 4

DISCUSSION: Revision anterior exposure within 2 weeks of total disk replacement incurs
relatively little additional morbidity because adhesion formation is minimal. Surgeons should
have a low threshold for revising implants that are clearly dangerously malpositioned or show
early migration within this 2-week window. Beyond this time period, a revision strategy must be
individualized to the particular clinical situation. A posterior fusion with instrumentation with or
without a laminectomy is currently the most effective salvage procedure.

REFERENCE: Tortolani JP, McAfee PC, Saiedy S: Failures of lumbar disc replacement. Sem
Spine Surg 2006;18:78-86.
30. Which of the following best describes the use of epidural morphine and steroid paste after
laminectomy?

1- Associated with an 11% rate of postoperative surgical site complications


2- Associated with a less than 1% rate of surgical site infections
3- Associated with a decreased rate of postoperative urinary retention
4- Considered the standard for outpatient microdiskectomy
5- Should only be used in the absence of radiculopathy

PREFERRED RESPONSE: 1

DISCUSSION: Kramer and associates conducted a retrospective review during an epidemic


period to identify the risk factors associated with a sudden increase in the rate of surgical site
infections. They found in a multivariate analysis that the use of morphine nerve paste resulted in
a 7.6-fold increase in postoperative surgical wound debridement, and an 11% rate of surgical site
complications.

REFERENCES: Kramer MH, Mangram AJ, Pearson ML, et al: Surgical-site complications
associated with a morphine nerve paste used for postoperative pain control after laminectomy.
Infect Control Hosp Epidemiol 1999;20:183-186.
Lowell TD, Errico TJ, Eskenazi MS: Use of steroids after discectomy may predispose to
infection. Spine 2000;25:516-519.
31. Figures 11a and 11b show the T 2 -weighted MRI scans of the lumbar spine of a 53-year-
old woman who has low back and right lower extremity pain. What structure is the arrow
pointing to in Figure 11a?

1- Ligamentum flavum
2- Lumbar synovial cyst
3- Tarlov cyst
4- Pseudomeningocele
5- Herniated nucleus pulposus

PREFERRED RESPONSE: 2

DISCUSSION: The arrow is pointing to a cystic-appearing structure with high signal intensity
on T 2 -weighted image sequencing. It appears to be contiguous with the hypertrophied right facet
joint, which appears to also have high signal intensity. The mass significantly narrows the right
lateral recess. The high signal intensity suggests that this is a fluid-filled mass. In addition, the
facet joints are degenerative and there is a very mild degree of anterolisthesis on the sagittal
image. These findings make a lumbar synovial cyst the most likely diagnosis. Most lumbar
juxtafacet cysts are observed at the L4-5 level, extradurally and adjacent to the degenerative
facet joint. They may contain synovial fluid and/or extruded synovium. Presentation is
indistinguishable from that of a herniated disk. The etiology of spinal cysts remains unclear, but
there appears to be a strong association between their formation and worsening spinal instability.
They occasionally regress spontaneously and may respond to aspiration and injection of
corticosteroids, though there is a high recurrence rate with nonsurgical management. Synovial
cysts resistant to nonsurgical management should be treated surgically. If the patients
symptoms can be attributable to radicular findings, a microsurgical decompression that limits
further destabilization should suffice. However, if there is significant low back pain attributable
to spinal instability, decompression and fusion remains an appropriate option.

REFERENCES: Banning CS, Thorell WE, Leibrock LG: Patient outcome after resection of
lumbar juxtafacet cysts. Spine 2001;26:969-972.
Deinsberger R, Kinn E, Ungersbock K: Microsurgical treatment of juxta facet cysts of the
lumbar spine. J Spinal Disord Tech 2006;19:155-160.
Khan AM, Synnot K, Cammisa FP, et al: Lumbar synovial cysts of the spine: An evaluation of
surgical outcome. J Spinal Disord Tech 2005;18:127-131.
32. A 38-year-old man reports a 2-week history of acute lower back pain with radiation into
the left lower extremity. There is no history of trauma and no systemic signs are noted.
Examination reveals a positive straight leg test at 35 degrees on the left side and a
contralateral straight leg raise on the right side. Motor testing demonstrates mild
weakness of the gluteus medius and weakness of the extensor hallucis longus of 3+/5.
Sensory examination demonstrates decreased sensation along the lateral aspect of the calf
and top of the foot. Knee and ankle reflexes are intact and symmetrical. Radiographs
demonstrate no obvious abnormality. MRI scans show a posterolateral disk hernation.
The diagnosis at this time is consistent with a herniated nucleus pulposus at

1- L1-2.
2- L2-3.
3- L3-4.
4- L4-5.
5- L5-S1.

PREFERRED RESPONSE: 4

DISCUSSION: The patients history and physical examination findings are consistent with a
lumbar disk herniation at the L4-5 level. Weakness of the extensor hallucis longus and gluteus
medius are consistent with an L5 lumbar radiculopathy. Nerve root tension signs are also
consistent with sciatica from a lumbar disk herniation. The MRI scans confirm a posterolateral
disk herniation at L4-5, which typically affects the exiting L5 nerve root.

REFERENCES: Hoppenfeld S: Orthopedic Neurology. Philadelphia, PA, JB Lippincott, 1977,


pp 45-74.
Spivak JM, Connolly PJ (eds): Orthopaedic Knowledge Update: Spine 3. Rosemont, IL,
American Academy of Orthopaedic Surgeons, 2006, pp 43-56.
33. A 42-year-old woman is brought to the emergency department following a motor vehicle
accident. She has sustained multiple injuries, and she is intubated and pharmacologically
paralyzed. Sagittal cervical CT scans through the right cervical facets, the left cervical
facets, and the midline are shown in Figures 12a through 12c, respectively. Definitive
management of her cervical injury should consist of

1- anterior diskectomy and fusion at C4-C5.


2- immobilization in a Philadelphia collar and voluntary flexion and extension
radiographs when awake.
3- occipital-cervical fusion with instrumentation.
4- halo immobilization for 12 weeks.
5- left C6 superior facetectomy and posterior fusion at C6-C7 with instrumentation.

PREFERRED RESPONSE: 3

DISCUSSION: The CT scans reveal an occipital-cervical dissociation with subluxation of the


occipitocervical joints bilaterally. Definitive management should consist of an occipital-cervical
fusion with instrumentation. Immobilization in a Philadelphia collar is inadequate for this highly
unstable injury, and halo immobilization, while affording adequate temporary immobilization, is
not appropriate definitive management for this ligamentous injury. The patient does not have an
injury at C4-C5 or C6-C7.

REFERENCES: Jackson RS, Banit DM, Rhyne AL III, et al: Upper cervical spine injuries.
J Am Acad Orthop Surg 2002;10:271-280.
Spivak JM, Connolly PJ (eds): Orthopaedic Knowledge Update: Spine 3. Rosemont, IL,
American Academy of Orthopaedic Surgeons, 2006, pp 201-216.
34. A 32-year-old motorcycle rider is involved in a motor vehicle accident and radiographs
show a burst fracture at L2 with 20 degrees of kyphosis. The neurologic examination is
consistent with unilateral motor and sensory involvement of the L5, S1, S2, S3, and S4
nerve roots. He has no other injuries. CT demonstrates 20% anterior canal compromise
with displaced laminar fractures at the level of injury. What is the best option for
management of this patient?

1- Bed rest for 6 weeks, followed by mobilization in a thoracolumbosacral orthosis


until the fracture has healed
2- Anterior corpectomy with strut grafting and placement of an anterior plate
spanning L1 to L3
3- Anterior corpectomy with strut grafting, followed by posterior spinal fusion and
instrumentation
4- Posterior spinal fusion and instrumentation from T11 to L4
5- L2 laminectomy and posterior spinal fusion and instrumentation from T11 to L4

PREFERRED RESPONSE: 5

DISCUSSION: The patient has a burst fracture with probable unilateral entrapment of the cauda
equina within the elements of the fractured lamina. A dural tear is likely in this scenario as well.
It is recommended that this type of burst fracture be treated surgically with laminectomy, freeing
of the entrapped nerve roots, and dural repair followed by stabilization of the fracture by either a
posterior or combined approach. The degree of kyphosis and the extent of anterior canal
compromise does not warrant corpectomy in this patient. Therefore, after completing the
laminectomy and dural repair, posterior fusion and instrumentation should be sufficient to
stabilize the fracture.

REFERENCES: Cammisa FP Jr, Eismont FJ, Green BA: Dural laceration occurring with burst
fractures and associated laminar fractures. J Bone Joint Surg Am 1989;71:1044-1052.
Spivak JM, Connolly PJ (eds): Orthopaedic Knowledge Update: Spine 3. Rosemont, IL,
American Academy of Orthopaedic Surgeons, 2006, pp 201-216.
35. A patient who underwent a L4-L5 hemilaminotomy and partial diskectomy for
radiculopathy 8 weeks ago now reports increasing low back pain without neurologic
symptoms. A sagittal T 2 -weighted MRI scan is shown in Figure 13a, and a contrast
enhanced T 1 -weighted MRI scan is shown in Figure 13b. What is the most appropriate
management for the patients symptoms?

1- Physical therapy
2- CT-guided needle biopsy and IV antibiotics
3- Revision laminotomy and diskectomy
4- L4-L5 anterior debridement and fusion
5- Open repair of the L4-L5 pseudomeningocele

PREFERRED RESPONSE: 1

DISCUSSION: The MRI scans show Modic changes in the L4-L5 vertebral bodies due to
spondylosis. There is no increased fluid signal or enhancement in the L4-L5 disk to suggest
infection or any other pathologic process. Therefore, the patients pain should be treated with a
course of physical therapy and rehabilitation. There is no infection; therefore, IV antibiotics and
debridement are not indicated. Similarly, a pseudomeningocele is not present. A revision
diskectomy is useful for recurrent radiculopathy but would not be helpful for degenerative low
back pain.

REFERENCES: Spivak JM, Connolly PJ (eds): Orthopaedic Knowledge Update: Spine 3.


Rosemont, IL, American Academy of Orthopaedic Surgeons, 2006, pp 319-329.
Shen FH, Samartzis D, Andersson GBJ: Nonsurgical management of acute and chronic low back
pain. J Am Acad Orthop Surg 2006;14:477-487.
36. What is the heaviest weight that can be safely applied to the adult cervical spine via
Gardner-Wells tong traction?

1- 40 pounds
2- 50 pounds
3- 75 pounds
4- 100 pounds
5- Greater than 100 pounds

PREFERRED RESPONSE: 5

DISCUSSION: Cotler and associates reported on the use of awake skeletal traction to reduce
facet fracture-dislocations in 24 patients. Seventeen patients required more than 50 pounds of
traction (the traditional limit) to achieve reduction. More than 100 pounds of traction was
safely used in one-third of the patients in this study. A cadaver study has supported the safe use
of traction with weights in excess of 100 pounds.

REFERENCES: Cotler JM, Herbison GJ, Nasuti JF, et al: Closed reduction of traumatic cervical
spine dislocation using traction weights up to 140 pounds. Spine 1993;18:386-390.
Anderson DG, Vacccaro AR, Gavin K: Cervical orthoses and cranioskeletal traction, in Clark
CR (ed): The Cervical Spine, ed 4. Philadelphia, PA, Lippincott Williams & Wilkins, 2005,
pp 110-121.
37. A 68-year-old man reports a 4-week history of progressive left-sided lower back and hip
pain. The pain is in the posterior buttock region with radiation to the groin and to the left
anterior knee region. The pain is aggravated with walking and improves with rest. There
is no history of previous trauma. Radiographs are seen in Figures 14a and 14b, and MRI
scans are seen in Figures 14c through 14e. What is the most appropriate treatment option
at this time?

1- Epidural steroid injection at L4-5


2- Outpatient physical therapy for the lower back
3- Non-weight-bearing of the left lower extremity
4- Home exercise program, analgesics, and limited use of muscle relaxants
5- Cortisone injection of the left greater trochanter region

PREFERRED RESPONSE: 3

DISCUSSION: Although the imaging reveals generalized lumbar spondylosis and stenosis, in
particular at L4-5, the MRI scan of the left hip clearly reveals a stress fracture of the femoral
neck. Therefore, the treatment of choice is non-weight-bearing of the left lower extremity.
During the evaluation of acute back pain, clinicians must include other possibilities within the
differential diagnosis that may mimic mechanical axial back pain; thus, potential complications
from a missed diagnosis can be avoided.

REFERENCES: Wong DA, Transfeldt E: Macnabs Backache, ed 4. Philadelphia, PA,


Lippincott Williams and Wilkins, 2007, pp 339-361.
Spivak JM, Connolly PJ (ed): Orthopaedic Knowledge Update: Spine 3. Rosemont, IL,
American Academy of Orthopaedic Surgeons, 2006, pp 43-56.
38. A 79-year-old woman reports a history of left leg pain with walking. Her pain is
exacerbated with walking and stair climbing, and her symptoms are improved by
standing after she stops walking. Lumbar flexion does not provide any significant
improvement of the symptoms and sitting does not significantly change symptoms. Her
leg pain is worse at night and she obtains relief by hanging her leg over the side of the
bed. The neurologic examination is essentially normal. Examination of the lower
extremities demonstrates mild early trophic changes, and her pulses distally are palpable
but are diminished bilaterally. Radiographs are shown in Figures 15a and 15b. What is
the next most appropriate step in management?

1- Decompression and posterior fusion at L4-L5


2- Epidural steroid injection at L4-5
3- Nonsteroidal medications and physical therapy for 6 weeks
4- Measurement of the ankle-brachial index
5- CT myelogram

PREFERRED RESPONSE: 4

DISCUSSION: The patient has symptoms that are more consistent with vascular claudication
than with the pseudoclaudication anticipated from lumbar spinal stenosis. Therefore, the patient
is a candidate for further vascular work-up. The radiographs reveal early spinal stenosis and
spondylolisthesis at L4-5 but also show significant calcification of the iliac arteries, suggestive of
peripheral vascular disease. Vascular claudication is a manifestation of peripheral vascular
disease and presents with crampy leg pain that is exacerbated by physical exertion. The pain is
easily relieved by standing still or sitting. Unlike pseudoclaudication, a forward-flexed posture
and/or sitting does not improve the symptoms. Night pain is common in vascular claudication
due to the elevation of the extremities and patients often report pain improvement by hanging
their extremities in a dependent position. In evaluation of a patient with suspected vascular
claudication, the five Ps of vascular insufficiency should be monitored, including
pulselessness, paralysis, paresthesia, pallor, and pain. While pain and paresthesias can be
common in both vascular claudication and pseudoclaudication, the presence of any of the
remaining symptoms is suggestive of vascular disease.

REFERENCES: Aufderheide TP: Peripheral arteriovascular disease, in Rosen P, Barkin R (eds):


Emergency Medicine: Concepts and Clinical Practice, ed 4. St Louis, MO, Mosby, 1998,
pp 1826-1844.
Mirkovic S, Garfin SR: Spinal stenosis: History and physical examination. Instr Course Lect
1994;43:435-440.
39. Figure 16 shows the MRI scan of a 43-year-old man who has had worsening low back
pain for the past 4 months. What is the most likely diagnosis?

1- Osteochondroma
2- Posttraumatic kyphosis
3- Staphylococcus aureus osteomyelitis
4- Ankylosing spondylitis
5- Tuberculosis

PREFERRED RESPONSE: 5

DISCUSSION: Tuberculosis of the spine is seen in 50% to 60% of skeletal disease and is most
commonly found in the lower thoracic or upper lumbar spine. Typically two or more adjacent
bodies are involved as seen in this MRI scan. The disk space is narrowed but still relatively
preserved as opposed to pyogenic infections (black arrow). Epidural extensions often spread
from vertebrae to vertebrae (white arrow); however, the posterior elements are not frequently
involved (arrowhead). Tumors rarely spread to adjacent vertebrae. The anterior and posterior
spread of the infectious process rules out trauma.

REFERENCES: Boachie-Adjei O, Squillante RG: Tuberculosis of the spine. Orthop Clin North
Am 1996;27:95-103.
Currier BL, Eismont FJ: Infections of the spine, in Rothman RH, Simeone FA (eds): The Spine.
Philadelphia, PA, WB Saunders, 1992, p 2614.
40. In patients without spondylolisthesis or scoliosis undergoing laminectomy for lumbar
spinal stenosis, spinal fusion is generally recommended if

1- a dural tear is repaired.


2- more than one level requires decompression.
3- less than one half of each facet is removed bilaterally.
4- the pars interarticularis is fractured.
5- the patient is a smoker.

PREFERRED RESPONSE: 4

DISCUSSION: With the notable exception of fusion for degenerative spondylolisthesis and
scoliosis, there is a paucity of evidence on the indications for spinal fusion in patients undergoing
laminectomy for spinal stenosis. However, it is generally recommended that if the spine is
destabilized (for example by removal of one complete facet joint or by an iatrogenic pars
fracture), spinal fusion should be considered. Although fusion can be considered for a very long
laminectomy, a two-level laminectomy does not represent, by itself, a clear indication for the
addition of a spinal fusion. The repair of a dural tear and the use of nicotine by the patient play
no role in the determination of whether or not to add fusion to a laminectomy procedure.

REFERENCES: Spivak JM, Connolly PJ (eds): Orthopaedic Knowledge Update: Spine 3.


Rosemont, IL, American Academy of Orthopaedic Surgeons, 2006, pp 299-409.
Fischgrund JS, Mackay M, Herkowitz HN, et al: 1997 Volvo Award winner in clinical studies.
Degenerative lumbar spondylolisthesis with spinal stenosis: A prospective, randomized study
comparing decompressive laminectomy and arthrodesis with and without spinal instrumentation.
Spine 1997;22:2807-2812.
41. An 18-year-old collegiate basketball player has had a 3-month history of activity-related
back pain. She describes isolated low back pain without radiation that increases with
training and playing basketball. Her pain resolves with rest. Physical therapy for 6
weeks has failed to provide relief. An axial CT scan is shown in Figure 17a, and Figures
17b and 17c show sagittal CT reconstructions through the right and left lumbar facets,
respectively. Further management should consist of which of the following?

1- CT-guided needle biopsy followed by radiation therapy


2- L5-S1 fusion with instrumentation
3- L5-S1 hemilaminotomy and partial diskectomy
4- Activity restriction and bracing
5- L5-S1 total disk arthroplasty

PREFERRED RESPONSE: 4

DISCUSSION: The sagittal and axial CT scans show a bilateral spondylolysis at L5. The defect
is in the pars interarticularis on the right side but at the base of the pedicle on the left. Having
failed a trial of physical therapy with only a 3-month history of pain, the next most appropriate
step in management should consist of activity modification and bracing in an antilordotic
lumbosacral orthosis. Surgical intervention is reserved for patients who have failed to respond to
a trial of bracing and activity restriction.

REFERENCES: Debnath UK, Freeman BJ, Grevitt MP, et al: Clinical outcome of symptomatic
unilateral stress injuries of the lumbar pars interarticularis. Spine 2007;32:995-1000.
Bono CM: Low-back pain in athletes. J Bone Joint Surg Am 2004;86:382-396.
42. Radiating pain associated with a posterolateral thoracic disk herniation typically follows
what pattern?

1- Extending down the spine into the lumbosacral region


2- Down the inner aspect of either upper extremity
3- Cephalad up to the cervicothoracic junction
4- Around or through the chest to the anterior wall
5- Down the contralateral lower extremity

PREFERRED RESPONSE: 4

DISCUSSION: Although symptomatic thoracic disk herniations can affect more caudal
structures, even to the point of paralysis, the pattern of radiating pain has been described as either
following the dermatomal band around the chest or feeling to the patient as if the pain passes
straight anteriorly to the chest wall.

REFERENCE: Skubic JW, Kostuik JP: Thoracic pain syndromes and thoracic disc herniation, in
Frymoyer JW (ed): The Adult Spine: Principles and Practice. New York, NY, Raven Press,
1991, pp 1443-1464.
43. A 53-year-old man reports a 5-week history of worsening low back pain accompanied by
bilateral knee and ankle pain and swelling. He also reports a lesser degree of neck and
left elbow pain. He denies any history of trauma or provocative episodes. His medical
history is significant for Reiters syndrome more than 25 years ago, with no subsequent
exacerbations. Furthermore, he has recently returned from a vacation in Costa Rica and
noted the development of infectious gastroenteritis with diarrhea within 1 week of his
return. This was treated with a 10-day course of oral antibiotics and has since resolved.
He denies any significant bowel or urinary symptoms at this time. His neurologic
examination is essentially within normal limits, but is somewhat limited by his low back
and leg pain. What further investigation is most appropriate at this time?

1- Radiographs of the lumbar spine and bilateral knees and ankles


2- MRI of the lumbar spine with and without gadolinium contrast
3- Synovial fluid analysis of the involved joints for crystals and bacteria
4- Laboratory tests including a CBC count, erythrocyte sedimentation rate (ESR),
and C-reactive protein (CRP)
5- Laboratory tests including CBC count, rheumatoid factor (RF), antinuclear
antibodies (ANA), and human leukocyte antigen-B27 (HLA-B27)

PREFERRED RESPONSE: 4

DISCUSSION: The patient has pain involving the cervical and lumbar spine as well as pain and
swelling in both the knees and ankles. As such, this can be classified as polyarticular arthritis.
The presence of multiple joint symptoms in the lower extremities, the absence of a history of
trauma, and the multiple joints involved direct attention away from the spine as the etiology of
this patients pain. Radiographs of the involved joints are not likely to yield much useful
information to assist with a diagnosis. Likewise, an MRI scan of the lumbar spine is not likely to
provide much information regarding the etiology of the patients condition. When a
rheumatologic illness is suspected, the selective use of confirmatory laboratory testing can aid in
arriving at a correct diagnosis. A presumed case of gout or chondrocalcinosis can be confirmed
by the presence of the appropriate crystals in a joint-fluid aspiration. Because of the patients
recent trip to Costa Rica and the subsequent gastroenteritis, a CBC count, ESR, and CRP should
be ordered to rule out infectious and inflammatory versus noninflammatory conditions.
Rheumatoid factor (RF) in general should only be ordered for patients with polyarticular joint
inflammation for more than 6 weeks. The presence of rheumatoid factor does not indicate
rheumatoid arthritis. Antinuclear antibodies (ANA) should be ordered when a connective tissue
disease such as systemic lupus erythematosus (SLE) is suspected on the basis of specific history
and physical examination findings, such as inflammatory arthritis. Human leukocyte antigen-
B27 (HLA-B27) should be ordered only when the patients history is compatible with ankylosing
spondylitis or Reiters syndrome and this patient had a history of Reiters syndrome.

REFERENCES: Gardner GC, Kadel NJ: Ordering and interpreting rheumatologic laboratory
tests. J Am Acad Orthop Surg 2003;11:60-67.
Shojania K: Rheumatology: 2. What laboratory tests are needed? CMAJ 2000;162:1157-1163.
44. The 5-year outcome for patients with sciatica secondary to lumbar disk herniation shows
which of the following results?

1- Patients have the same likelihood of receiving disability whether treated with or
without surgery.
2- Sixty percent of surgically treated patients undergo at least one more operation
within 5 years.
3- Only 20% of patients treated with surgery report improved symptoms of back
and/or leg pain.
4- A smaller portion of surgical patients, compared to nonsurgically treated patients,
report improvement.
5- Fifty percent of patients treated nonsurgically seek surgery within 5 years.

PREFERRED RESPONSE: 1

DISCUSSION: Atlas and associates, in the Maine Lumbar Spine Study, reported that overall,
patients treated initially with surgery reported better outcomes. By 5 years, 19% of surgical
patients had undergone at least one additional lumbar spine operation, and 16% of nonsurgical
patients had opted for at least one lumbar spine operation. At the 5-year follow-up, 70% of
patients initially treated surgically reported improvement in their predominant symptom (back or
leg pain) versus 56% of those initially treated nonsurgically. They also noted that there was no
difference in the proportion of patients receiving disability compensation at the 5-year follow-up.

REFERENCE: Atlas SJ, Keller RB, Chang Y, et al: Surgical and nonsurgical management of
sciatica secondary to a lumbar disc herniation: Five-year outcomes from the Maine Lumbar
Spine Study. Spine 2001;26:1179-1187.
45. What is one of the principle concerns when a fracture such as the one seen in Figure 18
is encountered?

1- Fractures of the lower extremities


2- Paroxysmal hypertension
3- Infection
4- Epidural hematoma
5- Gastrointestinal bleeding

PREFERRED RESPONSE: 4

DISCUSSION: The injury shown is a fracture-dislocation and it is highly unstable. In addition


to this concern, spinal epidural hematomas have a much higher incidence in people with
ankylosing spondylitis following knee fracture. It is felt to be due to disrupted epidural veins,
with hypervascular epidural soft tissue in the setting of a rigid spinal canal. Patients with
ankylosing spondylitis may have other significant comorbidities, especially cardiac and
pulmonary, and these should be carefully assessed.

REFERENCES: Ludwig S, Zarro CM: Complications encountered in the management of


patients with ankylosing spondylitis, in Vaccaro AR, Regan JJ, Crawford AH, et al (eds):
Complications of Pediatric and Adult Spine Surgery. New York, NY, Marcel Dekker, 2004,
pp 279-290.
Wu CT, Lee ST: Spinal epidural hematoma and ankylosing spondylitis: Case report and review
of the literature. J Trauma 1998;44:558-561.
46. Retrograde ejaculation is most commonly associated with what surgical approach?

1- Anterior retroperitoneal approach to L5-S1


2- Anterior transperitoneal approach to L5-S1
3- Anterior retroperitoneal approach to L4-5
4- Minimally invasive lateral trans-psoas approach to L4-5
5- Open lateral approach to L4-5

PREFERRED RESPONSE: 2

DISCUSSION: Retrograde ejaculation is the sequela of an injury to the superior hypogastric


plexus. This structure needs protection, especially during anterior exposure of the lumbosacral
junction. Although the superior hypogastric plexus can be injured with anterior or anterolateral
spine surgery at any lumbar level, it is most at risk with anterior transperitoneal approaches to
the lumbosacral junction. To avoid this complication, the use of monopolar electrocautery
should be avoided during deep dissection in this region. The ideal anterior exposure starts with
blunt dissection just to the medial aspect of the left common iliac vein sweeping the prevertebral
tissues toward the patients right side.

REFERENCES: Flynn JC, Price CT: Sexual complications of anterior fusion of the lumbar
spine. Spine 1984;9:489-492.
Watkins RG (ed): Surgical Approaches to the Spine. New York, NY, Springer-Verlag, 1983,
p 107.
An HS, Riley LH III: An Atlas of Surgery of the Spine. New York, NY, Lippincott Raven,
1998, p 263.
47. What nerve is most likely to be injured during the anterior exposure of C2-3?

1- Facial
2- Superior laryngeal
3- Vagus
4- Hypoglossal
5- Phrenic

PREFERRED RESPONSE: 4

DISCUSSION: The hypoglossal nerve exits from the ansa cervicalis at approximately the C2-3
level and can be injured during retraction up to the C2 level. The superior laryngeal nerve lies at
about C4-5. The facial nerve is much higher. The vagus nerve runs with the internal jugular and
carotid much more laterally. The phrenic nerve exits posteriorly.

REFERENCES: Chang U, Lee MC, Kim DH: Anterior approach to the midcervical spine, in
Kim DH, Henn JS, Vaccaro AR, et al (eds): Surgical Anatomy and Techniques to the Spine.
Philadelphia, PA, Saunders Elsevier, 2006, pp 45-54.
Netter GH: Atlas of Human Anatomy. Summit, NJ, Ciba-Geigy Corporation, 1989.
48. A 24-year-old man sustains the injury shown in Figures 19a through 19e in a paragliding
accident. He is neurologically intact. He also sustained fractures of his left femur and
right distal radius. Which of the following represents the best option for management of
the spinal injury?

1- Bed rest for 6 weeks, followed by mobilization in a thoracolumbosacral orthosis


(TLSO) until the fracture has healed
2- Immediate mobilization in a TLSO, continuing until the fracture has healed
3- Anterior corpectomy with strut grafting and placement of anterior fixation
4- Anterior corpectomy and strut grafting followed by posterior spinal fusion and
instrumentation
5- Posterior spinal fusion and instrumentation

PREFERRED RESPONSE: 5

DISCUSSION: The injury pattern is that of a burst fracture at L1 contiguous with a compression
fracture at T12. There is associated kyphosis and slight spondylolisthesis of T12 on L1.
Treatment of this type of burst fracture in neurologically intact patients is somewhat
controversial, with at least one study demonstrating equal long-term results comparing
nonsurgical treatment to surgical treatment. In this study, however, body casts were used
initially in the nonsurgical group. Moreover, because this patient has multiple fractures, spinal
fracture stabilization should be considered to facilitate early mobilization. Surgical stabilization
and fusion via a posterior approach is the best treatment option in this patient. Anterior
decompression is not necessary since the patient is neurologically intact.

REFERENCES: McLain RF, Benson DR: Urgent surgical stabilization of spinal fractures in
polytrauma patients. Spine 1999;24:1646-1654.
Wood K, Butterman G, Mehbod A, et al: Operative compared with nonoperative treatment of a
thoracolumbar burst fracture without neurological deficit: A prospective, randomized study.
J Bone Joint Surg Am 2003;85:773-781.
Spivak JM, Connolly PJ (eds): Orthopaedic Knowledge Update: Spine 3. Rosemont, IL,
American Academy of Orthopaedic Surgeons, 2006, pp 201-216.
49. An 82-year-old man is seen in consultation after being admitted for a fall from ground
level. There was no loss of consciousness and the patient recalls striking his head and
sustaining a hyperextension-type injury to the cervical spine. Examination reveals an
8-cm head laceration with only mild axial neck tenderness. He has generalized weakness
throughout the upper extremities and maintained motor function of the lower extremities.
There are no obvious sensory deficits, and the bulbocavernous reflex and deep tendon
reflexes are maintained. What is the most appropriate diagnosis at this time?

1- Anterior cord syndrome


2- Central cord syndrome
3- Posterior cord syndrome
4- Brown-Squard syndrome
5- Spinal shock

PREFERRED RESPONSE: 2

DISCUSSION: Incomplete cord syndromes have variable neurologic findings with partial loss of
sensory and/or motor function below the level of injury. Incomplete cord syndromes include the
anterior cord syndrome, the Brown-Squard syndrome, central cord syndrome, and posterior
cord syndrome. Central cord syndrome is characterized with greater motor weakness in the
upper extremities than in the lower extremities. The pattern of motor weakness shows greater
distal involvement in the affected extremity than proximal muscle weakness. Anterior cord
syndrome involves a variable loss of motor function and pain and/or temperature sensation, with
preservation of proprioception. The Brown-Squard syndrome involves a relatively greater
ipsilateral loss of proprioception and motor function, with contralateral loss of pain and
temperature sensation. Posterior cord syndrome is a rare injury and is characterized by
preservation of motor function, sense of pain, and light touch, with loss of proprioception and
temperature sensation below the level of the lesion. Spinal shock is the period of time, usually
24 hours, after a spinal injury that is characterized by absent reflexes, flaccidity, and loss of
sensation below the level of the injury.

REFERENCES: Penrod LE, Hegde SK, Ditunno JF: Age effect on prognosis for functional
recovery in acute, traumatic central cord syndrome. Arch Phys Med Rehab 1990;71:963-968.
Harrop JS, Sharan A, Ratliff J: Central cord injury: Pathophysiology, management, and
outcomes. Spine J 2006;6:198S-206S.
50. Kyphosis from a vertebral osteoporotic compression fracture often results in progressive
kyphosis due to

1- progressive increase in lumbar lordosis.


2- load transfer to the superior adjacent vertebra.
3- normalization of load transfer with working kyphosis.
4- reduced strain at the occipito-cervical junction.
5- reduced strain at the apex of the deformity.

PREFERRED RESPONSE: 2

DISCUSSION: Kayanja and associates, in a number of biomechanical studies, showed that in a


kyphotic spine the strain is located at the apex of the deformity, the force is transmitted to the
superior adjacent vertebrae, and that realignment and cement augmentation effectively normalize
the load transfer.

REFERENCES: Kayanja MM, Ferrara LA, Lieberman IH: Distribution of anterior cortical shear
strain after a thoracic wedge compression fracture. Spine J 2004;4:76-87.
Kayanja MM, Togawa D, Lieberman IH: Biomechanical changes after the augmentation of
experimental osteoporotic vertebral compression fractures in the cadaveric thoracic spine. Spine
J 2005;5:55-63.
Kayanja MM, Schlenk R, Togawa D, et al: The biomechanics of 1, 2, and 3 levels of vertebral
augmentation with polymethylmethacrylate in multilevel spinal segments. Spine 2006;31:769-
774.
Kayanja M, Evans K, Milks R, et al: The mechanics of polymethylmethacrylate augmentation.
Clin Orthop Relat Res 2006;443:124-130.
51. A 58-year-old woman with rheumatoid arthritis has progressive neck pain, upper
extremity and lower extremity weakness, and difficulty with fine motor movements.
Examination reveals hyperreflexia with mild to moderate objective weakness but the
patient has no difficulty with ambulation for short distances. What is the most important
preoperative imaging finding that predicts full neurologic recovery with surgical
stabilization?

1- Basilar invagination of less than 1 cm


2- Anterior atlanto-dens interval of 4 mm
3- Posterior atlanto-dens interval of greater than 14 mm
4- Rotatory subluxation of less than 10 degrees
5- Subaxial subluxation of less than 3.5 mm

PREFERRED RESPONSE: 3

DISCUSSION: Boden and associates article presents compelling evidence that patients with
rheumatoid arthritis and neurologic deterioration in C1-2 instability are more likely to achieve
some improvement if the posterior atlanto-dens interval is greater than 10 mm on preoperative
studies. All the patients in their series who had neurologic deterioration and a preoperative
posterior atlanto-dens interval of greater than 14 mm achieved complete motor recovery.

REFERENCES: Boden SD, Dodge LD, Bohlman HH, et al: Rheumatoid arthritis of the cervical
spine: A long-term analysis with predictors of paralysis and recovery. J Bone Joint Surg Am
1993;75:1282-1297.
Boden SD, Clark CR: Rheumatoid arthritis of the cervical spine, in Clark CR (ed): The Cervical
Spine, ed 3. Philadelphia, PA, Lippincott Raven, 1998, pp 755-764.
Monsey RD: Rheumatoid arthritis of the cervical spine. J Am Acad Orthop Surg 1997;5:240-
248.
52. Figures 20a through 20d show the radiographs and MRI scans of a 59-year-old woman
who has had symptoms consistent with progressive neurogenic claudication and back
pain for the past 9 months. In the last 6 months, nonsurgical management consisting of
nonsteroidal anti-inflammatory drugs, physical therapy, and a series of epidural steroid
injections have been used; however the injections, while beneficial, have provided only
temporary relief of her symptoms. What is the most appropriate management at this
time?

1- Repeat trial of epidural steroid injections


2- Pain management referral for narcotic management of symptoms
3- Lumbar laminectomies at L4-5
4- Lumbar laminectomies and fusion at L4-5
5- Bilateral lumbar laminotomies at L3-4 and L4-5

PREFERRED RESPONSE: 4

DISCUSSION: Patients with a degenerative spondylolisthesis and severe stenosis who have
failed appropriate nonsurgical management are candidates for surgical intervention. Most
studies show good to excellent results in more than 85% of patients after lumbar decompression
for stenosis. Atlas and associates found that at 8- to 10-year follow-up, leg pain relief and back-
related functional status were greater in those patients opting for surgical treatment of the
stenosis. Similarly, the decision to fuse a spondylolisthetic segment has been supported in the
literature. Herkowitz and Kurz compared decompressive laminectomy alone and decompressive
laminectomy with intertransverse arthrodesis in 50 patients with single-level spinal stenosis and
degenerative spondylolisthesis. They demonstrated good to excellent results in 90% of the fused
group compared to 44% in the nonfusion group. The decision to include instrumentation during
the fusion is more controversial. Whereas the use of instrumentation has shown to improve
fusion rates, it has not been conclusively shown to improve the overall clinical outcomes of
patients.

REFERENCES: Atlas SJ, Keller RB, Wu YA, et al: Long-term outcomes of surgical and
nonsurgical management of lumbar spinal stenosis: 8 to 10 year results from the Maine lumbar
spine study. Spine 2005;30:936-943.
Herkowitz HN, Kurz LT: Degenerative lumbar spondylolisthesis with spinal stenosis: A
prospective study comparing decompression with decompression and intratransverse process
arthrodesis. J Bone Joint Surg Am 1991;73:802-808.
Fischgrund JS, Mackay M, Herkowitz HN, et al: 1997 Volvo Award winner in clinical studies.
Degenerative lumbar spondylolisthesis with spinal stenosis: A prospective, randomized study
comparing decompressive laminectomy and arthrodesis with and without spinal instrumentation.
Spine 1997;22:2807-2812.
53. A 29-year-old man reports a 2-week history of severe neck pain after being struck sharply
on the back of the head and neck while moving a refrigerator down a flight of stairs.
Initial evaluation in the emergency department revealed no obvious fracture and he was
discharged in a soft collar. Neurologic examination is within normal limits, and
radiographs taken in the office are shown in Figures 21a through 21c. Subsequent MRI
scans show intra-substance rupture of the transverse atlantal ligament. What is the most
appropriate treatment option at this time?

1- Discontinue use of the soft collar and encourage range of motion


2- Semi-rigid collar immobilization for 6 to 8 weeks
3- Surgical stabilization
4- Halo skeletal fixation
5- Outpatient physical therapy with isometric neck exercises

PREFERRED RESPONSE: 3

DISCUSSION: Dickman and associates classified injuries of the transverse atlantal ligament into
two categories. Type I injuries are disruptions through the substance of the ligament itself. Type
II injuries render the transverse ligament physiologically incompetent through fractures and
avulsions involving the tubercle of insertion of the transverse ligament on the C1 lateral mass.
Type I injuries are incapable of healing without supplemental internal fixation. Type II injuries
can be treated with a rigid cervical orthosis with a success rate of 74%. Surgery may be required
for type II injures that fail to heal with 3 to 4 months of nonsurgical management.

REFERENCES: Findlay JM: Injuries involving the transverse atlantal ligament: Classification
and treatment guidelines based upon experience with 39 injuries. Neurosurgery 1996;39:210.
Dickman CA, Mamourian A, Sonntag VK, et al: Magnetic resonance imaging of the transverse
atlantal ligament for the evaluation of atlantoaxial instability. J Neurosurgery 1991;75:221-227.
54. Figure 22 reveals what anatomic variant of the lumbar spine?

1- Spina bifida occulta


2- Unilateral sacralization
3- An aplastic or hypoplastic pedicle
4- Lumbarization
5- Facet tropism

PREFERRED RESPONSE: 2

DISCUSSION: Unilateral sacralization of the fifth lumbar vertebra was first described by
Bertolotti in 1917. Bertolottis syndrome is present in 12% to 21% of the population. The
altered biomechanics have been postulated to cause low back pain by placing increased stress on
the adjacent cephalad disk, thus contributing to accelerated degenerative disk disease at this
level. It has also been found that the neoarticulation between the enlarged transverse process and
the sacrum and/or ilium may be a source of neural impingement on the exited L5 nerve root and
results in radicular pain syndrome. Brault and associates reported on a case treated surgically at
the Mayo Clinic, in which the pain generator was found to be the contralateral facet joint.

REFERENCES: Brault JS, Smith J, Currier BL: Partial lumbosacral transitional vertebra
resection for contralateral facetogenic pain. Spine 2001;26:226-229.
Quinlan JF, Duke D, Eustace S: Bertolottis syndrome: A cause of back pain in young people.
J Bone Joint Surg Br 2006;88:1183-1186.
Whelan MA, Feldman F: The variant lumbar pedicle. Neuroradiology 1982;22:235-242.
55. Posterior lumbar spine arthrodesis may be associated with adjacent segment degeneration
cephalad or caudad to the fusion segment. Which of the following is the predicted rate of
symptomatic degeneration at an adjacent segment warranting either decompression
and/or arthrodesis at mid-range follow-up (5-10 years) after lumbar fusion?

1- 2%
2- 10%
3- 25%
4- 50%
5- 80%

PREFERRED RESPONSE: 3

DISCUSSION: The rate of symptomatic degeneration at an adjacent segment warranting either


decompression or arthrodesis was predicted to be 16.5% at 5 years and 36.1% at 10 years based
on a Kaplan-Meier analysis.

REFERENCE: Ghiselli G, Wang J, Bhatia NN, et al: Adjacent segment degeneration in the
lumbar spine. J Bone Joint Surg Am 2004;86:1497-1503.
56. A 24-year-old man who was involved in a high speed motor vehicle accident is
transferred for definitive care after having been diagnosed with an acute spinal cord
injury from a fracture-dislocation at C6-7. He has a complete C6 neurologic level and it
is now approximately 10 hours from his injury. What is the most appropriate
pharmacologic treatment at this time?

1- No pharmacologic intervention is recommended at this time


2- Administration of methylprednisolone with an initial bolus of 30 mg/kg followed
by 5.4 mg/kg for 24 hours
3- Administration of methylprednisolone with an initial bolus of 30 mg/kg followed
by 5.4 mg/kg for 48 hours
4- Administration of naloxone with an initial bolus of 30 mg/kg followed by
5.4 mg/kg for 24 hours
5- Administration of naloxone with an initial bolus of 30 mg/kg followed by
5.4 mg/kg for 48 hours

PREFERRED RESPONSE: 1

DISCUSSION: The standard practice in the pharmacologic treatment of a spinal cord injury in
the United States has been the administration of methylprednisolone with an initial bolus of 30
mg/kg followed by 5.4 mg/kg for 24 hours, in accordance with the findings of the second and
third National Acute Spinal Cord Injury Studies (NASCIS). Although the studies have
subsequently drawn criticism for their methodology and outcomes, it has been generally
accepted that beneficial neurologic outcomes were anticipated in patients who were able to start
the protocol within 8 hours of their initial injury. Further improvement was noted in patients
receiving the methylprednisolone within 3 hours of their injury and continuing an infusion for
48 hours. In this patient, who is outside the 8-hour treatment window, no studies have supported
starting the methylprednisolone protocol at this time.

REFERENCES: Braken MB, Shepard MJ, Holford TR, et al: Administration of


methylprednisolone for 24 or 48 hours or tirilazad mesylate for 48 hours in the treatment of acute
spinal cord injury: Results of the third National Acute Spinal Cord Injury Randomized
Controlled Trial. National Acute Spinal Cord Injury Study. JAMA 1997;277:1597-1604.
Kwon BK, Tetzlaff W, Grauer JN, et al: Pathophysiology and pharmacologic treatment of acute
spinal cord injury. Spine J 2004;4:451-464.
57. Figures 23a and 23b show the MRI scans of a 50-year-old woman who has increasing
gait disturbance. She reports three falls in the past week. Examination reveals
hyperreflexia, motor weakness in the biceps and triceps, and a positive Hoffmans sign.
What is the most appropriate treatment plan?

1- Observation
2- Physical therapy
3- Epidural steroid injections
4- Cervical laminectomy
5- Anterior cervical diskectomy and fusion

PREFERRED RESPONSE: 5

DISCUSSION: The patient has obvious signs of progressive myelopathy. Based on her
significant physical examination findings, nonsurgical management will not significantly impact
her outcome. Cervical decompression alone is contraindicated in patients with cervical kyphosis
such as seen here. Anterior cervical fusion is the best option.

REFERENCES: Emery SE, Bohlman HH, Bolesta MJ, et al: Anterior cervical decompression
and arthrodesis for the treatment of cervical spondylotic myelopathy: Two to seventeen-year
follow-up. J Bone Joint Surg Am 1998;80:941-951.
Ferguson RJ, Caplan LR: Cervical spondylotic myelopathy. Neurol Clin 1985;3:373-382.
Herkowitz HN: A comparison of anterior cervical fusion, cervical laminectomy, and cervical
laminoplasty for the surgical management of multiple level spondylotic radiculopathy. Spine
1988;13:774-780.
58. What structure (arrow) is shown in Figure 24?

1- Ilioinguinal nerve
2- Sympathetic chain
3- Ureter
4- Iliac vein
5- L5 nerve

PREFERRED RESPONSE: 2

DISCUSSION: The structure illustrated is the sympathetic chain viewed from an anterolateral
view of the lower lumbar spine. It descends along the anterolateral aspect of the spine into the
pelvis closely adherent to the vertebral column. The spinal nerves, including L5, can be seen
exiting from the foramen. The ureters descend from the kidneys and cross anterior to the iliac
vessels to the bladder.

REFERENCES: Onibokun A, Khoo LT, Holly L: Anterior retroperitoneal approach to the


lumbar spine, in Kim DH, Henn JS, Vaccaro AR, et al (eds): Surgical Anatomy and Techniques
to the Spine. Philadelphia, PA, Saunders Elsevier, 2006, pp 101-105.
Netter GH: Atlas of Human Anatomy. Summit, NJ, Ciba-Geigy Corporation, 1989.
59. The best patient-related outcomes, following the surgical treatment of cauda equina
syndrome secondary to a large L5-S1 disk herniation, are most closely related to which of
the following?

1- Extent of bowel and bladder dysfunction


2- Extent of the motor deficit
3- Extent of the perianal saddle anesthesia
4- Timing of surgery
5- Location of the herniation

PREFERRED RESPONSE: 4

DISCUSSION: The most predictable positive outcome from spinal surgery due to a cauda equina
syndrome is early surgical intervention before any significant neurologic deficit develops. Meta-
analysis studies demonstrate that surgical intervention more than 48 hours after the onset of
cauda equina syndrome show an increased risk for poor outcomes.

REFERENCES: Ahn UM, Ahn NU, Buchowski JM, et al: Cauda equina syndrome secondary to
lumbar disc herniation: A meta-analysis of surgical outcomes. Spine 2000;25:1515-1522.
Kohles SS, Kohles JD, Karp AP, et al: Time-dependent surgical outcomes following cauda
equina syndrome diagnosis: Comments on meta-analysis. Spine 2004;29:1281-1287.
60. A 45-year-old man undergoes an anterior cervical diskectomy and fusion at C5-6 and
C6-7 with instrumentation. During the first postoperative visit at 1 week, the patient
reports difficulty swallowing and mild anterior cervical tightness. The anterior wound is
benign and the patient denies any dyspnea or shortness of breath. A postoperative
radiograph is seen in Figure 25. What is the most appropriate management at this time?

1- Admit for observation and reassurance


2- Surgical exploration and removal of the anterior instrumentation
3- Esophageal swallowing study
4- Soft cervical collar and early range-of-motion exercises
5- CT of the cervical spine

PREFERRED RESPONSE: 1

DISCUSSION: The radiograph shows significant prevertebral soft-tissue swelling following a


two-level anterior cervical diskectomy and fusion. The incidence of dysphagia 2 years after
anterior cervical spine surgery is 13.6%. Risk factors for long-term dysphagia after anterior
cervical spine surgery include gender, revision surgeries, and multilevel surgeries. The use of
instrumentation, higher levels, or corpectomy versus diskectomy did not significantly increase
the prevalence of dysphagia. Lee and associates demonstrated that while dysphagia after anterior
cervical spine surgery is a common early finding, it generally decreases significantly by
6 months with nonsurgical management. A minority of patients experience moderate or severe
symptoms by 6 months after the procedure. Female gender and multiple surgical levels have
been identified as risk factors for the development of postoperative dysphagia.

REFERENCES: Lee MJ, Bazaz R, Furey CG, et al: Risk factors for dysphagia after anterior
cervical spine surgery: A two-year prospective cohort study. Spine J 2007;7:141-147.
Bazaz R, Lee MJ, Yoo JU: Incidence of dysphagia after anterior cervical spine surgery:
A prospective study. Spine 2002;27:2453-2458.
61. Steroids are thought to prevent neurologic deterioration after traumatic spinal cord injury
by which of the following mechanisms?

1- Maintains calcium influx into damaged cells


2- Destabilizes lysosomal membranes in the zone of injury
3- Reduces TNF-alpha expression
4- Increases NF-kB binding capacity
5- Maintains free radical oxidation

PREFERRED RESPONSE: 3

DISCUSSION: The proposed mechanisms by which steroids such as methylprednisolone are


thought to prevent neurologic deterioration by limiting secondary insult, include: decreasing the
area of ischemia in the cord, reducing TNF-alpha expression and NF-kB binding activity,
decreasing free radical oxidation and thus stabilizing cell and lysosomal membranes, and
checking the influx of calcium into the injured cells, thus reducing cord edema.

REFERENCES: Slucky AV: Pathomechanics of spinal cord injury. Spine: State Art Rev
1999;13:409-417.
Torg JS, Thibault L, Sennett B, et al: The Nicolas Andry Award. The pathomechanics and
pathophysiology of cervical spinal cord injury. Clin Orthop Relat Res 1995;321:259-269.
62. Which of the following mechanisms of inhibition has been linked to cigarette smoking
and lumbar spinal fusion?

1- Diminished revascularization of cancellous bone graft


2- Increased activity of osteoblasts
3- Increased activity of osteocytes
4- Antibody-induced necrosis
5- Inhibition of prostaglandins

PREFERRED RESPONSE: 1

DISCUSSION: Cigarette smoking has been directly linked to pseudarthrosis in spinal fusions.
The direct mechanism of action is diminished revascularization of cancellous bone graft.
Additionally, a smaller area of revascularization is seen in these grafts, as well as an increased
area of necrosis. Increased activity of osteoblasts would result in more bone production.
Increased activity of osteocytes would not affect the fusion because osteocytes are mature bone
cells.

REFERENCE: Daftari TK, Whitesides TE Jr, Heller JG, et al: Nicotine on the revascularization
of bone graft: An experimental study in rabbits. Spine 1994;19:904-911.
63. Which of the following is considered the most effective means of identifying an evolving
motor tract injury during cervical spine surgery?

1- Mean arterial blood pressure monitoring


2- SSEP monitoring
3- Free-run electromyography
4- Transcranial motor monitoring (tceMEP)
5- Wake-up test

PREFERRED RESPONSE: 4

DISCUSSION: In a study of 427 patients undergoing cervical spine surgery, 12 patients


demonstrated substantial or complete loss of amplitude of the tceMEPs. Ten of those patients
had complete reversal of the loss following prompt intraoperative intervention. SSEP
monitoring failed to identify any changes in one of the two patients that awoke with a new motor
deficit. SSEP changes lagged behind the tceMEP changes in patients in which major changes
were detected by both modalities. TceMEP monitoring was 100% sensitive and 100% specific.
SSEP monitoring was only 25% sensitive and 100% specific.

REFERENCE: Hilibrand AS, Schwartz DM, Sethuraman V, et al: Comparison of transcranial


electric motor and somatosensory evoked potential monitoring during cervical surgery. J Bone
Joint Surg Am 2004;86:1248-1253.
64. A previously healthy 29-year-old man reports a 2-day history of severe atraumatic lower
back pain. He denies any bowel or bladder difficulties and no constitutional signs.
Examination is consistent with mechanical back pain. No focal neurologic deficits or
pathologic reflexes are noted. What is the most appropriate management?

1- Radiographs, including anterior, lateral, and oblique views


2- MRI of the lumbar spine and follow-up at the clinic in 1 week
3- Caudal epidural steroid injection
4- Reassurance, limited analgesics, and early range of motion as tolerated
5- Immediate MRI of the lumbar spine and possible urgent surgical decompression

PREFERRED RESPONSE: 4

DISCUSSION: In general, a previously healthy patient with an acute onset of nontraumatic


lower back pain does not need diagnostic imaging before proceeding with therapeutic treatment.
In the absence of any red flags during the history and physical examination, such as trauma or
constitutional symptoms (ie, fevers, chills, weight loss), the appropriate treatment for acute onset
lower back pain is purely symptomatic treatment including limited analgesics and early range of
motion. Diagnostic imaging is not necessary unless the initial treatment is unsuccessful and
symptoms are prolonged. Miller and associates suggested that the use of radiographs can lead to
better patient satisfaction but not necessarily better outcomes.

REFERENCES: Miller P, Kendrick D, Bentley E, et al: Cost effectiveness of lumbar spine


radiographs in primary care patients with low back pain. Spine 2002;27:2291-2297.
Wong DA, Transfeldt E: Macnabs Backache, ed 4. Philadelphia, PA, Lippincott Williams and
Wilkins 2007, pp 298-338.
65. Sacral fractures are most likely to be associated with neurologic deficits when they
involve what portion of the sacrum?

1- Zone 1 (the ala)


2- Zone 2 (the foramina)
3- Zone 3 (the central canal)
4- Zones 1 and 2
5- The sacral laminae

PREFERRED RESPONSE: 3

DISCUSSION: Denis divided the sacrum into three zones: zone 1 represents the lateral ala, zone
2 represents the foramina, and zone 3 represents the central canal. A fracture is classified
according to its most medial extension. Those in zone 3 are typically bursting-type fractures or
fracture-dislocations and are most prone to neurologic sequelae.

REFERENCES: Denis F, Davis S, Comfort T: Sacral fractures: An important problem.


A retrospective analysis of 236 cases. Clin Orthop Relat Res 1988;227:67-81.
Wood KB, Denis F: Fractures of the sacrum and coccyx, in Vacarro AR (ed): Fractures of the
Cervical, Thoracic and Lumbar Spine. New York, NY, Marcel Dekker, 2003, pp 473-488.
66. Which of the following is associated with the use of bisphosphonates in the setting of
metastatic breast cancer to the spine?

1- Reduction in skeletal-related events by 30% to 40%


2- Jaw osteonecrosis in 15% of patients
3- Pain improvement in only 30% of patients
4- Improvement in serum hypocalcemia in 40% of patients
5- Accelerated bone destruction in 10% of patients

PREFERRED RESPONSE: 1

DISCUSSION: The indications of bisphosphonate therapy in breast cancer patients range from
the correction of hypercalcemia to the prevention of cancer treatment-induced bone loss.
Bisphosphonates reduce metastatic bone pain in at least 50% of patients and can reduce the
frequency of skeletal-related events by 30% to 40%. Osteonecrosis of the jaw could occur in up
to 2.5% of breast cancer patients during long-term bisphosphonate therapy.

REFERENCE: Body JJ: Breast cancer: Bisphosphonate therapy for metastatic bone disease.
Clin Cancer Res 2006;12:6258s-6263s.
67. A 67-year-old retired steelworker was involved in a motor vehicle accident and sustained
a midcervical spinal cord injury. Radiographs and MRI scans reveal severe cervical
stenosis and spondylosis without fractures or dislocations. Neurologic examination
reveals an ASIA C spinal cord impairment with greater motor involvement of the upper
extremities than the lower extremities. What is the probability that the patient eventually
will become ambulatory?

1- 2% to 5%
2- 15% to 20%
3- 35% to 45%
4- 60% to 70%
5- Greater than 90%

PREFERRED RESPONSE: 3

DISCUSSION: The patient sustained an incomplete spinal cord injury known as central cord
syndrome. Central cord syndrome characteristically has disproportionate involvement of the
upper extremities with the lower extremities being relatively spared. It is most commonly seen
after cervical injuries in elderly patients with spondylosis and spinal stenosis, often without
fracture. Penrod and associates noted that 23 of 59 patients with central cord syndrome
(ASIA C and D) ultimately walked. The poorest prognosis, however, was in ASIA C patients
older than age 50, in which only 40% walked.

REFERENCES: Penrod LE, Hegde SK, Ditunno JF Jr: Age effect on prognosis for functional
recovery in acute, traumatic central cord syndrome. Arch Phys Med Rehab 1990;71:963-968.
Northrup BE: Acute injuries to the spine and spinal cord: Evaluation and early treatment, in
Clark CR (ed): The Cervical Spine, ed 4. Philadelphia, PA, Lippincott Williams & Wilkins,
2005, p 735.
68. A 20-year-old man involved in a motor vehicle accident is brought to the emergency
department with a C6-7 unilateral facet dislocation. His neurologic examination reveals
a focal left-sided C7 nerve root palsy. He is awake and cooperative with questioning
and has no other obvious traumatic injuries. What is the most appropriate treatment at
this time?

1- Further imaging studies, including MRI


2- An awake closed reduction with Gardner-Wells traction with neurologic
examination
3- Immobilization in a halo skeletal fixation for definitive treatment
4- Closed reduction under general anesthesia
5- Immediate open reduction and internal fixation in the surgical suite

PREFERRED RESPONSE: 2

DISCUSSION: In the patient who is neurologically intact or has an incomplete injury from a
cervical facet dislocation, a closed reduction with weighted tong traction is appropriate when the
patient is awake, alert, and cooperative. Although there is a risk that a cervical facet dislocation
could occur with an underlying cervical disk herniation, Vaccaro and associates have shown that
closed reduction can be safely carried out in the awake, responsive patient. Closed reduction can
be performed in the emergency department with traction with skull tongs or a halo ring. A slow
stepwise application of weight is added until a reduction is achieved. Any worsening of the
neurologic status of the patient requires immediate termination of the closed reduction and
further diagnostic imaging before proceeding with further treatment.

REFERENCES: Vaccaro AR, Falatyn SP, Flanders AE, et al: Magnetic resonance evaluation of
the intervertebral disc, spinal ligaments, and spinal cord before and after closed traction
reduction of cervical spine dislocations. Spine 1999;24:1210-1217.
Hart RA: Cervical facet dislocation: When is magnetic resonance imaging indicated? Spine
2002;27:116-117.
Cotler JM, Herbison GJ, Nasuti JF, et al: Closed reduction of traumatic cervical spine dislocation
using traction weights up to 140 pounds. Spine 1993;18:386-390.
69. A 66-year-old man reports a 2-week history of worsening low back and leg pain. He
reports that his pain is aggravated by lying down and relieved by standing and walking.
He notes that he has been losing weight recently and that his pain has been awakening
him during the night. His medical history is significant for hypertension, coronary artery
disease, and prostate cancer. His physical examination is essentially unremarkable.
Lumbar radiographs are within normal limits. What is the most appropriate management
for this patient?

1- MRI of chest
2- Laboratory studies, including a complete blood cell (CBC) count, erythrocyte
sedimentation rate (ESR), and urinalysis, PSA, CEA
3- Activity alterations to avoid undue back irritation
4- Comfort measures, including medications
5- Spinal manipulative therapy within the first 6 weeks

PREFERRED RESPONSE: 2

DISCUSSION: In the initial assessment of acute low back pain in adults, no diagnostic testing is
indicated during the first 4 weeks in the absence of red flags for a serious underlying
condition. The purpose of the initial assessment of acute low back pain in adults is to rule out
serious underlying conditions presenting as low back pain. The Agency for Healthcare Policy
and Research, in its 1994 clinical practice guideline, identified four serious conditions that may
present with low back pain, including fracture, tumor, infection, and cauda equina syndrome.
This patient has five red flags for a spinal tumor as a possible etiology of his low back pain,
including age of older than 50 years, constitutional symptoms (recent weight loss), pain worse
when supine, severe nighttime pain, and a history of cancer. Of these, his history of cancer is
most significant, as greater than 90% of spinal tumors are metastatic. In order of frequency,
breast, prostate, lung, and kidney make up approximately 80% of all secondary spread to the
spine. In the presence of red flags for tumor or infection, it is recommended that the clinician
obtain a CBC count, ESR, and a urinalysis. If these are within normal limits and suspicions still
remain, consider consultation or seek further evidence with a bone scan, radiographs, or
additional laboratory studies. Negative radiographs alone are insufficient to rule out disease. If
radiographs are positive, the anatomy can be better defined with MRI.

REFERENCES: Agency for Health Care Policy and Research, Bigos SJ (ed): Acute Low Back
Problems in Adults. Rockville, MD, US Department of Health and Human Services, AHCPR
Publication 95-0642, Clinical Practice Guideline #14, 1994.
Gertzbein SD: Metastatic spine tumors, in Herkowitz HN, Dvorak J, Bell G, et al (eds): The
Lumbar Spine, ed 3. Philadelphia, PA, Lippincott Williams & Wilkins, 2004, pp 792-802.
70. Which of the following increases radiation exposure to patients and personnel during
surgery?

1- Orienting the beam in the opposite direction of the working team and keeping the
team outside a 6-foot radius of the fluoroscopy machine
2- Orienting the cathode ray tube beneath the patient with the image intensifier
receptor as close to the patient as possible
3- Limiting the beam on time to only what is clinically important
4- The use of continuous fluoroscopy whenever possible to ensure proper placement
of implants
5- The use of lead glasses, a thyroid shield, and a lead apron with an equivalent lead
thickness of 0.25 mm

PREFERRED RESPONSE: 4

DISCUSSION: Continuous fluoroscopy and cine radiography expose the patient and personnel
to markedly increased levels of direct and scatter radiation exposure. Continuous fluoroscopy
should be limited to only what is absolutely needed for safe completion of the procedure. By
orienting the cathode ray tube beneath the patient and placing the image intensifier as close as
clinically possible to the patient, scatter radiation exposure to the personnel is minimized.

REFERENCE: Wagner L, Archer B: Minimizing Risks from Fluoroscopic X-rays: A


Credentialing Program for Anesthesiologists, Cardiologists, Surgeons, Radiologists, and
Urologists, ed 3. The Woodlands, TX, Partners in Radiation Management, 2000.
71. A 78-year-old woman undergoes her third lumbar decompression and fusion from L3 to
L5 without complication. On the morning of postoperative day 3, examination reveals
painless, flaccid weakness of both lower extremities. She also has an absent
bulbocavernous reflex and a mild saddle paresthesia. MRI scans of the lumbar spine are
shown in Figures 26a and 26b. What is the most appropriate management at this time?

1- Continued serial neurologic examinations


2- CT with a myelogram of the lumbar spine
3- Immediate surgical exploration and hematoma drainage
4- Electromyography of bilateral lower extremities
5- IV antibiotics for 24 hours, followed by surgical exploration if symptoms persist

PREFERRED RESPONSE: 3

DISCUSSION: The MRI scans reveal a large postoperative hematoma causing significant thecal
compression. An epidural hematoma with neurologic deficit is a surgical emergency requiring
immediate evacuation of the hematoma. Although the incidence of postoperative epidural
hematomas is rare, the consequences of a missed diagnosis can be catastrophic. Early
recognition and evacuation are essential in preserving or restoring neurologic function. Uribe
and associates attributed delayed postoperative hematomas to previous multiple lumbar surgeries
as a possible contributing factor.

REFERENCES: Yi S, Yoon do H, Kim KN, et al: Postoperative spinal epidural hematoma: Risk
factor and clinical outcome. Yonsei Med J 2006;47:326-332.
Uribe J, Moza K, Jimenez O, et al: Delayed postoperative spinal epidural hematomas. Spine J
2003;3:125-129.
72. Figures 27a through 27c show the radiographs and CT scan of a 27-year-old man who
sustained a low-velocity gunshot wound to the neck. He is quadriplegic (ASIA A),
hemodynamically stable, and does not have drainage from his wound. After initial
resuscitation and stabilization, the cervical spine and spinal cord injuries are best
managed by

1- wound debridement, anterior corpectomy, spinal cord decompression, dural


repair, and anterior fusion with strut graft and anterior plating.
2- wound debridement, anterior corpectomy, spinal cord decompression, dural
repair, anterior fusion with strut graft and anterior plating followed by posterior
laminectomy, and spinal cord decompression and dural repair with excision of the
bullet fragment.
3- wound debridement, anterior corpectomy, spinal cord decompression, dural
repair, anterior fusion with strut graft and anterior plating followed by
laminectomy and posterior fusion, and spinal cord decompression and dural repair
with excision of the bullet fragment.
4- laminectomy and posterior fusion, and spinal cord decompression and dural repair
with excision of the bullet fragment.
5- surgical treatment based on extraspinal pathology with orthotic treatment of the
spinal fractures.

PREFERRED RESPONSE: 5

DISCUSSION: Although the spinal canal has been penetrated, the lateral masses are intact
bilaterally with only partial destruction of the vertebral body and penetration of the lamina on
one side, thus the cervical spine is not unstable and surgical stabilization is not indicated. Dural
repair is not indicated since there is no external cerebrospinal fluid leakage. Surgical treatment
should be based on the need to treat extraspinal pathology only.

REFERENCES: Bono CM, Heary RF: Gunshot wounds to the spine. Spine J 2004;4:230-240.
Punjabi MM, Jue JJ, Dvorak J, et al: Cervical spine kinematics and clinical instability, in Clark
CR (ed): The Cervical Spine, ed 4. Philadelphia, PA, Lippincott Williams & Wilkins, 2005,
pp 55-87.
73. Which of the following is a true statement regarding thoracic disk herniations?

1- Are most commonly discovered during the fifth to seventh decades of life
2- Occur with similar frequency as cervical disk herniations
3- Occur most commonly in the midthoracic or apical region of the spine
4- Can be found in 40% of asymptomatic individuals
5- Are best treated surgically with posterior laminectomy and excision

PREFERRED RESPONSE: 4

DISCUSSION: Symptomatic herniations of the thoracic spine are much less common than those
of the cervical or lumbar region. They tend to occur most commonly during the third to fifth
decades of life and although they can be found at all levels, they are most common in the lower
third near the thoracolumbar region. Posterior laminectomy and disk excision has the highest
rate of neurologic deterioration and is not recommended. Multiple studies have shown that
herniated thoracic disks can be found at one or more levels in 40% of asymptomatic individuals.

REFERENCES: Shah RP, Grauer JN: Thoracoscopic excision of thoracic herniated disc, in
Vaccaro AR, Bono CM (eds): Minimally Invasive Spine Surgery. New York, NY, Informa
Healthcare, 2007, pp 73-80.
Bohlman HH, Zdeblick TA: Anterior excision of herniated thoracic discs. J Bone Joint Surg Am
1988;70:1038-1047.
74. A sentinel event is defined as an unexpected occurrence involving death or serious
physical or psychological injury, or the risk thereof. What is the most common sentinel
event related to spine surgery?

1- Surgery on the wrong patient


2- Surgery on the wrong side
3- Incorrect procedure performed
4- Intraoperative death
5- Surgery on the wrong level

PREFERRED RESPONSE: 5

DISCUSSION: Patient safety and prevention of medical errors is a major focus of recent national
advocacy groups. Analysis has shown that the most common sentinel event in spine surgery is
surgery on the wrong level. Therefore, it is recommended that every patient have the surgical
site signed, the level of surgery marked intraoperatively, and a radiograph taken. Surgery on the
wrong level is most likely to occur in single-level decompressive procedures.

REFERENCES: Wong DA, Watters WC III: To err is human: Quality and safety issues in spine
care. Spine 2007;32:S2-S8.
Wong DA: Spinal surgery and patient safety: A systems approach. J Am Acad Orthop Surg
2006;14:226-232.
75. What structure is most at risk with anterior penetration of C1 lateral mass screws?

1- Vertebral artery
2- External carotid artery
3- Internal carotid artery
4- Pharynx
5- Glossopharyngeal nerve

PREFERRED RESPONSE: 3

DISCUSSION: Posterior screw fixation of the upper cervical spine has gained a great deal of
popularity due to its stable fixation, obviating the use of halo vest immobilization, and its high
fusion rates. The use of screws in this location, however, has introduced a whole new set of
potential complications. Vertebral artery injury is one of the most feared complications
associated with screws in the C1/C2 region. This structure, however, is lateral and posterior at
the C2 level and then penetrates the foramen transversarium of C1 to lie cephalad to the arch of
C1 before entering the foramen magnum. It is the internal carotid artery that lies immediately
anterior to the arch of C1 that is particularly at risk by anterior penetration of C1 lateral mass or
C1-C2 transarticular screws as demonstrated by Currier and associates. The internal carotid
artery lies posterior to the pharynx. The external carotid artery and the glossopharyngeal nerve
are not at risk with this method of fixation.

REFERENCES: Currier BL, Todd LT, Maus TP, et al: Anatomic relationship of the internal
carotid artery to the C1 vertebra: A case report of cervical reconstruction for chordoma and pilot
study to assess the risk of screw fixation of the atlas. Spine 2003;28:E461-E467.
Grant JC: Grants Atlas of Anatomy, ed 6. Baltimore, MD, Williams & Wilkins, 1972.
Harms J, Melcher RP: Posterior C1-C2 fusion with polyaxial screw and rod fixation. Spine
2001;26:2467-2471.
76. During the application of halo skeletal fixation, the most appropriate position for the
placement of the anterior halo pins is approximately 1 cm above the superior orbital
rim and

1- lateral placement, directly within the temporalis muscle.


2- lateral to the medial third of the orbit.
3- lateral to the lateral aspect of the orbit.
4- above the medial third of the orbit.
5- lateral between the temporalis muscle and zygomatic temporal nerve.

PREFERRED RESPONSE: 2

DISCUSSION: Halo fixation is the most rigid form of cervical orthosis, but complications can
arise from improper placement of the initial halo ring. A relatively safe zone for anterior pin
placement is located 1 cm above the orbital rim and superior to the lateral two thirds of the orbit.
This position avoids the supraorbital and supratrochlear nerves and arteries over the medial one
third of the orbit. The more lateral positions in the temporal fossa have very thin bone and can
interfere with the muscles of mastication. Posterior pin site locations are less critical; positioning
on the posterolateral aspect of the skull, diagonal to the contralateral anterior pins, is generally
desirable.

REFERENCES: Botte MJ, Byrne TP, Abrams RA, et al: Halo skeletal fixation: Techniques of
application and prevention of complications. J Am Acad Orthop Surg 1996;4:44-53.
Garfin SR, Botte MJ, Nickel VL: Complications in the use of the halo fixation device. J Bone
Joint Surg Am 1987;69:954.
77. Figures 28a and 28b show the sagittal and axial lumbar MRI scans of a 72-year-old man
who reports dull aching back pain that spreads to his legs, calves, and buttocks. He has
had the pain for several years and it is precipitated by standing and walking and relieved
by sitting. His symptoms have been worsening over the past year and he notes that he is
leaning forward while walking to help relieve his symptoms. He has had no treatment to
date. What is his prognosis if he chooses to pursue nonsurgical management for this
condition?

1- He can expect complete resolution of his symptoms during the first month.
2- All patients experience relief within 3 months and continue to improve over the
next 4 years.
3- Most patients experience some pain relief within the first 3 months.
4- He may experience some improvement but if he requires surgery at a later date he
will have a poorer result because of the delay.
5- The patient requires immediate surgery to avoid permanent nerve damage.

PREFERRED RESPONSE: 3

DISCUSSION: The patient has lumbar spinal stenosis and the MRI scans reveal the pathology at
L4-5, which is secondary to posterior disk bulging and hypertrophy and infolding of the
ligamentum flavum, as well as degenerative facet arthrosis. The degree of spinal stenosis is
moderate and his symptoms are positional in nature. Tadokoro and associates reported on a
prospective study of 89 patients older than 70 years of age who underwent nonsurgical
management for lumbar spinal stenosis. They found the prognosis to be relatively good with
patients scoring at excellent or good for activities of daily living at final follow-up.
However, they did note that patients with a complete block on myelography did not respond
favorably to nonsurgical management. Amundsen and associates reported on a 10-year
prospective study comparing surgical care to nonsurgical management. They concluded that,
while the long-term results largely favored surgical treatment, more than half of the
nonsurgically managed patients had a satisfactory outcome. They also concluded that a delay of
surgery for some months did not worsen the prognosis. Therefore, their recommendation was for
an initial primarily nonsurgical approach.

REFERENCES: Amundsen T, Weber H, Nordal HJ, et al: Lumbar spinal stenosis: Conservative
or surgical management? A prospective 10-year study. Spine 2000;25:1424-1435.
Hilibrand AS, Rand N: Degenerative lumbar stenosis: Diagnosis and management. J Am Acad
Orthop Surg 1999;7:239-249.
Tadokoro K, Miyamoto H, Sumi M, et al: The prognosis of conservative treatments for lumbar
spinal stenosis: Analysis of patients over 70 years of age. Spine 2005;30:2458-2463.
78. Which of the following vertebrae has the smallest pedicle isthmic width in a
nondeformity patient?

1- T10
2- T11
3- T12
4- L1
5- L2

PREFERRED RESPONSE: 4

DISCUSSION: The smallest pedicle isthmic width is at L1, whereas T12 has the largest pedicle
width in the upper lumbar and lower thoracic spine. Although smaller in diameter than T12,
both T10 and T11 have larger pedicle widths than L1.

REFERENCE: Ofiram E, Polly DW, Gilbert TJ Jr, et al: Is it safe to place pedicle screws in the
lower thoracic spine than in the upper lumbar spine? Spine 2007;32:49-54.
79. Which of the following represents a contraindication for interspinous process
decompression for the treatment of lumbar spinal stenosis?

1- Grade I degenerative spondylolisthesis


2- Inability to walk at least 100 feet
3- Cauda equina syndrome
4- Fixed sensory deficit
5- Intermittent foot drop

PREFERRED RESPONSE: 3

DISCUSSION: Kondrashov and associates noted stable good outcomes at 4 years in 14 of 18


patients treated with X-STOP interspinous process decompression as defined as an improvement
over preoperative Oswestry scores of 15 points or more. Similar results were seen after 1 year in
a European study by Siddiqui and associates. Exclusion and inclusion criteria for these studies
varied somewhat, but cauda equina syndrome was the only exclusion criteria listed in both
studies. All of the other choices did not represent exclusion criteria in either study.

REFERENCES: Kondrashov DG, Hannibal M, Hsu KY, et al: Interspinous process


decompression with the X-STOP device for lumbar spinal stenosis: A 4-year follow-up study.
J Spinal Disord Tech 2006;19:323-327.
Siddiqui M, Smith FW, Wardlaw D: One-year results of X Stop interspinous implant for the
treatment of lumbar spinal stenosis. Spine 2007;32:1345-1348.
80. Which of the following statements about hoarseness due to vocal cord paralysis after
anterior cervical diskectomy and fusion is most accurate?

1- Vocal cord paralysis is three times as likely with a right-sided approach as


compared to a left-sided approach.
2- Vocal cord paralysis is twice as likely with a right-sided approach as compared to
a left-sided approach.
3- Vocal cord paralysis is equally likely with either a right-sided or a left-sided
approach.
4- Vocal cord paralysis is three times as likely with a left-sided approach as
compared to a right-sided approach.
5- Vocal cord paralysis is twice as likely with a left-sided approach as compared to a
right-sided approach.

PREFERRED RESPONSE: 3

DISCUSSION: It has been traditionally taught that a left-sided approach to the anterior cervical
spine is associated with a lower incidence of injury compared to the right-sided approach. This
is due in part to the anatomic differences in the path the recurrent laryngeal nerve (RLN) takes
on the right as compared to the left. Both nerves ascend in the tracheoesophageal groove after
branching off the vagus nerve in the upper thorax. The left-sided RLN loops around the aortic
arch and stays relatively medial as compared to the right-sided RLN which loops around the
right subclavian artery and is somewhat more lateral at this point, and therefore is theoretically
more vulnerable as it ascends toward the larynx before becoming protected in the
tracheoesophageal groove. Furthermore, the variant of a nonrecurrent inferior laryngeal nerve
branching directly off the vagus nerve at the level of the midcervical spine is much more
common on the right than the left. Despite this reasoning, there has been no clinical evidence to
suggest that laterality of approach for anterior cervical surgery makes any difference in the
incidence of vocal cord paralysis. Furthermore, two recent studies have shown that the incidence
of RLN injury and vocal cord paralysis is equal with either side of approach.

REFERENCES: Beutler WJ, Sweeney CA, Connolly PJ: Recurrent laryngeal nerve injury with
anterior cervical spine surgery risk with laterality of surgical approach. Spine 2001;26:1337-
1342.
Kilburg C, Sullivan HG, Mathiason MA: Effect of approach side during anterior cervical
discectomy and fusion on the incidence of recurrent laryngeal nerve injury. J Neurosurg Spine
2006;4:273-277.
81. A 23-year-old man is involved in a motor vehicle accident. An AP radiograph is shown
in Figure 29a, and axial and sagittal CT scans are shown in Figures 29b and 29c.
Neurologic examination shows 1/5 strength of his quadriceps and iliopsoas on the right,
with 1/5 quadriceps function on the left. Definitive treatment of his injury should
consist of

1- anterior corpectomy with interbody strut.


2- posterior fusion with instrumentation and posterolateral decompression.
3- closed reduction and a thoracolumbosacral orthosis (TLSO).
4- anterior reduction and instrumentation.
5- supine bed rest for 6 weeks, followed by immobilization in a TLSO.

PREFERRED RESPONSE: 2

DISCUSSION: The imaging studies show a fracture-dislocation. Surgical treatment of this


injury consists of a decompression reduction, stabilization, and fusion. A posterolateral
decompression can also be performed as necessary. An isolated anterior procedure in this type
of injury is contraindicated. The anterior longitudinal ligament is most likely intact; therefore, an
anterior procedure further destabilizes the spine. Reduction by an anterior approach would also
be difficult. Nonsurgical management of the neurologic injury in this patient is not indicated.

REFERENCES: Theiss SM: Thoracolumbar and lumbar spine trauma, in Stannard JP, Schmidt
AH, Kregor PJ (eds): Surgical Treatment of Orthopaedic Trauma. New York, NY, Thieme,
2007, pp 179-207.
Spivak JM, Connolly PJ (eds): Orthopaedic Knowledge Update: Spine 3. Rosemont, IL,
American Academy of Orthopaedic Surgeons, 2006, pp 201-216.
82. Surgical treatment for symptomatic disk herniations is associated with which of the
following?

1- Substantial rate of nerve root injury


2- Early relief of pain sustained out to 2 years
3- Recurrent herniation rate of 35%
4- Outcomes that are substantially worse than nonsurgical management
5- 10% rate of infectious diskitis

PREFERRED RESPONSE: 2

DISCUSSION: The recently published SPORT trial verifies that surgical treatment of
symptomatic disk herniations is associated with early and sustained pain relief. The trial also
verifies that nonsurgical management is associated with improved symptoms as well. Nerve root
injury, recurrent herniation, and diskitis are known complications of surgery, but all are less
common than described above.

REFERENCE: Weinstein JN, Lurie JD, Tosteson TD, et al: Surgical vs nonoperative treatment
for lumbar disk herniation: The Spine Patient Outcomes Research Trial (SPORT) observational
cohort. JAMA 2006;296:2451-2459.
83. A 25-year-old man is unresponsive at the scene of a high-speed motor vehicle accident
and remains obtunded. Initial evaluation in the emergency department reveals a left-
sided femoral shaft fracture and a right-sided humeral shaft fracture. The cervical spine
remains immobilized in a semi-rigid cervical collar, and the initial AP and lateral
radiographs obtained in the emergency department are unremarkable. What is the most
appropriate management at this time?

1- Lateral radiographs with passive flexion/extension views


2- Helical CT scan of the cervical-thoracic region
3- Careful manual palpation of the cervical spine for subtle defects or step-offs
4- MRI of the cervical spine
5- Continued use of the cervical collar until the patient becomes responsive for
examination

PREFERRED RESPONSE: 2

DISCUSSION: Clearance of the cervical spine can be difficult in the obtunded or unresponsive
patient. Various trauma series have been reported to detect up to 95% of cervical fractures but
only when ideal imaging views have been obtained, which is not often possible in the
unresponsive or uncooperative patient. Passively performed cervical flexion-extension under
live fluoroscopy has been suggested but is not without inherent risk in the potentially unstable
cervical spine. CT of the cervical spine has gained acceptance for the evaluation of these
patients given the excellent evaluation of the osseous anatomy and for the common availability
in most emergency departments. Sanchez and associates, using a protocol to evaluate for
cervical spine injuries after blunt trauma, were able to detect 99% of cervical fractures with
100% specificity.

REFERENCES: Chiu WC, Haan JM, Cushing BM, et al: Ligamentous injuries of the cervical
spine in unreliable blunt trauma patients: Incidence, evaluation, and outcome. J Trauma
2001;50:457-463.
Sanchez B, Waxman K, Jones T, et al: Cervical spine clearance in blunt trauma: Evaluation of a
computed tomography-based protocol. J Trauma 2005;59:179-183.
Nunez D Jr: Value of complete cervical helical computed tomographic scanning in identifying
cervical spine injury in the unevaluable blunt trauma patient with multiple injuries:
A prospective study. J Trauma 2000;48:988-989.
84. A 55-year-old woman undergoes an anterior cervical diskectomy and fusion at C5-C6
through a left-sided approach. One year later, she requires an anterior cervical
diskectomy and fusion on another level. Which of the following is considered a
contraindication to performing a right-sided approach for the revision procedure?

1- Revision surgery caudad to C6


2- Persistent left cervical radiculopathy
3- History of a left-sided Horners syndrome
4- Transient dysphagia following the initial anterior cervical procedure
5- Nonfunctional left vocal cord

PREFERRED RESPONSE: 5

DISCUSSION: When attempting a revision anterior cervical approach from the side opposite the
original approach, it is important to evaluate the function of the vocal cords. If this evaluation
reveals dysfunction of the vocal cord on the side of the original approach, then an approach on
the contralateral side should not be attempted. Injury to the stellate ganglion, which causes a
Horners syndrome, should not preclude an approach on the contralateral side. While the side of
the symptomatology can influence the surgeons choice as to the side of an anterior approach, it
does not preclude a certain approach. When approaching the lower cervical spine from the right
side, the recurrent laryngeal nerve can cross the surgical field and should be preserved.
Excessive intraoperative pressure on the esophagus can increase the incidence of dysphagia, but
its incidence is no different with either approach.

REFERENCES: Spivak JM, Connolly PJ (eds): Orthopaedic Knowledge Update: Spine 3.


Rosemont, IL, American Academy of Orthopaedic Surgeons, 2006, pp 387-394.
Edwards CC II, Riew KD, Anderson PA, et al: Cervical myelopathy: Current diagnostic and
treatment strategies. Spine J 2003;3:68-81.
85. A 56-year-old woman sustained the fracture shown in Figures 30a and 30b in a motor
vehicle accident. What mechanism is most likely responsible for the injury?

1- Flexion distraction
2- Vertical shear
3- Extension distraction
4- Flexion compression
5- Axial load

PREFERRED RESPONSE: 5

DISCUSSION: The CT scans show a burst fracture that results from an axial load injury. The
radiographic hallmark of a burst fracture is compression of the posterior cortex of the vertebral
body with retropulsion of bone into the spinal canal. AP radiographs often show widening of the
interpedicular distance with a fracture of the lamina.

REFERENCES: Theiss SM: Thoracolumbar and lumbar spine trauma, in Stannard JP, Schmidt
AH, Kregor PJ (eds): Surgical Treatment of Orthopaedic Trauma. New York, NY, Thieme,
2007, pp 179-207.
Spivak JM, Connolly PJ (eds): Orthopaedic Knowledge Update: Spine 3. Rosemont, IL,
American Academy of Orthopaedic Surgeons, 2006, pp 201-216.
86. In providing culturally competent care to a Muslim woman with a cervical spine injury,
which of the following most accurately describes the steps a male orthopaedist should
take to respect her religious beliefs during his examination?

1- No one should be in the exam room except the patient and the physician.
2- Another woman should be in the exam room and only the affected body part
should be exposed.
3- A chaperone of either gender should be in the exam room and no skin should be
exposed.
4- No particular steps need to be taken in this case.
5- The patients closest male relative should be in the exam room but a standard
hospital gown may be used.

PREFERRED RESPONSE: 2

DISCUSSION: In examining a traditional Muslim woman, a male physician should have another
woman present, and the patients husband, if possible. Only the affected limb or area needing
examination should be exposed.

REFERENCE: Jimenez R, Lewis VO (eds): Culturally Competent Care Guidebook. Rosemont,


IL, American Academy of Orthopaedic Surgeons, 2007.
87. Figure 31 shows the radiograph of a 64-year-old woman who is seen in the emergency
department following a motor vehicle accident. She has no voluntary motor function in
her distal upper extremities or lower extremities. She does not have a bulbocavernosus
reflex. She has a blood pressure of 80/50 mm Hg with a pulse of 50/min. Her
hypotension does not improve with initial fluid resuscitation. Further treatment of her
hypotension should consist of

1- continued rapid fluid infusion.


2- administration of broad-spectrum antibiotics.
3- administration of 30/mg/kg methylprednisolone over 1 hour.
4- administration of pressors.
5- cardioversion and implantation of a pacemaker.

PREFERRED RESPONSE: 4

DISCUSSION: The hallmark of neurogenic shock is hypotension without tachycardia. It is


associated most commonly with high cervical spinal cord injuries and results from loss of
function of the sympathetic nervous system. Because the peripheral vasculature is dilated due to
loss of its sympathetic tone, continued rapid administration of fluid corrects the hypotension and
can quickly lead to fluid overload and congestive heart failure. Therefore, neurogenic shock is
best treated by the use of pressors. Cardioversion or administration of antibiotics or systemic
steroids is not appropriate treatment for this patients hypotension.

REFERENCES: Spivak JM, Connolly PJ (eds): Orthopaedic Knowledge Update: Spine 3.


Rosemont, IL, American Academy of Orthopaedic Surgeons, 2006, pp 179-187.
Nockels RP: Nonoperative management of acute spinal cord injury. Spine 2001;26:S31-S37.
88. What is the typical axial plane transverse angulation of the thoracic pedicles?

1- 5 degrees medial at T 1 and T 2 ; 10 degrees from T3 to T10


2- 5 degrees lateral at T 1 ; neutral at T 2 ; 5 degrees medial from T3 to T12
3- 10 degrees medial from T1 to T10; 15 degrees medial at T11 and T12
4- 10 degrees medial from T 1 to T12
5- 25 degrees medial at T 1 ; 15 degrees at T 2 ; and 10 degrees medial from T3 to T10

PREFERRED RESPONSE: 5

DISCUSSION: Thoracic pedicles typically are angled 25 degrees medially at T 1 so the


starting point is more lateral. T 2 angles about 15 degrees, and then the pedicles average about
5 to 7 degrees down to T10. At T11 and 12, the angulation is minimal.

REFERENCES: Weinstein L: Pediatric Spine Principles and Practice. New York, NY, Raven
Press, 1994, pp 1659-1681.
Lenke LG, Orchowski J: Segmental posterior spinal instrumentation: Thoracic spine to sacrum,
in Frymoyer JW, Wiesel SW (eds): The Adult and Pediatric Spine, ed 3. Philadelphia, PA,
Lippincott Williams and Wilkins, 2004, pp 537-552.
89. What muscle is most often encountered during surgical approaches to C5-6?

1- Omohyoid
2- Cricohyoid
3- Splenius capitus
4- Thyrohyoid
5- Posterior digastric

PREFERRED RESPONSE: 1

DISCUSSION: The omohyoid muscle crosses the surgical field from inferior lateral to anterior
superior traveling from the scapula to the hyoid bone and may need to be transected. The
posterior digastric crosses the field as well but higher near C3-4. The other muscles run
longitudinally.

REFERENCES: Chang U, Lee MC, Kim DH: Anterior approach to the midcervical spine, in
Kim DH, Henn JS, Vaccaro AR, et al (eds): Surgical Anatomy and Techniques to the Spine.
Philadelphia, PA, Saunders Elsevier, 2006, pp 45-56.
Netter GH: Atlas of Human Anatomy. Summit, NJ, Ciba-Geigy Corporation, 1989.
90. Which of the following lumbar disk components has the highest tensile modulus to resist
torsional, axial, and tensile loads?

1- Nucleus pulposus
2- Cartilaginous end plate
3- Anterior longitudinal ligament
4- Annulus fibrosis
5- Cellular matrix

PREFERRED RESPONSE: 4

DISCUSSION: The annulus fibrosis has a multilayer lamellar architecture mode of type I
collagen fibers. Each successive layer is oriented at 30 degrees to the horizontal in the opposite
direction, leading to a criss-cross type pattern. This composition allows the annulus, which
has the highest tensile modulus, to resist torsional, axial, and tensile loads.

REFERENCE: Rhee JM, Schaufele M, Abdu WA: Radiculopathy and the herniated lumbar disc:
Controversies regarding pathophysiology and management. J Bone Joint Surg Am
2006;88:2070-2080.
91. When comparing the overall outcomes of surgical versus nonsurgical treatment of stable
thoracolumbar burst fractures in patients without neurologic injury, 5 years following
injury, the principle differences lie in

1- fracture kyphosis.
2- reduction of retropulsed bone.
3- pain reduction.
4- incidence of complications.
5- return to work.

PREFERRED RESPONSE: 4

DISCUSSION: When patients are compared at 5 years follow-up, there are no statistically
significant differences between the two groups with respect to kyphosis, the degree of
retropulsed bone resorption, pain and function levels, or the ability to return to work.
Nonsurgical management of stable neurologically intact burst fractures has a very low incidence
of complications.

REFERENCES: Wood K, Butterman G, Mehbod A, et al: Operative compared with


nonoperative treatment of a thoracolumbar burst fracture without neurological deficit: A
prospective, randomized study. J Bone Joint Surg Am 2003;85:773-781.
Shen WJ, Liu TJ, Shen YS: Nonoperative treatment versus posterior fixation for thoracolumbar
junction burst fractures without neurologic deficit. Spine 2001;26:1038-1045.
92. A 42-year-old woman who has had an 18-month history of severe low back pain is
referred to your office for surgical evaluation. She reports that the pain initially began
with right lower extremity pain and management consisted of oral analgesics,
nonsteroidal anti-inflammatory drugs, and muscle relaxants. She has seen a chiropractor
as well as a pain management specialist and she is status-post epidural steroid injections.
She has also completed exhaustive physical therapy, as she is a certified athletic trainer
and runs a health fitness program at a community hospital. Currently, she denies lower
extremity pain and her pain is isolated to her low back and is subjectively graded as 8/10,
with 10 being the worst pain she has ever experienced. The pain is interfering with her
activities of daily living and she is seeking definitive treatment. Figures 32a through 32c
show current MRI scans. Based on the current available medical literature, what is the
most appropriate treatment?

1- Continued nonsurgical management to include long-acting narcotic analgesics


2- Referral for vertebral axial decompression
3- Referral to interventional pain management for a spinal cord stimulator
4- Intradiskal electrothermal therapy (IDET) at L5-S1
5- Lumbar spinal fusion at L5-S1

PREFERRED RESPONSE: 5

DISCUSSION: The MRI scans reveal advanced degenerative disk disease at L5-S1.
Nonsurgical management has failed to provide relief and the patient is quite debilitated as a
result of her back pain. Fritzell and associates demonstrated that in a well-informed and selected
group of patients with severe low back pain, lumbar fusion can diminish pain and decrease
disability more efficiently than commonly used nonsurgical treatments. In a recent updated
Cochrane Review of surgery for degenerative lumbar spondylosis, it was noted that while Fritzell
and associates appeared to provide strong evidence in favor of fusion, a more recent trial by Brox
and associates demonstrated no difference between those patients undergoing lumbar fusion
compared to those receiving cognitive intervention and exercise. The Cochrane Review suggests
that this may reflect a difference between the control groups. Fritzell and associates compared
lumbar fusion to standard 1990s usual care, whereas Brox and associates compared lumbar
fusion to a modern rehabilitation program. Bear in mind that this patient is a certified athletic
trainer and runs a hospital health fitness department; therefore, at least for purposes of this
question, it can be assumed that she has participated in a modern rehabilitation program. The
Cochrane Review goes on to state that preliminary results of three small trials of intradiskal
electrotherapy suggest that it is ineffective and that preliminary data from three trials of disk
arthroplasty do not permit firm conclusions.

REFERENCES: Gibson JN, Waddell G: Surgery for degenerative lumbar spondylosis: Updated
Cochrane Review. Spine 2005;30:2312-2320.
Fritzell P, Hagg O, Wessberg P, et al: 2001 Volvo Award Winner in Clinical Studies: Lumbar
fusion versus nonsurgical treatment for chronic low back pain: A multicenter randomized
controlled trial from the Swedish Lumbar Spine Study Group. Spine 2001;26:2521-2532.
Brox JI, Sorensen R, Friis A, et al: Randomized clinical trial of lumbar instrumented fusion and
cognitive intervention and exercises in patients with chronic low back pain and disc
degeneration. Spine 2003;28:1913-1921.
93. Figure 33 shows the MRI scan of a 55-year-old woman who has had a 6-week history of
back and leg pain. Which of the following clinical scenarios is most consistent with the
MRI scan findings at L4-L5?

1- L4 nerve root radiculopathy


2- L5 nerve root radiculopathy
3- Associated bowel and bladder dysfunction
4- Symptoms associated with arachnoiditis
5- Wide-based gait, left-sided Hoffmans sign

PREFERRED RESPONSE: 1

DISCUSSION: The MRI scan reveals a L4-L5 foraminal disk herniation originating from the
L4-5 disk space that has migrated up into the foramen, compressing the left L4 nerve root. There
is normal distribution of the roots in the cerebrospinal fluid, excluding arachnoiditis as a
diagnosis, and disk herniation in this location would not result in cauda equina syndrome or
myelopathy.

REFERENCE: McCullouch JA, Transfeldt EE: Macnabs Backache, ed 3. Philadelphia, PA,


Williams and Wilkins, 1997, pp 569-608.
94. Intradiskal electrothermal therapy (IDET) uses an intradiskal catheter to deliver
controlled thermal energy to the inner periphery of the annulus fibrosis of a chronically
painful intervertebral disk. Lumbar diskography is used diagnostically to identify the
presumed pain generator to be targeted with IDET. Based on the medical literature, what
can be said about the current status of IDET?

1- IDET has been proven to seal annular tears in the annulus fibrosis.
2- IDET restores segmental stability by shrinking collagen fibrils in the disk.
3- IDET has demonstrated no significant benefit over placebo in controlled trials.
4- IDET is an unsafe procedure with significant risk of permanent complications.
5- IDET has demonstrated poor clinical results in all reported series to date.

PREFERRED RESPONSE: 3

DISCUSSION: Intradiskal electrothermal therapy (IDET) initial clinical results were reported in
2000. The early case series were quite encouraging with reported therapeutic success rates of
60% to 80%. Early enthusiasm was high as IDET provided a nonsurgical treatment option for an
otherwise complex and difficult clinical entity, chronic diskogenic low back pain. The actual
mechanism of action was not well understood, and while the theoretic explanation made good
sense, it did not hold up under laboratory testing. Soon clinical results from the field did not
meet the high expectations set by the developers of the technique. Since those early case studies,
a few level I evidence studies have been conducted, one by Freeman and associates and one by
Pauza and associates. These randomized, placebo-controlled trials demonstrated no significant
benefit of IDET over the placebo.

REFERENCES: Freeman BJ, Fraser RD, Cain CM, et al: A randomized, double-blind,
controlled trial: Intradiscal electrothermal therapy versus placebo for the treatment of chronic
discogenic low back pain. Spine 2005;30:2369-2377.
Pauza KJ, Howell S, Dreyfuss P, et al: A randomized, placebo-controlled trial of intradiscal
electrothermal therapy for the treatment of discogenic low back pain. Spine J 2004;4:27-35.
Wetzel FT, McNally TA: Treatment of chronic discogenic low back pain with intradiskal
electrothermal therapy. J Am Acad Orthop Surg 2003;11:6-11.
95. A 56-year-old mechanic has had pain in the hypothenar region of his dominant right hand
for the past 6 months. He reports weakness in his grip and pain is worse with activity.
Which of the following examination findings is most suggestive of a cervical etiology?

1- Relief of symptoms with shoulder abduction (placing hand over the head)
2- Hypothenar atrophy
3- Reproduction of pain with hyperflexion and contralateral rotation of the head
4- Positive Tinels sign at the levator scapulae
5- Subluxable ulnar nerve at the cubital tunnel

PREFERRED RESPONSE: 1

DISCUSSION: Hypothenar atrophy is a nonspecific sign that can be seen in ulnar neuropathy,
C8 radiculopathy, or even cervical myelopathy; however, the atrophy usually is not unilateral
and includes other muscle groups. The Spurling test is an excellent method of eliciting cervical
radicular pain but involves hyperextension and ipsilateral rotation of the cervical spine, resulting
in nerve root compression by reducing the cross-sectional area of the ipsilateral neuroforamen.
Tinels sign at the levator scapulae, if present, is indicative of an upper cervical (C3 or C4)
radiculopathy. A subluxable ulnar nerve at the cubital tunnel, while often asymptomatic, points
toward cubital tunnel syndrome as an etiology for this patients pain. The shoulder abduction
relief (SAR) sign (relief of upper extremity pain with shoulder abduction) is virtually
pathognomic of cervical radiculopathy because this maneuver results in relaxation of a
compressed and/or inflamed cervical nerve root. The SAR sign is the converse analog of the
straight leg raising sign in the lumbar examination for lumbar radiculopathy, as it relieves
tension in the nerve root, thereby relieving symptoms.

REFERENCES: Ducker TB, Zeidman SM: Neurologic and functional evaluation, in Clark CR
(ed): The Cervical Spine, ed 3. Philadelphia, PA, Lippincott Raven, 1998, pp 143-161.
An HS: Clinical presentation of discogenic neck pain, radiculopathy, and myelopathy, in Clark
CR (ed): The Cervical Spine, ed 3. Philadelphia, PA, Lippincott Raven, 1998, pp 755-764.
Hoppenfeld S: Physical examination of the cervical spine and temporomandibular joint, in
Physical Examination of the Spine and Extremities. New York, NY, Appleton-Century-Crofts,
1976, pp 105-132.
96. A 35-year-old woman reports an 8-week history of neck pain radiating to her right upper
extremity. She denies any history of trauma or provocative event. Examination reveals
decreased pinprick sensation in her right middle finger, otherwise sensation is intact
bilaterally. Finger flexors and interossei demonstrate 5/5 motor strength bilaterally.
Finger extensors are 4/5 on the right and 5/5 on the left. The triceps reflex is 1+ on the
right and 2+ on the left. The most likely diagnosis is a herniated nucleus pulposus at
what level?

1- C3-4
2- C4-5
3- C5-6
4- C6-7
5- C7-T 1

PREFERRED RESPONSE: 4

DISCUSSION: The patients neurologic examination is consistent with a C7 radiculopathy on


the right side. In a patient with this symptom complex in the absence of trauma, a cervical disk
herniation is the most common etiology for a C7 radiculopathy. There are eight cervical nerve
roots and the C7 nerve exits at the C6-7 disk space and is most frequently impinged by a disk
herniation at this level.

REFERENCES: Houten JK, Errico TJ: Cervical spondylotic myelopathy and radiculopathy:
Natural history and clinical presentation, in Clark CR (ed): The Cervical Spine, ed 4.
Philadelphia, PA, Lippincott Williams & Wilkins, 2005, pp 985-990.
Hoppenfeld S: Orthopaedic Neurology: A Diagnostic Guide to Neurologic Levels. Philadelphia,
PA, JB Lippincott, 1977, pp 7-43.
97. What is the most common nonanesthetic-related reversible cause of changes in
intraoperative neurophysiologic monitoring data?

1- Pedicle screw misplacement


2- Patient positioning
3- Spinal cord ischemia
4- Retractor placement
5- Hypotension

PREFERRED RESPONSE: 2

DISCUSSION: Patient positioning that results in local nerve compression, plexus traction, or
improper neck alignment is the most common nonanesthetic-related cause of changes in
intraoperative neurophysiologic monitoring data during spinal surgery.

REFERENCES: Jones SC, Fernau R, Woeltjen BL: Use of somatosensory evoked potentials to
detect peripheral ischemia and potential injury resulting from positioning of the surgical patient:
Case reports and discussion. Spine J 2004;4:360-362.
Schwartz DM, Sestokas AK, Hilibrand AS, et al: Neurophysiological identification of position-
induced neurologic injury during anterior cervical spine surgery. J Clin Monit Comput
2006;20:437-444.
98. During a left-sided transforaminal lumbar interbody fusion at the L4-5 level, the surgeon
notes a significant amount of bleeding that cannot be controlled while using a pituitary
rongeur. What anatomic structure has been injured?

1- Aorta
2- Common iliac artery
3- Common iliac vein
4- External iliac artery
5- External iliac vein

PREFERRED RESPONSE: 2

DISCUSSION: The surgeon perforated the anterior longitudinal ligament and injured the
common iliac artery. Bingol and associates described injuries to the vascular structures during
lumbar disk surgery. The common iliac artery was most commonly affected and constituted
76.9% of injuries.

REFERENCE: Bingol H, Cingoz F, Yilmaz AT, et al: Vascular complications related to lumbar
disc surgery: J Neurosurg 2004;100:249-253.
99. Six weeks after onset, what is the most clearly accepted indication for surgical
management for lumbar disk herniation?

1- Stable sensory loss


2- Stable motor weakness
3- Refractory radicular pain
4- Size of the herniation
5- Lost time at work

PREFERRED RESPONSE: 3

DISCUSSION: In the absence of a cauda equina syndrome or progressive weakness, the best
indication for surgical management is refractory radicular pain. Surgical decision-making should
not be based on the size of the herniation. Large extruded herniations tend to resolve more
predictably than smaller herniations. Stable motor weakness and numbness resolve similarly in
both surgical and nonsurgical management, although surgery hastens the process. When
intractable radicular pain is the strict indication for surgery, surgical intervention provides
substantial and more rapid pain relief than nonsurgical care.

REFERENCES: Rhee JM, Schaufele M, Abdu WA: Radiculopathy and the herniated lumbar
disc: Controversies regarding pathophysiology and management. J Bone Joint Surg Am
2006;88:2070-2080.
Atlas SJ, Keller RB, Wu YA, et al: Long-term outcomes of surgical and nonsurgical
management of sciatica secondary to a lumbar disc herniation: 10 year results from the Maine
lumbar spine study. Spine 2005;30:927-935.
100. A 45-year-old woman has idiopathic scoliosis. Surgery is to include an anterior thoracic
release through an open left thoracotomy. The thoracotomy will have what effect on the
patients pulmonary function postoperatively?

1- Unaffected
2- Transiently reduced postoperatively but ultimately improves to greater than
preoperative function
3- Transiently reduced immediately postoperatively but then quickly returns to
preoperative levels
4- Improves postoperatively due to correction of the scoliosis and is maintained long
term
5- Reduced postoperatively and often remains reduced long term

PREFERRED RESPONSE: 5

DISCUSSION: A thoracotomy in an adult with idiopathic scoliosis causes a reduction in


pulmonary function that often does not return to preoperative levels. What pulmonary function
that does recover, recovers over many months. Long-term improvement in pulmonary function,
compared to preoperative function, is rarely seen. This should be considered in planning surgical
intervention in adults with scoliosis.

REFERENCES: Graham EJ, Lenke LG, Lowe TG, et al: Prospective pulmonary function
evaluation following open thoracotomy for anterior spinal fusion in adolescent idiopathic
scoliosis. Spine 2000;25:2319-2325.
Kishan S, Bastrom T, Betz RR, et al: Thoracoscopic scoliosis surgery affects pulmonary function
less than thoracotomy at 2 years postsurgery. Spine 2007;32:453-458.

Вам также может понравиться